100

95

75

B΄ΕΥΚΛΕΙΔΗΣ ΑΣΚΗΣΕΙΣ

ΔΙΑΓΩΝΙΣΜΟΙ ΟΛΥΜΠΙΑΔΕΣ

ΓΕΓΟΝΟΤΑ

Μ α θ η μ α τ ι κό π ε ρ ι ο δ ι κό γ ι α τ ο ΛΥ Κ Ε Ι Ο

25

5

0

90

100

95

75

25

5

0

ΟΚΤΩΒΡΙΟΣ - ΝΟΕΜΒΡΙΟΣ - ΔΕΚΕΜΒΡΙΟΣ 20 1 3 ευρώ 3,5

95 Χρόνια Ε.Μ.Ε.

Ο κομήτης του αιώνα ISON κάηκε πλησιάζοντας στον Ήλιο σε απόσταση 1.174.821 km και σε θερμοκρασίες 2700ο C.

= “Θαλής” 2013 = “Μεσογειάδα” 2013

= Τα Μαθηματικά του Σύμπαντος = Ελληνική διάκριση στο “Le- math” eatre

= Απολλώνιος ο Περγαίος

100

95

ΠΛΗΡΩΜΕΝΟ ΤΕΛΟΣ

Ταχ. Γραφείο

ΚΕΜΠ.ΑΘ.

Αριθμός Άδειας

75

ΕΚΔΟΤΩΝ

100

95

ΕΛΤΑ Hellenic Post

4156

75

ΕΝΤΥΠΟ ΚΛΕΙΣΤΟ ΑΡ. ΑΔΕΙΑΣ 1099/96 ΚΕΜΠ.ΑΘ.

25

Ελληνικh

Μαθηματικh

Εταιρεiα

25

5

5

0

0

periexb90.qxp

20/12/2013

1:29

Page 1

ÅËËÇÍÉÊÇ ÌÁÈÇÌÁÔÉÊÇ ÅÔÁÉÑÅÉÁ Ôåý÷ïò 90 - Ïêôþâñéïò - ÍïÝìâñéïò - Äåêåìâñéïò 2013- Åõñþ: 3,50

Åðéêïéíùíßá: Ì Á È Ç Ì Á Ô É Ê Ï

ÃåíéêÜ èÝìáôá

               

e-mail: [email protected] - www.hms.gr

Ð Å Ñ É Ï Ä É Ê Ï

1 4 7 11 22 26 32 37 42 46 53 59 66 71 75 76

Áãáðçôïß ìáèçôÝò êáé óõíÜäåëöïé, Âñéóêüìáóôå óôçí åõ÷Üñéóôç èÝóç íá óáò ðáñáäþóïõìå ôï äåýôåñï ôåý÷ïò ôïõ Åõêëåßäç ´ ðñéí áðü ôéò åïñôÝò ôùí ×ñéóôïõãÝííùí. ÌåôÜ ôï åðéôõ÷çìÝíï óõíÝäñéï êáé ôéò åíäéáöÝñïõóåò åñãáóßåò ôùí óõíáäÝëöùí ôïõ ðáñáñôÞìáôïò Êáñäßôóáò ðïõ öéëïîåíÞóáìå óôï ðñþôï ôåý÷ïò ôïõ ðåñéïäéêïý, öñïíôßóáìå êáé áõôü ôï ôåý÷ïò íá ðåñéÝ÷åé åñãáóßåò ðïõ íá áíôáðïêñßíïíôáé óôéò áíÜãêåò ôùí ìáèçôþí êáé íá ðñïóöÝñïõí åíäéáöÝñïí õëéêü ãéá íá ôï áîéïðïéÞóïõí ïé óõíÜäåëöïé. " Óõã÷ñüíùò, îåêéíÞóáìå ìéá ðñïóðÜèåéá ãéá óõãêÝíôñùóç èåìÜôùí Á´ Ëõêåßïõ ðñïêåéìÝíïõ íá äçìéïõñãÞóïõìå ôñÜðåæá èåìÜôùí áíôßóôïé÷ç åêåßíçò ôçò ô Ëõêåßïõ ðïõ äçìéïýñãçóå êáé ëåéôïõñãåß ç ÅÌÅ åäþ êáé äÝêá ÷ñüíéá. Óôçí ðñïóðÜèåéÜ ìáò áõôÞ ðåñéìÝíïõìå êáé äéêÜ óáò èÝìáôá. " Èõìßæïõìå üôé ôï ðñþôï ôåý÷ïò ôïõ Óåðôåìâñßïõ ôïõ 2014, ôï Ý÷åé áíáëÜâåé ôï ÐáñÜñôçìá ÂÝñïéáò, üðïõ êáé èá ãßíåé ôï åðüìåíï óõíÝäñéï ôçò ÅÌÅ ôï ÍïÝìâñéï 2014. Ìå ôçí åõêáéñßá ôùí åïñôþí ôùí ×ñéóôïõãÝííùí êáé ôçò Ðñùôï÷ñïíéÜò óáò åõ÷üìáóôå õãåßá, åõôõ÷ßá êáé ï êáéíïýñãéïò ÷ñüíïò íá åßíáé ÷áñïýìåíïò êáé äçìéïõñãéêüò. Ï ðñüåäñïò ôçò ÓõíôáêôéêÞò ÅðéôñïðÞò: Ãéþñãïò Ôáóóüðïõëïò Ï áíôéðñüåäñïé: ÂáããÝëçò Åõóôáèßïõ, ÃéÜííçò Êåñáóáñßäçò

19 Ïêôùâñßïõ 2013 18 Éáíïõáñßïõ 2014 22 Öåâñïõáñßïõ 2014

Åîþöõëëï: Öùôïãñáößåò áðü ôï ISON óôçí áôìüóöáéñá ôïõ Çëéáêïý ìáò óõóôÞìáôïò Èõìßæïõìå üôé êÜèå ÓÜââáôï ãßíïíôáé ÄÙÑÅÁÍ ìáèÞìáôá, ãéá ôïõò äéáãùíéóìïýò, óôá ãñáöåßá ôçò Å.Ì.Å. ¸íáñîç áðü 21 Óåðôåìâñßïõ 2013 ÓõíôáêôéêÞ åðéôñïðÞ

ÅÊÄÏÓÇ ÔÇÓ ÌÁÈÇÌÁÔÉÊÇÓ ÅÔÁÉÑÅÉÁÓ

ÅêôåëåóôéêÞ Ãñáììáôåßá

Ðñüåäñïò: Ôáóóüðïõëïò Ãéþñãïò Áíôéðñüåäñïé: Åêäüôçò: Åõóôáèßïõ ÂáããÝëçò ÄçìÜêïò Ãåþñãéïò Êåñáóáñßäçò ÃéÜííçò ÄéåõèõíôÞò: ÃñáììáôÝáò: Êñçôéêüò ÅììáíïõÞë ×ñéóôüðïõëïò Ðáíáãéþôçò ÅðéìÝëåéá ¸êäïóçò: ÌÝëç: ÁñãõñÜêçò Ä. Æþôïò ÂáããÝëçò Ëïõñßäáò Ó. Kùäéêüò EË.ÔÁ.: 2055 ÓôåöáíÞò Ð. Ôáðåéíüò Í. ISSN: 1105 - 7998 Ó÷üëéï: Ïé åñãáóßåò ãéá ôï ðåñéïäéêü óôÝëíïíôáé Ôçë.: 210 3617784 - 3616532 Fax: 2103641025

êáé çëåêôñïíéêÜ óôï

e-mail: [email protected].

×ÏÑÇÃÏÓ ÅÊÄÏÓÇÓ

Ë Õ Ê Å É Ï

ÃñÜììá ôçò Óýíôáîçò

Ìáèçìáôéêïß Äéáãùíéóìïß:

ÐÁÍÅÐÉÓÔÇÌÉÏÕ 34 106 79 ÁÈÇÍÁ

Ô Ï

ÐÅÑÉÅ×ÏÌÅÍÁ

Áðïëëþíéïò Nobel êáé Âñáâåßá Ìáèçìáôéêþí Âéïëïãßá êáé ïé ðñþôïé áñéèìïß ÌáèçìáôéêÝò ÏëõìðéÜäåò Homo Mathematicus Á´ ÔÜîç ¢ëãåâñá: Åîéóþóåéò êáé Áíéóþóåéò Ãåùìåôñßá: Ðáñáëëçëüãñáììá - ÔñáðÝæéá B´ ÔÜîç ¢ëãåâñá: Ðïëõþíõìá - Ôñéãùíïìåôñßá Ãåùìåôñßá: ÅìâáäÜ Êáôåýèõíóç: Åõèåßá êáé êýêëïò ô ÔÜîç ÃåíéêÞ Ðáéäåßá: ÓôáôéóôéêÞ Êáôåýèõíóç: ÐáñÜãùãïò ÔñÜðåæá ÈåìÜôùí Ôï ÂÞìá ôïõ Åõêëåßäç - ÉóôïñéêÝò ÁíáöïñÝò Åêëåßäçò Ðñïôåßíåé ... Áëëçëïãñáößá - Åðéêáéñüôçôá - ÓõíÝäñéï

ÈáëÞò: ÈáëÞò Åõêëåßäçò: Åõêëåßäçò Áñ÷éìÞäçò: Áñ÷éìÞäçò

Ã É Á

Áèáíáóüðïõëïò Ãåþñãéïò ÁíäñïõëáêÜêçò Íßêïò Áíôùíüðïõëïò Ãåþñãéïò Áíôùíüðïõëïò Íßêïò ÁñãõñÜêçò ÄçìÞôñéïò Âáêáëüðïõëïò Êþóôáò ÃáâñÜò ÔÜóïò Åõóôáèßïõ ÂáããÝëçò Æá÷áñüðïõëïò Êùí/íïò Æþôïò ÂáããÝëçò ÈåïäùñÝëïò ×ñÞóôïò ÊáêêáâÜò Áðüóôïëïò Êáëßêáò ÓôáìÜôçò Êáìðïýêïò ÊõñéÜêïò ÊáíÝëëïò ×ñÞóôïò ÊáñêÜíçò Âáóßëçò Êáôóïýëçò Ãéþñãïò Êåñáóáñßäçò ÃéÜííçò Êáñäáìßôóçò Óðýñïò ÊëÜäç Êáôåñßíá Êïíüìçò ¢ñôé Êüíôæéáò Íßêïò ÊïôóéöÜêçò Ãéþñãïò

ÊõñéáæÞò ÉùÜííçò Êõñéáêüðïõëïò Áíôþíçò Êõñéáêoðïýëïõ Êùí/íá ÊõâåñíÞôïõ ×ñõóô. Ëáæáñßäçò ×ñÞóôïò ËÜððáò ËåõôÝñçò ËïõñéäÜò ÃéÜííçò Ëïõñßäáò ÓùôÞñçò ÌáëáöÝêáò ÈáíÜóçò ÌáíéÜôçò ÁíäñÝáò ÌáõñïãéáííÜêçò Ëåùíßäáò Ìåíäñéíüò ÃéÜííçò ÌåôáîÜò Íéêüëáïò ÌÞëéïò Ãåþñãéïò Ìðåñóßìçò Öñáãêßóêïò Ìðñßíïò Ðáíáãéþôçò ÌõëùíÜò ÄçìÞôñçò Ìþêïò ×ñÞóôïò ÐáíäÞò ×ñÞóôïò Óßóêïõ Ìáñßá ÓáÀôç Åýá Óôáúêïò Êþóôáò

ÓôÜúêïò Ðáíáãéþôçò ÓôåöáíÞò Ðáíáãéþôçò ÓôñáôÞò ÃéÜííçò Ôáðåéíüò Íéêüëáïò Ôáóóüðïõëïò Ãéþñãïò ÔæåëÝðçò ¢ëêçò Ôæéþôæéïò ÈáíÜóçò ÔñéáíôÜöõëëïò Ãéþñãïò ÔñéÜíôïò Ãåþñãéïò ÔóáãêÜñçò ÁíäñÝáò ÔóáãêÜñçò Êþóôáò ÔóéêáëïõäÜêçò Ãéþñãïò Ôóéïýìáò ÈáíÜóçò ÔõñëÞò ÉùÜííçò ÖáíÝëç ¢ííõ ×áñáëáìðÜêçò ÅõóôÜèéïò ×áñáëáìðïðïýëïõ Ëßíá ×áñáëÜìðïõò ÈÜíïò ×ñéóôéÜò Óðýñïò ×ñéóôüðïõëïò ÈáíÜóçò ×ñéóôüðïõëïò Ðáíáãéþôçò Øý÷áò ÂáããÝëçò

Ç Ýãêáéñç ðëçñùìÞ ôçò óõíäñïìÞò âïçèÜåé óôçí Ýêäïóç ôïõ ðåñéïäéêïý • Ôá äéáöçìéæüìåíá âéâëßá äå óçìáßíåé üôé ðñïôåßíïíôáé áðü ôçí Å.Ì.Å. • Ïé óõíåñãáóßåò, [ôá Üñèñá, ïé ðñïôåéíüìåíåò áóêÞóåéò, ïé ëýóåéò áóêÞóåùí êëð.] ðñÝðåé íá óôÝëíïíôáé Ýãêáéñá, óôá ãñáöåßá ôçò

Å.Ì.Å. ìå ôçí Ýíäåéîç “Ãéá ôïí Åõêëåßäç B´”. Ôá ÷åéñüãñáöá äåí åðéóôñÝöïíôáé.

ÔéìÞ Ôåý÷ïõò: åõñþ 3,50

ÅôÞóéá óõíäñïìÞ (12,00 + 2,00 Ôá÷õäñïìéêÜ = åõñþ 14,00). ÅôÞóéá óõíäñïìÞ ãéá Ó÷ïëåßá åõñþ 12,00 Ôï áíôßôéìï ãéá ôá ôåý÷ç ðïõ ðáñáããÝëíïíôáé óôÝëíåôáé: (1). Ìå áðëÞ ôá÷õäñïìéêÞ åðéôáãÞ óå äéáôáãÞ Å.Ì.Å. Ôá÷. Ãñáöåßï ÁèÞíá 54 Ô.È. 30044 (2). Óôçí éóôïóåëßäá ôçò Å.Ì.Å., üðïõ õðÜñ÷åé äõíáôüôçôá ôñáðåæéêÞò óõíáëëáãÞò ìå ôçí ôñÜðåæá EUROBANK (3). Ðëçñþíåôáé óôá ãñáöåßá ôçò Å.Ì.Å. (4). Ìå áíôéêáôáâïëÞ, óå åôáéñåßá ôá÷õìåôáöïñþí óôï ÷þñï óáò, êáôÜ ôçí ðáñáëáâÞ. Åêôýðùóç: ROTOPRINT (A. ÌÐÑÏÕÓÁËÇ & ÓÉÁ ÅÅ). ôçë.: 210 6623778 - 358 Õðåýèõíïò ôõðïãñáöåßïõ: Ä. Ðáðáäüðïõëïò

“Περί Επαφών” του Απολλώνιου Σωτήρης Χρ. Γκουντουβάς Ο Απολλώνιος ο Περγαίος είναι ο τρίτος κατά σειρά μεγάλος αρχαίος έλληνας μαθηματικός μετά τον Ευκλείδη και τον Αρχιμήδη. Γεννήθηκε στη Μικρά Ασία, στην Πέργη της Παμφυλίας γύρω στο 260 π.Χ και ήταν λίγο νεώτερος από τον Αρχιμήδη (278-212 π.Χ). Σπούδασε στην Αλεξάνδρεια, πήγε μετά στην Έφεσο και στη συνέχεια εγκαταστάθηκε στην Πέργαμο που λειτουργούσε φημισμένο Πανεπιστήμιο και Βιβλιοθήκη. Επέστρεψε κατόπιν στην Αλεξάνδρεια και συνεργάστηκε με τους διαδόχους του Ευκλείδη. Δίδαξε για πολλά χρόνια στο Μουσείο, στην Αίθουσα Ε΄, για αυτό και τον αποκαλούσαν Απολλώνιος ο Ε΄. Ο Πάππος ο Αλεξανδρινός (3ος μ.Χ αιώνας) τον περιγράφει ως ματαιόδοξο και υπερόπτη αναφέροντας ότι υποτιμούσε τους άλλους Γεωμέτρες. Ίσως αυτός να ήταν ο λόγος που ο Αρχιμήδης είχε με τους Αλεξανδρινούς μαθηματικούς φιλικές σχέσεις, κάτι που δεν συνέβαινε με τον Απολλώνιο. Μάλιστα κάποιες εργασίες του τις αφιέρωσε σε αυτούς με πιο γνωστή αυτή στον Ερατοσθένη (276-194 π.Χ) με τίτλο «Περί των μηχανικών θεωρημάτων προς Ερατοσθένη έφοδος». Ο Απολλώνιος συνέγραψε πολλά έργα αλλά σώθηκαν μόνο τέσσερα. Το σπουδαιότερο έργο του ήταν τα Κωνικά με 8 βιβλία. Εκεί γίνεται μια εκτενέστατη μελέτη των Κωνικών Τομών. Μάλιστα τις ονομασίες Παραβολή, Έλλειψη και Υπερβολή τις καθιέρωσε ο Απολλώνιος, ενώ λίγο πιο πριν ο Αρχιμήδης τις ονόμαζε αντίστοιχα Ορθογωνίου Κώνου Τομή, Οξυγωνίου Κώνου Τομή και Αμβλυγωνίου Κώνου Τομή. Τα Κωνικά είχαν τεράστια απήχηση και καθιέρωσαν τον Απολλώνιο σε τέτοιο βαθμό ώστε οι σύγχρονοί του να τον αποκαλούν «Μέγα Γεωμέτρη». Τα άλλα τρία σωζόμενα έργα είναι: Περί λόγου αποτομής (2 βιβλία), Κατασκευή δύο μέσων αναλόγων και Σύγκριση Δωδεκάεδρου – Εικοσάεδρου. Ο Απολλώνιος πέθανε σε μεγάλη ηλικία γύρω στο 180 π.Χ. Ένα έργο του Απολλώνιου που δεν έχει σωθεί, όμως έφτασε σε μας από μεταγενέστερους σχολιαστές και συγγραφείς, είναι η πραγματεία “Περί επαφών”. Η πραγματεία αυτή του Απολλώνιου αναφέρεται σε γεωμετρικές κατασκευές. Το κατασκευαστικό πρόβλημα που τίθεται είναι το εξής: «Δοθέντων τριών στοιχείων, που το καθένα μπορεί να είναι σημείο, ευθεία ή κύκλος, να γραφεί κύκλος ο οποίος να διέρχεται από τα ση-

μεία και να εφάπτεται στις ευθείες ή τους κύκλους». Το πρόβλημα αυτό περιλαμβάνει δέκα επιμέρους προβλήματα και όλα λύνονται με κανόνα και διαβήτη. Τα δέκα προβλήματα είναι τα πιο κάτω. Να κατασκευαστεί κύκλος ο οποίος να : 1. 2. 3. 4. 5. 6.

Διέρχεται από τρία σημεία Εφάπτεται σε τρεις ευθείες Διέρχεται από δύο σημεία και εφάπτεται σε μία ευθεία Διέρχεται από ένα σημείο και εφάπτεται σε δύο ευθείες Διέρχεται από δύο σημεία και εφάπτεται σε ένα κύκλο Διέρχεται από ένα σημείο και εφάπτεται σε ευθεία και σε κύκλο 7. Διέρχεται από ένα σημείο και εφάπτεται σε δύο κύκλους 8. Εφάπτεται σε δύο ευθείες και ένα κύκλο 9. Εφάπτεται σε δύο κύκλους και μία ευθεία 10. Εφάπτεται σε τρεις κύκλους.

Το 1ο πρόβλημα έχει για λύση τον περιγεγραμμένο κύκλο του τριγώνου που ορίζουν τα τρία σημεία. Το κέντρο του ζητούμενου κύκλου είναι το περίκεντρο του τριγώνου. Το περίκεντρο ως σημείο τομής των μεσοκαθέτων των πλευρών ισαπέχει από τα άκρα των πλευρών άρα ισαπέχει από τις κορυφές του τριγώνου. Βέβαια το πρόβλημα έχει λύση αν τα σημεία είναι μη συνευθειακά.

Το 2ο πρόβλημα έχει 3 περιπτώσεις:  Αν οι ευθείες τέμνονται ανά δύο τότε λύση του προβλήματος είναι ο εγγεγραμμένος κύκλος του τριγώνου που ορίζουν οι τρεις ευθείες. Το κέντρο του ζητούμενου κύκλου είναι το έγκεντρο του τριγώνου, δηλαδή το σημείο τομής των διχοτόμων. Επίσης λύση στο πρόβλημα είναι και οι 3 παρεγγεγραμμένοι κύκλοι του τριγώνου.

Παρεγγεγραμμένος είναι ο κύκλος που εφάπτεται σε μια πλευρά του τριγώνου και στις προεκτάσεις άλλων δύο. Τα κέντρα αυτών των κύκλων είναι τα 3 παράκεντρα του τριγώνου Ια, Ιβ, Ιγ.

ΕΥΚΛΕΙΔΗΣ Β΄ 90 τ.2/1

------------------------------------------------------------------------------------------------------------------- “Περί Επαφών” του Απολλώνιου ---------------------------------------------------------------------------------------------------------------Παράκεντρο του τριγώνου είναι το σημείο τομής δύο εξωτερικών διχοτόμων του τριγώνου. Δηλαδή το παράκεντρο Ια είναι το σημείο τομής των διχοτόμων των εξωτερικών γωνιών Β και Γ.

 Αν δύο ευθείες είναι παράλληλες και τέμνουν την τρίτη τότε έχουμε δύο κύκλους που εφάπτονται στις ευθείες.

 Αν και οι τρεις ευθείες είναι παράλληλες ή συντρέχουν τότε το πρόβλημα δεν έχει λύση. Το 2ο πρόβλημα λοιπόν έχει 4, 2, ή καμία λύση. Θα

διαπραγματευθούμε τώρα την κατασκευή του 3ου προβλήματος. Έστω ότι τα δοθέντα στοιχεία είναι τα σημεία Α, Β και η ευθεία ε. Ζητείται λοιπόν να κατασκευάσουμε τον κύκλο που εφάπτεται στην ευθεία ε και διέρχεται από τα σημεία Α και Β. Ανάλυση: Έστω (Κ,ρ) ο ζητούμενος κύκλος που διέρχεται από το σημεία Α, Β και εφάπτεται στην ευθεία ε στο σημείο Δ. Αν η ευθεία ΑΒ τέμνει την ευθεία ε στο σημείο Γ τότε η δύναμη του σημείου Γ ως προς τον κύκλο δίνει ΓΔ2 = ΓΒΓΑ.

Τα τμήματα ΓΒ και ΓΑ είναι σταθερά οπότε μπορούμε να προσδιορίσουμε το σημείο Δ στην ευθεία ε που να ικανοποιεί την σχέση ΓΔ2=ΓΒΓΑ. Κατασκευή: Φέρουμε την ευθεία ΑΒ που τέμνει την ευθεία ε στο σταθερό σημείο Γ. Η δύναμη του σημείου Γ ως προς τον κύκλο είναι ΓΔ2 = ΓΒΓΑ. Κατασκευάζουμε το τμήμα ΓΔ (κατασκευή μέσης αναλόγου των τμημάτων ΓΒ, ΓΑ) στην ευθεία ε και έτσι προσδιορίζουμε το σταθερό σημείο Δ, που είναι το σημείο επαφής της ευθείας ε και του κύκλου. Το πρόβλημα τώρα ανάγεται

στην πρώτη περίπτωση, δηλαδή ζητείται να κατασκευαστεί ο κύκλος που να διέρχεται από τρία σταθερά σημεία τα Α, Β και Δ. Προσδιορίζουμε το περίκεντρο Κ του τριγώνου ΑΒΔ σαν σημείο τομής των μεσοκάθετων δύο πλευρών του τριγώνου. Κατασκευάζουμε τώρα τον κύκλο με κέντρο το Κ και ακτίνα ρ=ΚΑ. Διερεύνηση: Το πρόβλημα δέχεται δύο, μία ή καμία λύση.  Αν η ευθεία ΑΒ δεν είναι παράλληλη στην ε τότε το πρόβλημα έχει δύο λύσεις  Αν η ευθεία ΑΒ είναι παράλληλη στην ε τότε το πρόβλημα έχει μία λύση  Αν τα σημεία Α και Β βρίσκονται πάνω στην ευθεία ε ή αν τα Α, Β είναι εκατέρωθεν της ε τότε το πρόβλημα δεν έχει λύση.

Ας δούμε τώρα την κατασκευή της μέσης αναλόγου δύο δοθέντων τμημάτων α και β. Ζητούμε δηλαδή την κατασκευή ενός τμήματος x τέτοιο ώστε x2 = αβ. Έστω ότι ΓΑ=α και ΓΒ=β. Πάνω σε ευθεία ε τοποθετούμε διαδοχικά τα τμήματα α, β και γράφουμε τον κύκλο με διάμετρο δ=α+β. Φέρουμε από το Γ κάθετη ευθεία στην ΑΒ που τέμνει τον κύκλο στο Δ. Τότε το ζητούμενο τμήμα x είναι το ΓΔ. Αυτό αποδεικνύεται αμέσως από τις μετρικές σχέσεις στο ορθογώνιο τρίγωνο ΑΔΒ (ΑΔΒ=90) που έχουμε ότι ΓΔ2 = ΓΑΓΒ.

Θα διαπραγματευθούμε τέλος και την κατασκευή του 6ου προβλήματος. Έστω ότι τα δοθέντα στοιχεία είναι το σημείο Α, η ευθεία ε και ο κύκλος (Κ,ρ). Ανάλυση: Έστω Ο το κέντρο του ζητούμενου κύκλου και x η ακτίνα του. Φέρουμε την ΟΒε. Φέρουμε την ΚΕε που τέμνει τον κύκλο στα Δ και Η. Από το σταθερό σημείο Δ φέρουμε τη ΔΑ που τέμνει τον κύκλο (Ο,x) στο Ζ και τη ΔΒ που τέμνει τον κύκλο (Κ,ρ) στο Γ. Η δύναμη του Δ ως προς τον κύκλο (Ο,x) θα δώσει ΔΑΔΖ=ΔΒΔΓ (1)

Το τετράπλευρο ΒΓΗΕ έχει δύο γωνίες ορθές

ΕΥΚΛΕΙΔΗΣ Β΄ 90 τ.2/2

------------------------------------------------------------------------------------------------------------------- “Περί Επαφών” του Απολλώνιου ----------------------------------------------------------------------------------------------------------------

(Γ και Ε), άρα η δύναμη του Δ ως προς τον περιγεγραμμένο του κύκλο δίνει ΔΕΔΗ = ΔΒΔΓ (2). Από τις (1)&(2) έχουμε : ΔΑΔΖ = ΔΕΔΗ 

   . Άρα το ΔΖ είναι η τέταρτη ανάλογος  

των σταθερών τμημάτων ΔΑ, ΔΕ και ΔΗ. Κατασκευή: Από το κέντρο Κ του δοθέντος κύκλου φέρουμε ευθεία κάθετη στην ε, που τέμνει τον κύκλο στα σημεία Δ και Η. Από το Δ φέρουμε την ευθεία ΔΑ. Κατασκευάζουμε στην ευθεία ΔΑ το τμήμα ΔΖ σαν τέταρτη ανάλογο των σταθερών τμημάτων ΔΑ, ΔΕ και ΔΗ (σχολικό βιβλίο Α΄ & Β΄ Λυκείου, σελίδα 154). Τώρα το πρόβλημα ανάγεται στην τρίτη περίπτωση, γιατί ο ζητούμενος κύκλος θα διέρχεται από τα σημεία Α, Ζ και εφάπτεται στην ευθεία ε. Διερεύνηση: Το πρόβλημα έχει γενικά δύο λύσεις, έναν κύκλο που εφάπτεται εξωτερικά στον δοθέντα κύκλο (όπως στο παραπάνω σχήμα) και έναν που ο δοθείς κύκλος εφάπτεται εσωτερικά σε αυτόν (όπως στο παρακάτω σχήμα). Σε ειδική περίπτωση το πρόβλημα έχει τέσσερις λύσεις. Αν το σημείο Α και ο κύκλος (Κ,ρ) είναι εκατέρωθεν της ευθείας ε τότε το πρόβλημα δεν έχει λύση. Το δέκατο πρόβλημα δηλαδή η κατασκευή ενός κύκλου που να εφάπτεται σε τρεις κύκλους είναι και το πιο δύσκολο. Η λύση που δίνει ο Πάππος γίνεται με αναγωγή στο έβδομο πρόβλημα. Οι περιπτώσεις του προβλήματος είναι πολλές ανάλογα με το αν οι κύκλοι τέμνονται ή εφάπτονται και αν είναι εσωτερικοί ή εξωτερικοί μεταξύ τους. Επίσης και οι λύσεις σε κάθε περίπτωση είναι πολλές. Στην περίπτωση που έχουμε τρεις κύκλους εξωτερικούς μεταξύ τους που τα κέντρα τους είναι μη συνευθειακά τότε το πρόβλημα έχει 8 λύσεις. Στα παρακάτω σχήματα έχουμε 4 από τις 8 λύσεις αυτού του προβλήματος.

Κωνικά του Απολλώνιου σε αραβικό αντίγραφο

Σχήμα 1

Σχήμα 2

Σχήμα 3

Σχήμα 4 Οι 8 λύσεις του προβλήματος είναι ο κύκλος :  Να εφάπτεται εξωτερικά και στους τρεις κύκλους (σχήμα 1)  Να εφάπτεται εξωτερικά σε δύο κύκλους και ο τρίτος εσωτερικά σε αυτόν (σχήμα 2). Έχουμε 3 περιπτώσεις εναλλάσσοντας τους κύκλους.  Να εφάπτεται εξωτερικά σε ένα κύκλο και οι άλλοι δύο εσωτερικά σε αυτόν (σχήμα 3). Έχουμε 3 περιπτώσεις εναλλάσσοντας τους κύκλους  Οι τρεις κύκλοι να εφάπτονται εσωτερικά σε αυτόν (σχήμα 4).

Με το δέκατο πρόβλημα είχαν ασχοληθεί πολλοί μεγάλοι μαθηματικοί όπως οι Vieta, Newton, Euler, Poncelet, Gergone, Bodillier, Mannheim, Fouche που το έλυσαν με πολλούς διαφορετικούς τρόπους με Ευκλείδεια αλλά και Αναλυτική Γεωμετρία. Μάλιστα το 1892 ο E. Lemoine ταξινόμησε τις λύσεις του προβλήματος με βάση την απλότητα και την ακρίβεια της κατασκευής. Κλείνοντας το άρθρο θα σταθούμε στην επιστημονική πληρότητα του έργου του Απολλώνιου που όχι μόνο εξάντλησε όλες τις περιπτώσεις του προβλήματος, αλλά, έδωσε και τις αντίστοιχες κατασκευές.

ΕΥΚΛΕΙΔΗΣ Β΄ 90 τ.2/3

 

Nobel και βραβεία Μαθηματικών Τα Βραβεία Nobel είναι ετήσια διεθνή βραβεία που δίνονται από το 1901 για επιτεύγματα στην Ιατρική, στην Χημεία, στην Φυσική, στην Λογοτεχνία και για την Ειρήνη. Το 1968 η Σουηδική Τράπεζα καθιέρωσε βραβείο στις Οικονομικές επιστήμες στη μνήμη του Σουηδού A. Nobel. Κάθε βραβείο απαρτίζεται από ένα μετάλλιο, ένα σημαντικό χρηματικό ποσό και ένα δίπλωμα. Ο Nobel [1833-1896] ήταν χημικός, μηχανικός και βιομήχανος, γιος του επίσης μηχανικού και βιομήχανου Ε. Nobel., εφευρέτη της υποβρύχιας νάρκης. Σπούδασε στη Ρωσία και την Αμερική. Για τα Μαθηματικά δεν καθιέρωσε βραβείο και ο λόγος δεν είναι γνωστός. Ίσως να είναι αλήθεια, κάποιες από τις πικάντικες ιστορίες που λέγονται, αν και είχε καλή σχέση με τα Μαθηματικά.

Αναγνωρισμένα διεθνώς βραβεία ή μετάλλια που δίνονται σε μαθηματικούς μερικά από αυτά δίνονται και στα οικονομικά. Βραβεία (Abel, Adams, Alfréd Rényi, Bartolozzi, Bôcher Memorial, Caccioppoli, Carl Friedrich Gauss, Chauvenet, Clay Mathematics Institute's Millennium Problems, Clay Research Award, Cole, Erdős, Fermat, Frederic Esser Nemmers, Fröhlich, Fulkerson , J. H. Wilkinson for Numerical Software, Geometry, Heinz Hopf , Infosys in Mathematical Science – Mathematics and statistics (Infosys Science Foundation), John Bates Clark – Economics, John von Neumann – Economics, John von Neumann Theory – Systems science, Leslie Fox for Numerical Analysis, Leroy P. Steele, Lobachevsky, Loève, Nevanlinna, Norbert Wiener in Applied Mathematics, Ostrowski , Oswald Veblen in Geometry, Pólya, Ramanujan, Rolf Schock in Mathematics, Salem, Shaw, Wolf, το 2002 θεσπίστηκε και το βραβείο Kristie Strom Boole). Μετάλλια ( Chern, De Morgan, Euler, Fields, Kirkman, John Bates Clark – Economics, Stampacchia ) Ενδεικτικά αναφέρουμε μερικά:  Θυμίζουμε ότι το βραβείο Shaw το 2011 πήρε ο Έλληνας μαθηματικός και φυσικός Δημήτρης Χριστοδούλου.  Το μετάλλιο Fields δίνεται κάθε τέσσερα χρόνια και βάζει όριο ηλικίας. Ο Wiles έχασε το μετάλλιο Fields επειδή ήταν 40 ετών όταν ανακοίνωσε την απόδειξη (21-6-1993, αφού γεννήθηκε 11-4-1953) για το τελευταίο Θεώρημα του Fermat.  Το βραβείο Abel 2013 δόθηκε στο Βέλγο μαθηματικό Pierre Deligne για τη σημαντική συνεισφορά του στην αλγεβρική γεωμετρία, τη θεωρία των αριθμών και άλλα συναφή πεδία. Ο 68χρονος Deligne είναι ομότιμος καθηγητής του Ινστιτούτου Προωθημένων Σπουδών του πανεπιστημίου Πρίνστον, το 1970 είχε διοριστεί στο Ινστιτούτο Ανωτέρων Επιστημονικών Σπουδών της Γαλλίας. Το 1978 πήρε το μετάλλιο Fields επειδή το 1974 είχε αποδείξει την τελευταία και πιο δύσκολη από τις λεγόμενες «Εικασίες Weil». Έχει συγγράψει πάνω από 100 μαθηματικές εργασίες και διάφορες μαθηματικές έννοιες ήδη φέρουν τιμητικά το όνομά του. Μεταξύ άλλων, είναι επίτιμο μέλος των Μαθηματικών Εταιρειών του Λονδίνου και της Μόσχας. Το Βραβείο Abel δίνεται κάθε χρόνο στο Όσλο από τη Νορβηγική Ακαδημία Επιστημών και Γραμμάτων σε ανάμνηση του διάσημου Νορβηγού μαθηματικού Abel. Ο Pierre Deligne βραβεύτηκε για τη «δημιουργική συνεισφορά του στην αλγεβρική γεωμετρία και την μεταμορφωτική επίδρασή του στη θεωρία αριθμών, τη θεωρία αντιπροσώπευσης και σχετικά πεδία». Η αλγεβρική γεωμετρία εξετάζει γεωμετρικά αντικείμενα που αποτελούν σύνολα λύσεων αλγεβρικών εξισώσεων, για παράδειγμα η ακτίνα ενός κύκλου r μπορεί να περιγραφεί από την εξίσωση x2+y2=r2. Η αλγεβρική γεωμετρία έχει αποδειχθεί πως σχετίζεται με πολλά πεδία μαθηματικών, ιδιαίτερα με τη θεωρία αριθμών, τον κλάδο που ασχολείται με τις ιδιότητες των ακέραιων αριθμών.Η σχέση αυτή φαίνεται στην αναλογία μεταξύ της υπόθεσης Riemann, που περιγράφει τη σχέση μεταξύ πρώτων αριθμών, και τις λεγόμενες εικασίες Weil, οι οποίες προτάθηκαν πρώτη φορά από το Γάλλο Α. Weil το 1949 και αποτελούν το πιο διάσημο επίτευγμα του Deligne. Από τις τέσσερις εικασίες Weil, οι τρεις πρώτες αποδείχθηκαν τη δεκαετία του ’60, ενώ ο Deligne απέδειξε την τέταρτη και πιο δύσκολη, απευθείας ανάλογη της υπόθεσης Riemann. Η υπόθεση Riemann παραμένει το πιο διάσημο άλυτο πρόβλημα μαθηματικών. ΕΥΚΛΕΙΔΗΣ Β΄ 90 τ.2/4

------------------------------------------------------------------------------------------------------------------- Nobelκαι βραβεία Μαθηματικών --------------------------------------------------------------------------------------------------------------------

 Το βραβείο Shaw 2013 στα Μαθηματικά δόθηκε στον David L Donoho καθηγητή Στατιστικής στο Πανεπιστήμιο του Στάνφορντ των ΗΠΑ για τη βαθιά συμβολή του στην σύγχρονη μαθηματική στατιστική και ειδικότερα για την ανάπτυξη της βέλτιστης αλγορίθμων, για στατιστική εκτίμηση στην παρουσία θορύβου και αποτελεσματικές τεχνικές για την αραιή αντιπροσώπευση και αποκατάσταση σε μεγάλα σύνολα δεδομένων. Οι δραματικές εξελίξεις στην τεχνολογία κατά το τελευταίο μισό αιώνα, οδηγούν σε θεμελιώδεις νέες προκλήσεις σε θεωρητική και εφαρμοσμένη μαθηματική στατιστική. Ο David Donoho έχει διαδραματίσει σημαντικό ρόλο στην ανάπτυξη νέων μαθηματικών και στατιστικών εργαλείων για την αντιμετώπιση αυτών των προβλημάτων που κυμαίνονται από μεγάλα σύνολα δεδομένων σε υψηλές διαστάσεις [μόλυνση με το θόρυβο]. Το έργο του προσφέρει γρήγορη, αποτελεσματική και συχνά βέλτιστη με αλγόριθμους, που βασίζονται σε αυστηρή μαθηματική ανάλυση. Πολλά από αυτά αναδύονται από την ανάπτυξη του αλγορίθμου για στατιστικούς εκτιμητές υπό την παρουσία θορύβου. Αυτά είναι αξιοσημείωτα, το ότι θα ξεπεραστούν οι δυσκολίες που συνδέονται με το θόρυβο, με πολύ μικρή απώλεια της αποτελεσματικότητας ή της αξιοπιστίας. Στην πορεία, απέδειξε τη δύναμη της μαθηματικής θεωρίας των wavelets στην αντιμετώπιση τέτοιων προβλημάτων στον τομέα των στατιστικών. Η Donoho-Johnstone soft-κατωφλίου αλγόριθμος, έχει χρησιμοποιηθεί ευρέως σε στατιστικά όπως επίσης και σε εφαρμογές επεξεργασίας σήματος.  Το βραβείο Shaw στην Αστρονομία φέτος δόθηκε στους Steven A Balbus καθηγητής Αστρονομίας στο Πανεπιστήμιο της Οξφόρδης και John F Hawley πρόεδρος του τμήματος Αστρονομίας του Πανεπιστημίου Βιρτζίνια. Το βραβείο δόθηκε για την ανακάλυψη - μελέτη της magnetorotational αστάθειας, καθώς και για να αποδείξει ότι αυτή η αστάθεια οδηγεί σε αναταραχές και είναι ένας βιώσιμος μηχανισμός για γωνιακή μεταφορά ορμής στους αστροφυσικούς δίσκους προσαύξησης.

Nobel Φυσικής Το Nobel Φυσικής δόθηκε στους επιστήμονες Peter Higgs Βρετανό και Francois Englert Βέλγο, (αλλά δεν βρίσκεται στη ζωή ο άλλος πρωτεργάτης Robert Brut). Μετά την πειραματική του ανακάλυψη το 2012 στο CERN του μποζονίου Higgs το οποίο αποτελεί πλέον το 17ο στοιχειώδες σωματίδιο κατά το Καθιερωμένο Πρότυπο στη Φυσική. Σύμφωνα με την επικρατούσα και πλέον επιβεβαιωμένη θεωρία, τα στοιχειώδη σωματίδια είναι: τα 12 φερμιόνια (που έχουν ημιακέραιη ιδιοπεριστροφή) -6 κουάρκς (που ανά 2 ή 3 συνθέτουν άλλα σωματίδια όπως τα πρωτόνια και τα νετρόνια του ατομικού πυρήνα) και 6 λεπτόνια (ηλεκτρόνια, μυόνια, ταυ, με τα αντίστοιχα νετρίνα τους)- και τα 4 μποζόνια (ακέραιο σπιν) που είναι οι φορείς των 4 βασικών δυνάμεων στη φύση (ηλεκτρομαγνητικής, ισχυρής και ασθενούς πυρηνικής ελληλεπίδρασης και βαρύτητας). Έτσι λοιπόν και επίσημα, το μποζόνιο του Higgs αναγνωρίζεται ως το σωματίδιο που αντιστοιχεί στο πεδίο του Higgs με μάζα 125 γιγαντοηλεκτρονιοβόλτ (Gev), δηλ. περίπου 133 φορές μεγαλύτερη από του πρωτονίου. Είναι αυτό το οποίο προσδίδει μάζα στην ύλη, δηλαδή είναι στοιχειώδες σωματίδιο χωρίς συστατικά σωματίδια.

Nobel Ιατρικής To Νόμπελ Ιατρικής ή Φυσιολογίας απονεμήθηκε στους Αμερικανούς James Rothman, Randy Schekman και στον Γερμανό Thomas Südhof για την έρευνα που διεξήγαγαν σχετικά με το πώς τα κύτταρα οργανώνουν το σύστημα μεταφοράς τους. Ανακάλυψαν στην ενδοκυτταρική κυκλοφορία ότι τα κύτταρα είναι ουσιαστικά εργοστάσια παραγωγής μορίων, όπως για παράδειγμα η ινσουλίνη που απελευθερώνεται στο αίμα ή οι νευροδιαβιβαστές που μεσολαβούν στην επικοινωΕΥΚΛΕΙΔΗΣ Β΄ 90 τ.2/5

------------------------------------------------------------------------------------------------------------------- Nobelκαι βραβεία Μαθηματικών --------------------------------------------------------------------------------------------------------------------

νία των νευρικών κυττάρων. Τα μόρια αυτά πακετάρονται μέσα στα κύτταρα σε σφαιρίδια από μεμβράνες τα οποία ονομάζονται κυστίδια. Οι τρεις ερευνητές ανακάλυψαν τον μηχανισμό που ρυθμίζει την κυκλοφορία των κυστιδίων, ένα σημαντικό σύστημα μεταφοράς στα κύτταρά μας και εξήγησαν πώς τα κυστίδια φτάνουν τη σωστή στιγμή στο σωστό στόχο.

Nobel Ειρήνης Το Νομπέλ Ειρήνης απονεμήθηκε στον Οργανισμό για την Απαγόρευση των Χημικών Όπλων για τις εντεινόμενες προσπάθειές του στην εξάλειψη των χημικών όπλων. Τα πρόσφατα γεγονότα στη Συρία, όπου χρησιμοποιήθηκαν χημικά όπλα, δηλώνουν την ανάγκη να ενισχυθούν οι προσπάθειες για την εξάλειψή τους. Ο [ΟPCW] εξάλλου, που ιδρύθηκε το 1997 και εδρεύει στη Χάγη, έχει αναλάβει το κρίσιμο έργο της επίβλεψης της διάλυσης του χημικού οπλοστασίου στη Συρία, ύστερα από απόφαση του ΟΗΕ. Στο βραβείο αυτό συμμετέχει και ο Έλληνας γιατρός και ποιητής Δημήτρης Κρανιώτης που είναι ένα από τα μέλη του Global Harmony Association που συνέγραψαν το βιβλίο για την παγκόσμια ειρήνη "The ABC of Harmony".

Nobel Λογοτεχνίας Το Νομπέλ Λογοτεχνίας απονεμήθηκε αι στην 82χρονη καναδή πεζογράφο Alice Munro. Το βραβείο της απονέμεται για τη δεξιοτεχνία της στη μικρή φόρμα του σύγχρονου διηγήματος. Μιλά για οικουμενικά αισθήματα καθημερινών ανθρώπων και η οποία αποτυπώνει θαυμάσια την ανθρώπινη ύπαρξη. Η Alice Munro δεν έχει γράψει μυθιστόρημα. Στα διηγήματά της διερευνά τις σχέσεις μεταξύ ανδρών και γυναικών, τις απλές όψεις της καθημερινότητας στις μικρές πόλεις και τα παιχνίδια της μνήμης. Το 2012 εξέδωσε την 14η συλλογή διηγημάτων της υπό τον τίτλο «Dear Life» και δήλωσε επισήμως ότι θα είναι και η τελευταία. Στα βιβλία της έχει αρκετές αναφορές στα Μαθηματικά και στους τρόπους αντιμετώπισης καθημερινών προβλημάτων. Σε σχετικό βιβλίο της, αναφέρεται σε γυναίκα μαθηματικό σε σουηδικό Πανεπιστήμιο και εξιστορεί με αρκετά διεισδυτική ματιά, χαρακτηριστικά στιγμιότυπα της ακαδημαϊκής ζωής

Nobel Οικονομικών Το Νόμπελ Οικονομικών κέρδισαν οι οικονομολόγοι E. Fama, L. Hansen και ο R. Shiller για την «εμπειρική ανάλυση των τιμών των περιουσιακών στοιχείων». Οι βραβευθέντες έθεσαν τις βάσεις για την κατανόηση της λειτουργίας των τιμών των περιουσιακών στοιχείων, όπως είναι οι μετοχές και τα ομόλογα. Η διαμόρφωση των τιμών εξαρτάται εν μέρει από τις διακυμάνσεις του ρίσκου και του επενδυτικού συναισθήματος απέναντι στο ρίσκο, καθώς και εν μέρει σε συμπεριφοριακές προκαταλήψεις και στρεβλώσεις στην αγορά. Ο E. Fama είναι καθηγητής Οικονομικών στο Πανεπιστήμιο του Σικάγο, ο L. Hansen καθηγητής Οικονομικών και Στατιστικής στο ίδιο πανεπιστήμιο, ενώ ο R. Shiller είναι καθηγητής Οικονομικών στο Γιέιλ.

Nobel Χημείας Το Νόμπελ Χημείας απονεμήθηκε από κοινού στον Αμερικανό καθηγητή χημείας M. Karplus, 83 ετών, στο Νοτιοαφρικανό 66χρονο καθηγητή χημείας του πανεπιστημίου Στάνφορντ, M. Levitt, καθώς και στον 73χρονο Ισραηλινό καθηγητή χημείας και βιοχημείας A. Wsrshell του Πανεπιστημίου της Νότιας Καλιφόρνιας για την ανάπτυξη μοντέλων πολλαπλών κλιμάκων για την κατανόηση περίπλοκων χημικών συστημάτων. ΕΥΚΛΕΙΔΗΣ Β΄ 90 τ.2/6

 

H ΒΙΟΛΟΓΙΑ ΚΑΙ OI ΠΡΩΤΟΙ ΑΡΙΘΜΟΙ Παναγιώτης Χριστόπουλος Ως γνωστόν, οι αριθμοί μετέχουν με διάφορους τρόπους στα φαινόμενα της ζωής και όχι μόνο. Τα Βιομαθηματικά και η Βιοπληροφορική στις μέρες μας γνωρίζουν μεγάλη ανάπτυξη και θα έχουν μεγαλύτερη στο μέλλον. Κάποιοι αριθμοί ειδικότερα έχουν πολύ μεγάλο ενδιαφέρον και προσδίδουν μια ξεχωριστή γοητεία τόσο για την άβια όσο και για την ζωντανή ύλη. Παράδειγμα, οι χαρακτηριστικοί αριθμοί π και φ που μετέχουν σε βιολογικά θέματα με διάφορους τρόπους. Συγκεκριμένα το π=3,14159... εμφανίζεται στους μαθηματικούς τύπους της διαφοράς δυναμικού στις μεμβράνες των κυττάρων, στη ροή υγρών μέσα σε αγγεία κ.λ.π. Το φ = 1,618... γνωστό και ως χρυσός λόγος ή χρυσή αναλογία, εμφανίζεται σε διάφορες αναλογίες του ανθρώπινου σώματος, στον τρόπο διάταξης των φύλλων στα φυτά κ.λ.π. Η ακολουθία Fibonacci που συνδέεται με το φ προέκυψε σαν βιολογικό πρόβλημα απογόνων ενός ζεύγους κουνελιών. Κάτι αξιοσημείωτο που ίσως διαφεύγει σε πολλούς και σχετίζεται με το φ είναι το εξής: Οι όγκοι του πυρήνα, του κυτταροπλάσματος και του κυττάρου αποτελούν διαδοχικούς όρους γεωμετρικής προόδου και ισχύει

Ένα παράδειγμα από το χώρο ταυ ζωικού βασιλείου: δύο είδη τζιτζικιών Magicicada tredecim και Maicicada septendecim ζουν στο ίδιο περιβάλλον με κύκλο ζωής 13 και 17 χρόνια αντίστοιχα. Σε όλη τους την ζωή, εκτός από τον τελευταίο χρόνο, ζουν στο έδαφος και τρέφονται από τους χυμούς των ριζών των δέντρων. Στο τελευταίο χρόνο της ζωής τους και για λίγες εβδομάδες μεταμορφώνονται από νύμφες σε ενήλικα, καταλαμβάνουν το δάσος, τρώνε, γεννούν αυγά και πεθαίνουν. Πως όμως τα είδη αυτά εξελίχτηκαν ώστε ο κύκλος της ζωής τους να είναι πρώτοι αριθμοί; Μια απάντηση είναι ότι η από κοινού ανάδυση τους στο δάσος αργεί πολύ και αυτό συμβαίνει μια φορά στα 13 x 17 = 221 χρόνια. Ενώ αν οι κύκλοι ζωής τους ήταν σύνθετοι αριθμοί, π.χ. 12 και 18 χρόνια στον πιο πάνω χρόνο θα ανταγωνίζονταν έξι φορές, όσα είναι τα κοινά πολλαπλάσια του 12 και 18.

Vκπ =φ Vπ

κυτταροπλασματοπυρηνική σταθερά. Ας δούμε τώρα πιο διεξοδικά τους πρώτους αριθμούς σε βιολογικά θέματα. Οι πρώτοι αριθμοί είναι για τα μαθηματικά ότι για την Χημεία και τη Φυσική τα άτομα της ύλης: οι δομικοί λίθοι πάνω στους οποίους χτίζονται όλοι οι υπόλοιποι αριθμοί, τα "άτομα" που απαρτίζουν το Μαθηματικό σύμπαν. Κάποιος είπε ότι οι μαθηματικοί αγαπούν τους πρώτους αριθμούς όπως οι χημικοί αγαπούν τα άτομα και οι βιολόγοι τα γονίδια. Ο βιολόγος Μαρίνος Σπηλιόπουλος μας έστειλε μια εργασία για να μας δείξει ότι τα μαθηματικά υπάρχουν παντού μέσα στη βιολογία και προσδίδουν ξεχωριστή γοητεία στη ζωντανή αλλά και την άβια ύλη.

Η ΒΙΟΛΟΓΙΑ ΚΑΙ ΟΙ ΠΡΩΤΟΙ ΑΡΙΘΜΟΙ Μαρίνος Σπηλιόπουλος Ας εξετάζουμε σχέσεις πρώτων αριθμών με βιολογικά

θέματα, ξεκινώντας με κάποιες απλές διαπιστώσεις: α) Τα κωδικόνια του γενετικού κώδικα που κωδικοποιούν αμινοξέα είναι 61, ενώ υπάρχουν και 3 (λήξης) που δεν κωδικοποιούν κανένα αμινοξύ. Κάθε κωδικόνιο φυσικά αποτελείται από 3 νουκλεοτίδια. β) Από τα 37 γονίδια του μιτοχονδριακού DNA τα 13 κωδικοποιούν πρωτείνες. γ) Η μικρότερη μορφή ζωής, ένα βακτήριο που ανήκει στα μυκοπλάσματα έχει 521 γονίδια (πρώτος αριθμός). δ) Στα νουκλεϊκά οξέα (DNA, RNA), ο προσανατολισμός της πολυνουκλεοτιδικής αλυσίδας είναι 5' -> 3'. ε) Οι μικροσωληνίσκοι, δομές του κυτταρικού σκελετού, των βλεφαρίδων και των μαστιγίων διαφόρων ευκαρυωτικών κυττάρων, αποτελούνται από 13 παράλληλα πρωτοϊνίδια (με διάμετρο 5nm το καθένα) από εναλλασσόμενα μόρια α και β τουμπουλίνης. στ) 23 χρωμοσώματα στο ανθρώπινο αναπαρωγικό κύτταρο κ.λ.π.

Οι πρώτοι αριθμοί σχετίζονται και με την μοριακή βάση της απόπτωσης, ή προγραμματισμένου κυτταρικού θανάτου, που μελετήθηκε αρχικά στον νηματώδη σκώληκα Caenorhabditis elegans. Αργότερα, μελετήθηκε και σε άλλα ασπόνδυλα και σπονδυλωτά. Μόνο που στον ερμαφρόδιτο νηματώδη σκώληκα έχουμε το καλύτερο σύστημα μελέτης της απόπτωσης μέχρι του σημείου να θεωρηθεί ότι δημιουργήθηκε για αυτό το σκοπό. Η απόπτωση είναι η βασική βιολογική διεργασία που είναι απαραίτητη για την απομάκρυνση των πλεοναζόντων κυττάρων κατά την εμβρυϊκή ανάπτυξη, για- την διατήρηση της ομοιόστασης στον ώριμο οργανισμό; την ωρίμανση των κυτταρικών πληθυσμών του αιμοποιητικού και ανοσοποιητικού συστήματος και την καταβολή της κακοήθους εξαλλαγής. Συνήθως η απόπτωση δεν είναι επακόλουθο βλάβης και συνεπώς διαφέρει από την νέκρωση, η -οποία είναι πάντοτε αποτέλεσμα .βλαπτικής επίδρασης. Στον νηματώδη ^σκώλικα, κατά την ανάπτυξη του παράγονται 1090 κύτταρα (άρα 1091 μαζί με το αρχικό ζυγωτό), από τα οποία 131 σωματικά κύτταρα εξαλείφονται με τη διαδικασία του προγραμματισμένου κυτταρικού θανάτου ενώ 3 γονίδια είναι υπεύθυνα γι' αυτόν. Το 1091, 131 και 3 είναι πρώτοι αριθμοί, και τα

ΕΥΚΛΕΙΔΗΣ Β΄ 90 τ.2/7

------------------------------------------------------------------------------------------------------------------------------------------------------------------------------- Βιολογία και πρώτοι αριθμοί -----------------------------------------------------------------------------------------------------------------------------------------------------------------------

παραπάνω αποτελούν μια διαπίστωση, της οποίας τα βαθύτερα αίτια μας διαφεύγουν προς το παρόν. Αμινοξέα, πρωτεΐνες και πρώτοι αριθμοί Η Ελληνική γλώσσα είναι μια γλώσσα ζωντανή, δυναμική, μυστήρια. Ίσως είναι η μοναδική γλώσσα που συναντάται σε τέτοιο μεγάλο βαθμό πλήρης ταύτιση σημαίνοντος και σημαινόμενου. Οι πρωτεΐνες έχουν τα αμινοξέα ως δομικούς λίθους. Η λέξη πρωτεΐνη προέρχεται από την ελληνική λέξη "πρώτος", δηλαδή η σημασία των πρωτεϊνών για την δομή και λειτουργία των κυττάρων και των οργανισμών είναι πρωταρχική. Ας δούμε λοιπόν πως σχετίζονται με τους πρώτους αριθμούς. Με βάση το χημικό τους τύπο 5 αμινοξέα έχουν μοριακό βάρος που είναι πρώτος αριθμός. Συγκεκριμένα: Μεθειονίνη 149, Αλανίνη 89, Λευκίνη 131, Ισολευκίνη 131 και Τυροσίνη 181. Το μοριακό βάρος των άλλων αμινοξέων υπολογίζεται εύκολα και δεν υπάρχει λόγος να παρατεθούν σε πίνακα. Γιατί το ενδιαφέρον δεν είναι τα μοριακά βάρη μεμονωμένων αμινοξέων αλλά το ότι μπορούμε "τεχνητά" και εύκολα να κατασκευάσουμε πεπτίδια και πρωτεΐνες που να έχουν συνολικό μοριακό βάρος πρώτο αριθμό είτε σαν απλό άθροισμα των μοριακών βαρών των ελεύθερων αμινοξέων τους είτε όταν αφαιρούμε και το σύνολο των μορίων νερού που απομακρύνονται, πολλαπλασιασμένα επί 18 (μοριακό βάρος νερού), ή και με άλλα κριτήρια. Για παράδειγμα, έχουμε το επταπεπτίδιο: Μεθειονίνη Γλυκίνη - Τρυπτοφάνη - Ασπαρτικό οξύ - Θρεονίνη Γλουταμίνη -Κυστεΐνη. Το άθροισμα των μοριακών βαρών των αμινοξέων σε ελεύθερη κατάσταση είναι: 149 + 75 + 204 + 133 + 119 + 146 + 121 = 947 (πρώτος αριθμός). Αν αφαιρέσουμε 6x18 = 108 (το νερό που απομακρύνεται με τη συμπύκνωση), έχουμε 947 - 108 = 839 (πρώτος αριθμός). Αν πολλαπλασιάσουμε τα μοριακά βάρη των αμινοξέων με την αντίστοιχη θέση τους στην αλυσίδα, δηλαδή 1x149 4- 2x75 Η- 3x204 + 4x133 + 5x119 + 6x146 + 7x121 = 3761 πρώτος αριθμός. DNA και πρώτοι αριθμοί To DNA λειτουργεί εκτός των άλλων και σαν ζωντανός υπολογιστής. Ας δούμε κάποιες ειδικές πτυχές του στον τομέα αυτό και συγκεκριμένα στο θέμα που έχει σχέση με τους πρώτους αριθμούς. Μην ξεχνάμε ότι και στους ηλεκτρονικούς υπολογιστές που κατασκευάζει ο άνθρωπος οι πρώτοι αριθμοί παίζουν σημαντικό ρόλο σε συστήματα ελέγχου, κρυπτογραφίας και εξέλιξης των υπολογιστών. Τα νουκλεοτίδια της Αδενίνης (Α), της Θυμίνης (Τ), της Γουανίνης (G) και της Κυτοσίνης (C), που είναι οι δομικοί λίθοι του DNA έχουν αντίστοιχα μοριακά βάρη 331, 322, 347, καί 307. Οι αριθμοί 331, 347, 307 είναι πρώτοι ενώ ο 322 είναι σύνθετος. Με τα σύμβολα Α, Τ, G, C θα συμβολίζουμε ταυτόχρονα τα αντίστοιχα νουκλεοτίδια, τις αντίστοιχες βάσεις των νουκλεοτιδίων και τα μοριακά τους βάρη. Με βάση τα ανωτέρω διαπιστώνουμε ότι A+Τ=331+322=653 (1) που είναι πρώτος αριθμός. Επίσης Α+Τ+G+C=331+322+347+307=1037(2) Που είναι και αυτός πρώτος αριθμός. Επίσης εύκολα διαπιστώνουμε ότι ( G + T ) - ( A + C ) = 3 1 ( 3 ) A T - G C = 53 (4) A + T + C - G =613 (5) και AT - GC + A + Τ + C = 1013 (6) που είναι όλοι τους πρώτοι αριθμοί. Τεχνητά και με τη βοήθεια Η/Ύ, μπορεί κάποιος να φτιάξει αλυσίδες DNA που να έχουν άθροισμα μοριακών

βαρών νουκλεοτιδίων πρώτο αριθμό. Παράδειγμα: από το τμήμα μιας αλυσίδας DNA που περιέχει διάσπαρτα μεν αλλά συνολικά 29Α, 59Τ, 131G, 107C, προκύπτει μοριακό βάρος 106903 που είναι πρώτος αριθμός. Οι αριθμοί 29, 59, 131, 107 έχουν επιλεγεί και αυτοί να είναι πρώτοι. Στο συγκεκριμένο παράδειγμα και το συμπληρωματικό τμήμα της αλυσίδας με σύνολο νουκλεοτιδίων 29Τ, 59Α, 131C, 107G έχει μοριακό βάρος 106213, που είναι και αυτός πρώτος αριθμός. Έχει ιδιαίτερο ενδιαφέρον το εξής: Περιληπτικά και χωρίς να κάνουμε πολλές πράξεις αν αφαιρέσουμε τη σχέση (5) από τη σχέση (6) που είδαμε πιο πάνω προκύπτει AT-GC + G = 400  AT- GC+G =10(G - C) διότι 347-307 = 40. Τελικά προκύπτει η σχέση G=

Α.Τ  10.C (7). 9C

Μια σχέση απλή και πολύ σημαντική που συνδέει το μοριακό βάρος του νουκλεοτιδίου της γουανίνης με τα μοριακά βάρη των υπολοίπων νουκλεοτιδίων στο DNA. Όμως το θέμα δεν τελειώνει εδώ γιατί η σχέση (7) μπορεί συχνά να αποτελέσει "μηχανή" παραγωγής πρώτων αριθμών, ως εξής: θέτοντας για το C και το G τιμές πρώτων βρίσκουμε μια τιμή για το AT που μπορεί να αναλυθεί σε γινόμενο πρώτων παραγόντων. Κρατούμε για το Α μια τιμή πρώτου και για το Τ το γινόμενο των υπολοίπων (σύνθετος). Συχνά διαπιστώνουμε ότι το άθροισμα Α + Τ + G + C είναι πρώτος αριθμός, όπως συμβαίνει στο DNA. Παράδειγμα με C = 47 G=173 βρίσκουμε Α=419 και Τα=22. Το άθροισμα όλων αυτών είναι 661 και είναι πρώτος. Συχνά επίσης διαπιστώνεται ότι ο αριθμός που προκύπτει από τη σχέση (4), δηλαδή AT - GC είναι πρώτος.

Η πιο πάνω διαδικασία έχει δυο προοπτικές. Πρώτον θυμίζει μια περίπτωση στην οποία έχουμε "υποθετικά, μόρια DNA" που τα μοριακά βάρη των νουκλεοτίδίων τους ορίζονται κάθε φορά με διάφορους αριθμούς, οι τρεις ( G', C΄, Α') να είναι πρώτοι και ο ένας (Τ') σύνθετος, όπου τα αθροίσματά τους να δημιουργούν ένα καινούργιο πρώτο κ.λ.π. Αυτό μπορεί να είναι ένα όπλο στην αναζήτηση, αγνώστων πρώτων αριθμών για ερευνητές μαθηματικούς. Επίσης και για αυτούς που χρησιμοποιούν τους πρώτους για πρακτικούς λόγους σε συστήματα ελέγχου, κρυπτογραφία κ.λ.π. Η δεύτερη προοπτική είναι οι αριθμοί που προκύπτουν από τη σχέση (7) να εκφράζουν αριθμούς νουκλεοτιδίων κατά μήκος μιας αλυσίδας ή τμήματος DNA πάντα σε σχέση με το γενετικό κώδικα και φυσικά με συγκεκριμένη πληροφορία ως προς την λειτουργικότητα ή το ρόλο των προϊόντων που παράγονται.

ΕΥΚΛΕΙΔΗΣ Β΄ 90 τ.2/8

Μαθηματικοί ∆ιαγωνισμοί Μαθηματικές Ολυμπιάδες Επιτροπή Διαγωνισμών της E.M.E.

74ος ΠΑΝΕΛΛΗΝΙΟΣ ΜΑΘΗΤΙΚΟΣ ΔΙΑΓΩΝΙΣΜΟΣ ΣΤΑ ΜΑΘΗΜΑΤΙΚΑ  “Ο ΘΑΛΗΣ”

19 Οκτωβρίου 2013

Α΄ ΛΥΚΕΙΟΥ Πρόβλημα 1

1 1 1 x  y  4   Αν τα συστήματα (1 )   και 3 4 1      x y 2  βρείτε την τιμή των παραμέτρων α και β.

 2 

 x   y  4    έχουν την ίδια λύση  2x  3 y  8 

 x, y  ,

να

Λύση 1 1   και   , το σύστημα  1  γίνεται: x y 1   1  1 1                       4   4     4 2      , 3  4  1  3  1     4  1    1  3     1    4 2     2  2 4  4   1 1 οπότε το σύστημα  1  έχει τη λύση:  x, y    ,    2, 4  .    Όμως από την υπόθεση την ίδια λύση έχει και το σύστημα   2  , οπότε θα έχουμε; Αν θέσουμε

 2  4   4    2   2    2  2    10      . 4  12   8    3  2      4    4 Πρόβλημα 2 Για τους θετικούς πραγματικούς αριθμούς x, y και z ισχύει ότι: z  2  x  y  και z  3  x  y  . (α) Να αποδείξετε ότι: y  x  z . (β) Να βρείτε την τριάδα  x, y, z  για την οποία: x2  y2  z2  680 .

Λύση (α) Επειδή z  3 x  y   0  x  y  0 , έπεται ότι x  y . Επίσης από τις δεδομένες ισότητες έχουμε: z  2  x  y   3 x  y   2x  2 y  3x  3y  x  5 y , οπότε προκύπτει: z  2 x  2 y  12 y , οπότε z  x 12y 5y  7y  0 , οπότε z  x . Άρα έχουμε: z  x  y  y  x  z . (β) Από τις προηγούμενες σχέσεις, δεδομένου ότι είναι y  0, έχουμε: x 2  y 2  z 2  680  25 y 2  y 2  144 y 2  680  170 y 2  680  y 2  4  y  2 . Άρα είναι:  x, y, z   10, 2, 24  . Πρόβλημα 3 Να βρεθούν οι ακέραιοι x για τους οποίους οι αριθμοί A  8x  1 και B  2x  3 είναι και οι δύο τέλεια τετράγωνα ακεραίων. Λύση

Έστω A  8 x  1   2 και B  2 x  3   2 . Τότε λαμβάνουμε ότι: ΕΥΚΛΕΙΔΗΣ Β΄ 90 τ.2/9

-------------------------------------------------------------------------------- Μαθηματικοί Διαγωνισμοί – Μαθηματικές Ολυμπιάδες -----------------------------------------------------------------------------

x

 2 1



2 3

(1)

 2  4 2  13 .

και

(2) 8 2 Από τη σχέση (2) έχουμε:  2  4  2  13    2    2    13    2   13   2   13    2  1     2   1   ή  ή  ή      2   1     2   1    2   13   2   13   ,     7,3 ή  ,     7, 3 ή  ,     7, 3 ή  ,     7,3 . Από όλα τα παραπάνω ζεύγη, από τις σχέσεις (1), προκύπτει ότι: x  6 . Πρόβλημα 4 ˆ  200 . Θεωρούμε σημείο Δ πάνω στην πλευρά ΑΓ Δίνεται ισοσκελές τρίγωνο ΑΒΓ με    και  τέτοιο ώστε    . Από το σημείο Α φέρουμε ευθύγραμμο τμήμα ΑΕ τέτοιο ώστε    ,    και με τα σημεία Ε και Δ να βρίσκονται στο ίδιο ημιεπίπεδο ως προς την ευθεία ˆ . ΑΒ. Στη συνέχεια κατασκευάζουμε το παραλληλόγραμμο ΒΑΕΖ. Να βρείτε το μέτρο της γωνίας  Λύση

Επειδή

είναι

ˆ  20 

  

και

έχουμε

ότι:

ˆ  ˆ  ˆ  180  20  80 . Άρα τα τρίγωνα ΑΒΓ και  2 ΕΑΔ είναι ίσα, αφού έχουν δύο πλευρές τους ίσες μία προς μία    ,     και τις περιεχόμενες γωνίες ίσες 



ˆ  80  . ˆ     

Επομένως, έχουμε:      ,

ˆ  20 . Επειδή το παραλληλόγραμμο ΒΑΕΖ έχει δύο  διαδοχικές πλευρές ίσες     , είναι ρόμβος, οπότε

Σχήμα 1

     , δηλαδή το τρίγωνο ΕΔΖ είναι ισοσκελές.  ˆ ˆ ˆ   ˆ   ˆ  80  20  60 , οπότε το τρίγωΕπιπλέον, ισχύει:     80 .Επομένως  ˆ   ˆ   ˆ  100  60  40 , οπότε από το ισονο ΕΔΖ είναι ισόπλευρο. Τότε είναι:    ˆ  180  40  70 . σκελές τρίγωνο         προκύπτει ότι:  2

Β΄ ΛΥΚΕΙΟΥ Πρόβλημα 1 Για κάθε θετικό πραγματικό αριθμό x να αποδείξετε ότι:

Για ποιες τιμές του x ισχύει η ισότητα; Λύση

9x 2  3x  1 27x  2  6. x 9x  3x  1

Επειδή είναι x  0 θα είναι και 9 x 2  3 x  1  0 , οπότε αρκεί να αποδείξουμε ότι:

9x 3x 1  27x  6x9x 3x 1 9x 3x 1 6x9x 3x 1  27x  0  9x  3x 1 9x 3x 1  27x  0  9x 1 9x  27x  0  9x 1 36x  0  9x 1 2

2

2

2

2

2

2

2

2

2

2

2

2

2

που ισχύει η ισότητα ισχύει όταν 9 x 2  1  0  x 2 

2

2

2

2

2

2

 0,

1 1  x  , αφού x  0. 9 3

Πρόβλημα 2 Να υπολογιστούν οι ακέραιοι συντελεστές , ,  της εξίσωσης x 2   x    0 με   0 , αν αυτή έχει ρίζες x1  1 και x 2  . Λύση ΕΥΚΛΕΙΔΗΣ Β΄ 90 τ.2/10

-------------------------------------------------------------------------------- Μαθηματικοί Διαγωνισμοί – Μαθηματικές Ολυμπιάδες -----------------------------------------------------------------------------

Αφού οι αριθμοί 1 και  είναι ρίζες της εξίσωσης, έχουμε:      0, (1)  2   2    0. (2) Με αφαίρεση κατά μέλη των (1) και (2) λαμβάνουμε    2  1      1  0     1       0    1 ή       0.

Αν υποθέσουμε ότι είναι   1, τότε     1 και     0 , αδύνατο. Άρα είναι   1 , οπότε θα είναι:

      0    

  1

 1 

1 .  1

1      2, 0 . Επομένως, έχουμε τις περιπτώσεις:  1   0 , οπότε έχουμε:     0 και   0      0, το οποίο απορρίπτεται αφού από την υπόθεση έχουμε   0 .   2 , οπότε έχουμε     2 και 4    4    2,   4 . Επομένως προκύπτει η τριάδα συντελεστών  ,  ,     2, 2, 4  .

Επειδή    πρέπει:  

Πρόβλημα 3 Να βρείτε όλες τις τιμές του πραγματικού αριθμού x για τις οποίες αριθμητική τιμή του κλάσματος 2x 2  x  4 είναι θετικός ακέραιος. x2  x  2 Λύση

Θέλουμε να βρούμε για ποιους θετικούς ακεραίους  έχει λύση ως προς x η εξίσωση 2x2  x  4     2    x2  1   x  2 2     0 . Αν   2 προκύπτει από την εξίσωση η λύση 2 x x2 8 x  . Αν   2 , τότε η εξίσωση είναι δευτέρου βαθμού και έχει λύση ως προς x , αν, και μόνον 3 αν, η διακρίνουσά της είναι μη αρνητική. Έχουμε 2 2 2 2 2     1  8 4     7  2  33     2    33  6  . Παρατηρούμε ότι για   3 και οι δύο παρενθέσεις είναι αρνητικές, οπότε   0 . Επομένως, αφού ο  είναι θετικός ακέραιος, διάφορος του 2, έπεται ότι:   1 . Τότε η εξίσωση γίνεται x 2  2 x  6  0  x  1  7 . Άρα 8 για x  το κλάσμα παίρνει την ακέραια τιμή 2 και για x  1  7 παίρνει την ακέραια τιμή 1. 3 Πρόβλημα 4 Δίνεται οξυγώνιο τρίγωνο AB (με      ) εγγεγραμμένο σε κύκλο C(O, R) (με κέντρο O και ακτίνα R ). Ο κύκλος CB (B, AB) (με κέντρο B και ακτίνα AB ), τέμνει την A στο σημείο  και τον κύκλο C(O, R) στο σημείο  . Ο κύκλος C (  , A ) (με κέντρο  και ακτίνα A ), τέμνει την A στο σημείο  και τον κύκλο C(O, R) στο σημείο  . Να αποδείξετε ότι το τετράπλευρο που ορίζουν τα σημεία  ,  ,  ,  είναι ισοσκελές τραπέζιο. Λύση

Έστω  το δεύτερο κοινό σημείο των κύκλων C  και C . Θα αποδείξουμε ότι τα σημεία  ,  , είναι συνευθειακά. Οι χορδές  και  του κύκλου C είναι ίσες μεταξύ τους, διότι είναι ακτίνες του κύκλου C  , οπότε οι εγγεγραμμένες (στο κύκλο C ) γωνίες που βαίνουν στα ˆ = ΒΓΛ ˆ = Γˆ (1) . Η  είναι διάαντίστοιχα τόξα, θα είναι ίσες μεταξύ τους, δηλαδή ΒΓΑ

κεντρος των κύκλων C και C  , οπότε θα είναι μεσοκάθετη της κοινής χορδής  και θα διˆ = ΒΓΤ ˆ = Γˆ (2) . Άρα τα σημεία  ,  , είναι συνευχοτομεί τη γωνία ˆ , δηλαδή ΒΓΑ θειακά. Με όμοιο τρόπο αποδεικνύουμε ότι και τα σημεία  , , είναι συνευθειακά. ΕΥΚΛΕΙΔΗΣ Β΄ 90 τ.2/11

-------------------------------------------------------------------------------- Μαθηματικοί Διαγωνισμοί – Μαθηματικές Ολυμπιάδες -----------------------------------------------------------------------------

Το τρίγωνο  είναι ισοσκελές (    ). Άρα  ˆ   ˆ , οπότε τα αντίστοιχα τόξα  και  (του κύκλου C ) είναι ίσα μεταξύ τους. Από την ισότητα των τόξων      και      , προκύπτει η ισότητα των τόξων  και  . Άρα το τετράπλευρο  είναι ισοσκελές τραπέζιο με  //  . Με ανάλογο τρόπο αποδεικνύουμε ότι το τετράπλευρο  είναι ισοσκελές τραπέζιο με  //  . Άρα  //  και κατά συνέπεια το  είναι τραπέζιο και η  είναι κοινή μεσοκάθετη των παράλληλων πλευρών του. Τα τρίγωνα  και  είναι ίσα. Άρα το  είναι ισοσκελές τραπέζιο.

Σχήμα 2

Γ΄ ΛΥΚΕΙΟΥ Πρόβλημα 1 Να λύσετε στους πραγματικούς αριθμούς την εξίσωση 2x 2  5x  2x x 2  5x  1 .

Λύση Περιορισμός: x 2  5 x  0  x  x  5   0  x  0 ή x  5. Η εξίσωση, για x  0 ή x  5 , είναι

ισοδύναμη με την εξίσωση



x2   x2  5x   2 x x2  5x  1  x  x2  5 x 

 1  : x 

 1  x 2

x2  5x  1  1  ή x  x2  5x  1  2 

x 2  5 x  1  x  1  x 2  5 x , x   , 0  5,   , x  1

1  x 2  2 x  1  x 2  5 x, με x  5  x   , x  5, απορρίπτεται. 3 

 2  : x 

x 2  5 x  1  x  1  x 2  5 x , x   , 0  5,   , x  1

1 1  x 2  2 x  1  x 2  5 x, με  1  x  0 ή x  5  x   ,  1  x  0 ή x  5  x   . 7 7 Πρόβλημα 2 Αν ,  ακέραιοι και ο αριθμός    2  2 είναι τέλειο τετράγωνο ακεραίου, να αποδείξετε ότι ο αριθμός    2   ισούται με το άθροισμα δύο τέλειων τετραγώνων ακεραίων αριθμών. Λύση

x2   2 . Επειδή    , πρέπει ο αριθμητής 2 x 2   2 να είναι άρτιος ακέραιος, το οποίο συμβαίνει μόνον όταν οι ακέραιοι  και x είναι ή και οι δύο άρτιοι ή και οι δύο περιττοί. Έτσι έχουμε

Έστω ότι    2  2   x 2 , όπου x   . Τότε  

x2 2 x2 2  x     x    x    x   x  x          και    , όπου οι αριθμοί 2 2 4 2 2  2   2  είναι ακέραιοι, αφού οι ακέραιοι  και x είναι ή και οι δύο άρτιοι ή και οι δύο περιττοί. 2

2

2

2

2

2

Πρόβλημα 3 Βρείτε για ποιες τιμές της πραγματικής παραμέτρου α η εξίσωση 4x4   8  4a x3  a2  8a  4 x2  a3  8 x  a2  0 έχει όλες τις ρίζες της πραγματικούς αριθμούς.









Λύση Έχουμε

4x4   8  4a x3   a2  8a  4 x2   a3  8 x  a2   4x4  8x3  a2 x2    4ax3  8ax2  a3 x   4x2  8x  a2 

 x2  4x2  8x  a2   ax  4x2  8x  a2    4x2  8x  a2    4x2  8x  a2  x2  ax 1 . ΕΥΚΛΕΙΔΗΣ Β΄ 90 τ.2/12

-------------------------------------------------------------------------------- Μαθηματικοί Διαγωνισμοί – Μαθηματικές Ολυμπιάδες -----------------------------------------------------------------------------

Επομένως, η εξίσωση έχει όλες τις ρίζες της πραγματικούς αριθμούς, αν, και μόνον αν, και τα δύο τριώνυμα x 2  ax  1 και 4 x 2  8 x  a 2 έχουν πραγματικές ρίζες  a 2  4  0 και 64  16a 2  0  a 2  4  0 και a 2  4  0  a 2  4  a  2 ή a  2. Πρόβλημα 4 Δίνεται οξυγώνιο τρίγωνο AB (με      ) εγγεγραμμένο σε κύκλο C(O, R) (με κέντρο O και ακτίνα R ) και ευθεία ( ) που περνάει από την κορυφή Α και είναι παράλληλη στη πλευρά ΒΓ. Ο κύκλος CB (B, AB) (με κέντρο Β και ακτίνα ΑΒ), τέμνει την ( ) στο σημείο Κ και τον κύκλο C(O, R) στο σημείο Λ. Ο κύκλος C ( , A ) (με κέντρο Γ και ακτίνα ΑΓ), τέμνει την ( ) στο σημείο Ν και τον κύκλο C(O, R) στο σημείο Μ. Οι κύκλοι CB (B,AB) , C (,A) τέμνονται στο σημείο Τ και η ( ) τέμνει τον C(O,R) στο σημείο  . (α) Να αποδείξετε ότι τα σημεία  ,  ,  ,  είναι συνευθειακά. (β) Να αποδείξετε ότι οι  ,  ,  περνάνε από το ίδιο σημείο. Λύση

(α) Το τρίγωνο  είναι ισοσκελές (    ως ακτίνες του κύκλου C  ). Άρα ˆ  . Από την παραλληλία (  ) //  ˆ   ˆ   ˆ  ˆ . (με τέμνουσα την ΑΓ) έχουμε:  Από τις προηγούμενες ισότητες γωνιών, προκύπτει: ˆ  ˆ ( 1 ) . Από την ισότητα των χορδών  και  του κύκλου C( O,R ) (οι χορδές  και  είναι ακτίνες του κύκλου C B ) έχουμε: ˆ  ˆ  ˆ ( 2 ). Από τις σχέσεις (1) και (2) συμπεραίνουμε ότι:

Σχήμα 3 ˆ ˆ ˆ      , δηλαδή τα σημεία  , ,  είναι συνευθειακά. Η διάκεντρος  (των κύκλων C και C ) είναι μεσοκάθετη της κοινής χορδής τους  . Άρα ˆ  ˆ  ˆ . Από B



την ισότητα των γωνιών ˆ και ˆ , προκύπτει ότι τα σημεία  , ,  είναι συνευθειακά, οπότε σε συνδυασμό με το προηγούμενο συμπέρασμα έπεται ότι τα σημεία  ,  , , είναι συνευθειακά. (β) Με ανάλογο τρόπο αποδεικνύουμε ότι και τα σημεία  , ,  , είναι συνευθειακά, οπότε τα σημεία  και  είναι μέσα των πλευρών  και  , αντίστοιχα, του τριγώνου  . Θα αποδείξουμε ότι το σημείο  είναι το μέσο της πλευράς  (οπότε οι  ,  ,  θα συντρέχουν στο βαρύκεντρο του τριγώνου  ). Πράγματι, το τετράπλευρο  είναι ισοσκελές τραπέζιο εγγεγραμμένο στον κύκλο C( O , R ) , οπότε ισχύουν οι παρακάτω ισότητες γωˆ  (από το ισοσκελές τραπέζιο  ) νιών: ˆ   ˆ   ˆ  (από το ισοσκελές τρίγωνο  ). Άρα η  είναι παράλληλη προς την  , δηλαδή το  είναι το μέσο της  . Παρατήρηση Δεν είναι απαραίτητο (για την απόδειξη του δευτέρου ερωτήματος) να αποδείξουμε ότι το σημείο  ανήκει στην ίδια ευθεία με τα σημεία  , , . Χρειάζεται όμως για να αποδείξουμε ότι και  , , συντρέχουν και να συμπεράνουμε ότι τα σημεία ο κύκλος C( O , R ) είναι ο κύκλος Euler του τριγώνου  . ΕΥΚΛΕΙΔΗΣ Β΄ 90 τ.2/13

-------------------------------------------------------------------------------- Μαθηματικοί Διαγωνισμοί – Μαθηματικές Ολυμπιάδες -----------------------------------------------------------------------------

16ος ΜΕΣΟΓΕΙΑΚΟΣ ΜΑΘΗΜΑΤΙΚΟΣ ΔΙΑΓΩΝΙΣΜΟΣ 2013  Στη μνήμη Peter O’ Halloran

28 Απριλίου 2013

Πρόβλημα 1. Υπάρχουν δύο πολυώνυμα P  x  και Q  x  βαθμού 3, με συντελεστή του

τριτοβάθμιου όρου μονάδα, τέτοια ώστε το πολυώνυμο P  Q  x   να έχει ρίζες εννέα δια-

φορετικούς μεταξύ τους μη αρνητικούς ακέραιους με άθροισμα 72; Λύση Έστω z1 , z2 , z3 οι τρεις ρίζες του πολυωνύμου P  x    x  z1  x  z2  x  z3  και έστω Q  x   x3  sx 2  tx  u . Τότε οι εννέα ρίζες του πολυωνύμου

P  Q  x     Q  x   z1   Q  x   z2   Q  x   z3 

είναι οι ρίζες ai , bi , ci των πολυωνύμων Q  x   zi , i  1, 2,3 . Από τους τύπους του Vieta λαμβάνουμε ai  bi  ci  s, i  1, 2,3 ai bi  bi ci  ci ai  s, i  1, 2,3,

Επειδή, σύμφωνα με την υπόθεση, το άθροισμα των εννέα ριζών του πολυωνύμου P  Q  x   είναι 72, έπεται ότι s  24 , δηλαδή είναι ai  bi  ci  24, i  1, 2,3 (1) Επομένως εξετάζουμε την ύπαρξη εννέα διαφορετικών ανά δύο αριθμών, από τους οποίους προκύπτουν τρεις διαφορετικές τριάδες μη αρνητικών ακέραιων τα στοιχεία των οποίων έχουν άθροισμα 24 και το ίδιο άθροισμα τετραγώνων ai2  bi2  ci2   ai  bi  ci

Λύνοντας το σύστημα



2

 2  ai bi  bi ci  ci ai   576  2t , i  1, 2,3 .

a  b  c  24, a

2

(2)

 b 2  c 2  576  2t  στους μη αρνητικούς ακέραι-

ους βρίσκουμε ότι για t  143 , προκύπτουν οι τριάδες  a1 , b1 , c1    0,11,13 ,  a2 , b2 , c2   1,8,15  ,  a3 , b3 , c3    3,5,16  , οπότε η απάντηση στο ερώτημα του προβλήματος είναι καταφατική. Πρόβλημα 2. Να προσδιορίσετε τον ελάχιστο ακέραιο k για τον οποίο μπορεί να ισχύει: Σε μία συνάντηση σκακιού με 24 παίκτες, κάθε ζευγάρι παικτών παίζουν μεταξύ τους τουλάχιστον δύο και το πολύ k παιγνίδια. Στο τέλος της συνάντησης προκύπτει ότι κάθε παίκτης έχει παίξει διαφορετικό αριθμό παιγνιδιών. Λύση Αν ήταν k  3 , τότε ο κάθε παίκτης θα έπαιζε 2 ή 3 παιγνίδια με καθένα από τους υπόλοιπος 23 παίκτες. Επομένως ο κάθε παίκτης θα έπαιζε το λιγότερο 2  23  46 παιγνίδια και το περισσότερο 3  23  69 παιγνίδια. Όμως είναι αδύνατον να υπάρχει παίκτης Α που έχει παίξει 46 παιγνίδια (δύο απέναντι σε καθέναν από τους υπόλοιπους 23 παίκτες) και ταυτόχρονα να υπάρχει παίκτης Β ο οποίος έχει παίξει 69 παιγνίδια ( 3 παιγνίδια απέναντι σε κάθε άλλο παίκτη, άρα και στον Α). Επομένως από το σύνολο 46, 47,...68, 69 υπάρχουν διαθέσιμοι μόνον 23 αριθμοί που θα είναι οι διαφορετικοί αριθμοί των παιγνιδιών των 24 παικτών, που είναι αδύνατον. Στη συνέχεια θα αποδείξουμε με επαγωγή ότι η τιμή k  4 είναι δυνατή. Θα αποδείξουμε ότι για κάθε n  3 υπάρχει συνάντηση σκακιού Tn με n παίκτες, στην οποία κάθε ζευγάρι παικτών παίζει τουλάχιστον 2 και το πολύ 4 παιγνίδια μεταξύ τους, όπου τελικά κάθε παίκτης παίζει διαφορετικό αριθμό παιγνιδιών. Για n  3 θεωρούμε τρεις παίκτες οι οποίοι παίζουν 2, 3 και 4 παιγνίδια, αντιστοίχως, μεταξύ τους, οπότε παίζουν συνολικά 5, 6 και 7 παιγνίδια, αντιστοίχως. ΕΥΚΛΕΙΔΗΣ Β΄ 90 τ.2/14

-------------------------------------------------------------------------------- Μαθηματικοί Διαγωνισμοί – Μαθηματικές Ολυμπιάδες -----------------------------------------------------------------------------

Για το βήμα της επαγωγής θεωρούμε μία συνάντηση σκακιού Tn , όπου οι n παίκτες παίζουν ο καθένας a1  a2  ...  an παιγνίδια. Τότε έχουμε τις περιπτώσεις: 

Αν στη συνάντηση Tn δεν υπάρχει παίκτης που να έχει παίξει ακριβώς δύο παιγνίδια εναντίον κάθε άλλου παίκτη, τότε a1  2n  2 . Τότε θεωρούμε ένα νέο παίκτη ο οποίος παίζει ακριβώς δύο παιγνίδια εναντίον κάθε άλλου παίκτη, οπότε οι νέοι αριθμοί παιγνιδιών θα είναι 2n  a1  2  a2  2  ...  an  4  4n .



Διαφορετικά, κανένας παίκτης στη συνάντηση Tn δεν μπορεί να έχει παίξει ακριβώς 4 παιγνίδια εναντίον κάθε άλλου παίκτη, οπότε a  4n  4 . Τότε θεωρούμε ένα νέο παίκτη ο οποίος παίζει ακριβώς τέσσερα παιγνίδια εναντίον κάθε άλλου παίκτη, οπότε οι νέοι αριθμοί a1  4  a2  4  ...  an  4  4n . παιγνιδιών θα είναι Πρόβλημα 3. Αν x, y, z  0 με x 2 y 2  y 2 z 2  z 2 x 2  6 x 2 y 2 z 2 , να αποδείξετε ότι:

x  x  yz

y z   3. y  zx z  xy

Λύση

Η συνάρτηση f  x  

1 , x   1,   είναι κυρτή, αφού 1 x 3 f  x  , x   1,   , 2 4 1  x  1  x

οπότε από την ανισότητα του Jensen λαμβάνουμε:  1  yz zx xy    zx  1   yz   xy    f    f    f    f      y z  3  x   z   y  3 x x y z 3 3 3      (1) x  yz y  zx z  xy  yz zx xy  1  yz zx xy  1     3    y z  y z  3 x  x Επιπλέον, από την υπόθεση x 2 y 2  y 2 z 2  z 2 x 2  6 x 2 y 2 z 2 έπεται ότι: yz zx xy   6. (2) x y z Από τις (1) και (2), έπεται ότι: x y z 3 3      3. x  yz y  zx z  xy 3 6 Η ισότητα ισχύει όταν x  y  z  2. Πρόβλημα 4. Το τετράπλευρο ABCD είναι εγγεγραμμένο σε κύκλο  . Οι ευθείες AB, DC τέμνονται στο σημείο E , ενώ οι ευθείες BC , AD τέμνονται στο σημείο F . Να αποδείξετε ότι ο κύκλος διαμέτρου EF τέμνει τον κύκλο  ορθογώνια. Λύση Έστω Ο το κέντρο και R η ακτίνα του κύκλου Γ. Έστω ακόμη G το δεύτερο κοινό σημείο των περιγεγραμμένων κύκλων των τριγώνων FDC και BCE. Τότε το σημείο G, θα ανήκει στην ευθεία που ορίζουν τα σημεία E,F (γνωστή πρόταση που θα αποδείξουμε στο τέλος (**)).

ΕΥΚΛΕΙΔΗΣ Β΄ 90 τ.2/15

-------------------------------------------------------------------------------- Μαθηματικοί Διαγωνισμοί – Μαθηματικές Ολυμπιάδες -----------------------------------------------------------------------------

Έστω L το μέσο της ΕF. Από τα σημεία F, L και E θεωρούμε τα εφαπτόμενα τμήματα FH, LM και EK, προς τον κύκλο Γ. Από τη δύναμη του σημείου F ως προς τον κύκλο Γ και τον περιγεγραμμένο κύκλο του τριγώνου BCE, έχουμε: 2 FG  FE  FC  FB  FH (1) . Από τη δύναμη του σημείου Ε ως προς τον κύκλο Γ και τον περιγεγραμμένο κύκλο του τριγώνου FDC, έχουμε: 2 EG  EF  EC  ED  EK (2) .

Σχήμα 1 Προσθέτοντας τις σχέσεις (1) και (2) , έχουμε:

FG  FE  EG  EF  FC  FB  EC  ED  FH 2  EK 2   EF ( FG  EG )  FH 2  EK 2  EF 2  FH 2  EK 2  4 FL2  FH 2  EK 2 .

Στην τελευταία ισότητα προσθέτουμε 2 R 2 και την μετασχηματίζουμε σε: 4 FL2  2 R 2  FH 2  R 2  EK 2  R 2 . Επειδή όμως FH 2  R 2  FH 2  OH 2  FO 2 (από το ορθογώνιο τρίγωνο ΟΗF) και  2  R 2   2  O 2  O 2 (από το ορθογώνιο τρίγωνο ΟΚΕ), καταλήγουμε στη σχέση: 4 FL2  2 R 2  F 2  E 2 . Επίσης, από το θεώρημα των διαμέσων στο τρίγωνο OEF έχουμε: F 2  E 2  2L2  2 FL2 , οπότε λαμβάνουμε: 4 FL2  2 R 2  2L2  2 FL2  OL2  FL2  R 2 (3) . Από το ορθογώνιο τρίγωνο OLM έχουμε: OL2  L 2   2  LM 2  R 2 (4) . Από τις σχέσεις (3) και (4) , έχουμε τελικά: L  LE  LF . Δηλαδή ο κύκλος με κέντρο το μέσο L του EF διέρχεται από το Μ και L   . Άρα οι κύκλοι τέμνονται ορθογώνια. (**) Το τετράπλευρο BCGE είναι εγγεγραμμένο, οπότε ˆ  Bˆ . Το τετράπλευρο CDFG είναι εγγεγραμμένο, G 1 1 ˆ D ˆ . Επίσης, το τετράπλευρο ABCD είναι οπότε G 2

1

ˆ  180 εγγεγραμμένο, οπότε ˆ 1  D 1 Σχήμα 2 Από τις προηγούμενες ισότητες γωνιών, καταλήγουμε:  ˆ ˆ G1  G2  180 , οπότε τα σημεία E,G,F είναι συνευθειακά. Παρατήρηση: Το σημείο G είναι το σημείο Miquel του πλήρους τετραπλεύρου ABCDEF, στο οποίο ισχύει η πρόταση: “Αν το τετράπλευρο ABCD είναι εγγράψιμο, τότε το σημείο Miquel ανήκει στην ευθεία EF που ορίζουν τα σημεία τομής των απέναντι πλευρών του τετραπλεύρου ”. ΕΥΚΛΕΙΔΗΣ Β΄ 90 τ.2/16

-------------------------------------------------------------------------------- Μαθηματικοί Διαγωνισμοί – Μαθηματικές Ολυμπιάδες -----------------------------------------------------------------------------

Ασκήσεις για Διαγωνισμούς  Επιτροπή Διαγωνισμών της E.M.E.

Λύσεις  τεύχους 89 

Α20. Βρείτε όλες τις γνησίως αύξουσες συναρτήσεις f :    που ικανοποιούν την ισότητα

f  x  f  y    f  x  y   1, για κάθε x , y   .

[Ουκρανία 2013]

Λύση Θέτοντας x   y στη δεδομένη σχέση, λαμβάνουμε: f   y  f  y    f  0   1, οπότε, λόγω του

ότι η f είναι γνησίως αύξουσα, υπάρχει μοναδική τιμή c   τέτοια ώστε  y  f  y   c , για κάθε y   . Για την εύρεση της σταθεράς c από τη δεδομένη σχέση έχουμε

f  x  f  y    f  x  y  c   ( x  y  c )  c,

f  x  y   1  ( x  y  c)  1  x  y  c  1,

οπότε x  y  2c  x  y  c  1  c  1. Άρα η ζητούμενη συνάρτηση είναι f  x   x  1. xn

Α21. Δίνεται η ακολουθία xn , n  * με x1  a  0, xn1  xn 

, για κάθε n  1. Να αποδείn2 ξετε ότι η ακολουθία xn , n * είναι γνησίως αύξουσα και φραγμένη πάνω. [Βιετνάμ 2013] Λύση Από την αναδρομική σχέση ορισμού της ακολουθίας xn , άμεσα προκύπτει με επαγωγή ότι: xn  0, δηλαδή ισχύει: xn 1  xn , n2 για κάθε n  1, 2,3,.. Επομένως, η ακολουθία xn είναι γνησίως αύξουσα. Επιπλέον, έχουμε

xn  0, για κάθε n  1, 2,3,... , οπότε θα είναι xn 1  xn  2

x 1 1 1  1   xn  2 2n  4   xn  2   xn 1  xn  2 , n  1, 2,3,... . 4 4n 2n 4n 2n  2n  Χρησιμοποιώντας όλες τις παραπάνω σχέσεις, λαμβάνουμε 1 1  1 n 1 1 1 n  1 1 1            ... xn 1  a    a a  2 2  2n  2 k  2 2k  k  1 2 2 k 2  k  1 k   2 22 xn 1  xn 1 





2 1 1 1  xn 1  a   1    a  1  xn 1  a  1 , για κάθε n  0,1, 2,... 2 2 n Επομένως, ακολουθία xn είναι φραγμένη πάνω. Α22. Να αποδείξετε ότι για όλους τους θετικούς πραγματικούς αριθμούς x, y, z ισχύει:

x 2x  y y  2z  x 



y 2 y  z

z 2x  y



z  2z  x 

x 2 y  z

 1.

[Τουρκία 2013]

Λύση Ο πρώτος όρος του πρώτου μέλους της ανισότητας με πρόσθεση μιας μονάδας γράφεται: 2  x 2  yz  x  2x  y  1  y  2z  x y  2z  x Χρησιμοποιώντας αντίστοιχες σχέσεις και για τους άλλους δύο όρους μετατρέπουμε την ανισόx 2  yz y 2  zx z 2  xy τητα στην παρακάτω ισοδύναμη της: f  x, y, z      2. y  2z  x z  2x  y  x  2 y  z  ΕΥΚΛΕΙΔΗΣ Β΄ 90 τ.2/17

-------------------------------------------------------------------------------- Μαθηματικοί Διαγωνισμοί – Μαθηματικές Ολυμπιάδες -----------------------------------------------------------------------------

Στη συνέχεια θα χρησιμοποιήσουμε την ανισότητα Cauchy – Schwarz  a2 a2 a2  2 ( x1  x2  ...  xn )  1  2  ...  n    a1  an  ...  an  xn   x1 x2 για θετικούς πραγματικούς αριθμούς x1 , x2 ,..., xn , σε δύο παραστάσεις που έχουν άθροισμα την

 x  y  z , x2 y2 z2 παράσταση f  x, y, z  . Έχουμε: g  x, y, z      y  2z  x z  2x  y  x  2 y  z  3 xy  yz  zx  2

x  y  z z x y h  x, y , z      . 2 z  x 2 x  y 2 y  z 2  x 2  y 2  z 2   xy  yz  zx 2

Επειδή ισχύει ότι f  x, y, z   g  x, y, z   h  x, y, z  λαμβάνουμε   1 1 2  f  x, y , z    x  y  z    2 2 2  3  xy  yz  zx  2  x  y  z   xy  yz  zx  

2 x  y  z

3  xy  yz  zx   2  x 2  y 2  z 2   xy  yz  zx  2 x  y  z



2 x  y  z

4



4

3  xy  yz  zx   2  x  y  z   3( xy  yz  zx)    2

4

 2, όπου στην τελευταία ανισότητα χρη2  3  xy  yz  zx   2  x  y  z 2  3( xy  yz  zx      2   σιμοποιήσαμε την ανισότητα αριθμητικού – γεωμετρικού μέσου. Ν14. Βρείτε όλους τους φυσικούς αριθμούς n για τους οποίους υπάρχουν θετικοί ακέραιοι

p,q τέτοιοι ώστε

n

2

 2    2n  1 . p

q

[Ουκρανία 2013]

Λύση Για n  0,1, 2,3, 4 εύκολα διαπιστώνουμε ότι δεν μπορεί να αληθεύει η δεδομένη εξίσωση. Για

n  5 η εξίσωση γίνεται 27 p  9q  33 p  32q  3 p  2q , η οποία αληθεύει για όλα τα ζεύγη  p, q    2k ,3k  , k  1, 2,3,... Έστω n  6. Αν υποθέσουμε ότι για κάποιο ζεύγος  p, q  αληθεύει η εξίσωση και r είναι ένας πρώτος διαιρέτης του n 2  2 , τότε r 2n  1 και αντιστρόφως, αν r είναι ένας πρώτος διαιρέτης του 2n  1, τότε r n 2  2 . Ας υποθέσουμε ότι r είναι ένας κοινός πρώτος διαιρέτης των n2  2, 2n  1 . Τότε θα ισχύει: r n2  2 και r 2n 1  r n2  2  2n 1   n  1  r n  1, 2

αφού r πρώτος. Άρα r 2  n  1   2n  1  3  r  3 , δηλαδή ο μοναδικός πρώτος διαιρέτης των αριθμών n 2  2 , 2n  1 είναι ο 3. Αυτό σημαίνει ότι οι αριθμοί n 2  2 , 2n  1 θα έχουν τη μορφή n2  2  3m , 2n  1  3s , με m  s  3, αφού έχουμε θεωρήσει n  6. Τότε από τις δύο τελευταίες ισότητες λαμβάνουμε (2n)2 8  4 3m και 2n  3s 1 3s 1 8  4 3m 32s  23s 9  4 3m 2

 3s  3s  2  4  3m  s  3  9  3s 9, με s  3. άτοπο.

 n   2n   3n   n 2   Ν15. Έστω Sn    ,   ,   , ... ,    , n   * . Να αποδείξετε ότι:  n   n   n   n   (α) υπάρχουν άπειροι σύνθετοι φυσικοί αριθμοί n τέτοιοι ώστε το σύνολο Sn δεν είναι ένα πλήρες σύστημα υπολοίπων modulo n. ΕΥΚΛΕΙΔΗΣ Β΄ 90 τ.2/18

-------------------------------------------------------------------------------- Μαθηματικοί Διαγωνισμοί – Μαθηματικές Ολυμπιάδες -----------------------------------------------------------------------------

(β) υπάρχουν άπειροι σύνθετοι φυσικοί αριθμοί n τέτοιοι ώστε το σύνολο Sn είναι ένα πλήρες σύστημα υπολοίπων modulo n. [Σερβία 2013] Λύση (α) Θυμίζουμε ότι ένα σύνολο S ακεραίων που έχει n στοιχεία λέγεται πλήρες σύστημα υπολοίπων modulo n , αν για κάθε ακέραιο a υπάρχει μοναδικό στοιχείο x  S με την ιδιότητα a  x  mod n  . Ισοδύναμα, ένα σύνολο S ακεραίων είναι πλήρες σύστημα υπολοίπων modulo

n , αν έχει n στοιχεία και αν x, y  S με x  y , τότε x  y  mod n  . Θα αποδείξουμε ότι για τους φυσικούς αριθμούς n  2 p , με p πρώτο περιττό φυσικό αριθμό, το σύνολο S n δεν είναι ένα πλήρες σύστημα υπολοίπων modulo n  2 p . p 1 p 1  2kp  2kp  i 2kp  p  i     k  2k  1 mod p  . Παρατηρούμε ότι ο 2 1 Έχουμε:  k k      pi i 1 2 p  i i 1  2p   2kp   p 1  αριθμός  , p, 2 p  , οπότε το σύνολο S 2 p έχει τρία του διαιρείται με το p , αν k    2   2p  λάχιστον στοιχεία του που διαιρούνται με το p . Αυτό σημαίνει ότι το σύνολο S 2 p έχει δύο τουλάχιστον στοιχεία που διαιρούμενα με το 2 p δίνουν το ίδιο υπόλοιπο. Πράγματι, αν  2 ki p     p i , i  1, 2,3, τότε δύο τουλάχιστον από τα πηλίκα  i , i  1, 2,3 θα είναι άρτιοι ή θα  2p   2k p  είναι περιττοί. Τότε θα έχουμε  i   2 p  0 mod2 pi , για δύο τιμές του i  1, 2,3 ή θα εί 2p   2k p  ναι  i   p  2i 1  p  mod2 p , για δύο τιμές του i  1, 2,3 .  2p  Αυτό σημαίνει ότι το σύνολο S n δεν μπορεί είναι ένα πλήρες σύστημα υπολοίπων modulo n 2p. (β) Θα αποδείξουμε ότι για τους φυσικούς αριθμούς n  p 2 , με p πρώτο περιττό φυσικό αριθμό, το σύνολο S n είναι ένα πλήρες σύστημα υπολοίπων modulo n  2 p . Έχουμε: p 1  kp 2  p 1 kp 2  i kp 2  jp kp 2  i k ,     2   2   2 p  i jp p  i p   i 0 j 1 j p   p 1 p 1 p 1  kn   kp  1 kp  j 2 2 το οποίο γίνεται modulo n  p ως εξής:    k   k  1    k  k p  . j j  j 1 j j 1 j 1  n 2

p 1

2 Όμως ισχύει ότι:  1    1  1 p j 1 j j 1  j p 1

p 1

 kp 2   2 p 2 , οπότε 0 mod p      2   k  m od p  .   j  j 1 j  p  j  p  

Σημείωση: Οι παραπάνω περιπτώσεις δεν είναι μοναδικές. Υπάρχουν και άλλες δυνατότητες για τις τιμές του n. Για παράδειγμα, το ερώτημα (α) μπορούμε να αποδείξουμε ότι επαληθεύεται για n  8k  6, k   , ενώ το ερώτημα (β) επαληθεύεται και για n  p k , k  , k  2 όπου p πρώτος.

Ν16. Έστω   n  ο αριθμός των θετικών ακέραιων μικρότερων του n που είναι σχετικά πρώτοι με τον n, όπου n θετικός ακέραιος. Βρείτε όλα τα ζεύγη θετικών ακέραιων  m , n  που ικανοποιούν την εξίσωση

2n   n    n   1 !  nm  1 .

[Τουρκία 2013]

Λύση Αν n  1 , τότε η εξίσωση γίνεται: 2  0!  2  3  2, αδύνατη. Αν n πρώτος, τότε   n  n 1, οπότε η εξίσωση γίνεται: 2n  nm  n  m  2 . Αν n  p 2 , όπου p πρώτος, τότε   n  p2  p και ΕΥΚΛΕΙΔΗΣ Β΄ 90 τ.2/19

-------------------------------------------------------------------------------- Μαθηματικοί Διαγωνισμοί – Μαθηματικές Ολυμπιάδες -----------------------------------------------------------------------------

η εξίσωση γίνεται: 2 p   p  1 !  p 2 m  1. (1) Για την εξίσωση (1) διακρίνουμε τις περιπτώσεις: 2



Αν p  2, τότε από την εξίσωση προκύπτει ότι  p  1 !  2  mod 4  , το οποίο είναι δυνατό



μόνο για p  3. Τότε όμως προκύπτει η εξίσωση 29  2  32 m  1  514  32 m  1 , η οποία δεν έχει ακέραιες λύσεις. Αν p  2, τότε προκύπτει τελικά η εξίσωση 22 m  16  m  2 . Άρα  m, n    2, 4  .

Για όλες τις άλλες περιπτώσεις για την τιμή του n , έστω p ο ελάχιστος πρώτος θετικός διαιρέτης του n . Επειδή 1  p  2 p  ...  p 2  n, έχουμε n  1    n   p, οπότε p  n    n   1 ! .

Επομένως από την εξίσωση προκύπτει ότι p 2 n  1 και ο p είναι περιττός. Όμως τότε p 2 p 1  1 και p 2 d  1 ,όπου d  gcd  n, p  1 .

Επειδή ο p είναι ο ελάχιστος πρώτος θετικός διαιρέτης του n , έπεται ότι d  1  p 1, άτοπο. Άρα οι λύσεις τις εξίσωσης είναι τα ζεύγη  m, n    2, 2  και  m, n    2, 4  .

Γ14. Έστω M , N και P τα μέσα των πλευρών BC , CA και AB , αντίστοιχα, του τριγώνου ABC . Έστω O το περίκεντρο του τριγώνου ABC . Οι περιγεγραμμένοι κύκλοι των τριγώνων BOC και MNP τέμνονται σε διαφορετικά σημεία X , Y εσωτερικά του τριγώνου ABC . Να αποδείξετε ότι BAX  CAY . [Σερβία 2013] Λύση Συμβολίζουμε με c1 και c 2 τους περιγεγραμμένους κύκλους των τριγώνων MNP και BOC , αντίστοιχα. Ο κύκλος c1 είναι ο κύκλος του Euler του τρι-

γώνου BC και περνά από τα ίχνη D και E των υψών BD και CE . Ο κύκλος c1 περνάει επίσης από το μέσο  1 του  (όπου Η το ορθόκεντρο του τριγώνου ABC ). Ισχυριζόμαστε ότι το δεύτερο σημείο τομής  της  και του κύκλου c1 , ανήκει στο κύκλο c 3 (που είναι ο κύκλος

του Euler του τριγώνου ADE ). Υποθέτουμε ότι το  βρίσκεται μεταξύ των σημείων  και  (οι άλλες περιπτώσεις αντιμετωπίζονται ανάλογα). Έστω D1 και E1 τα μέσα των

Σχήμα 1

AD AE και αντίστοιχα. Εφόσον AY  AZ  AD  AN  AD1  AC , τα σημεία Y ,Z ,C , D1 είναι ομοκυκλικά, οπότε AZˆD1  ACˆY . Όμοια αποδεικνύουμε ότι AZˆ 1  Aˆ Y , οπότε έχουμε: ˆ C  BA ˆC. D1 Zˆ 1  AZˆD1  AZˆE1  ACˆY  ABˆY  BYˆC  BA

Άρα το σημείο  ανήκει στο κύκλο c 3 . Στη συνέχεια παρατηρούμε ότι τα τρίγωνα ABC και ADE είναι όμοια με ομόλογες κορυφές A  A, B  D, C  E . Επίσης ομόλογοι κύκλοι των δύο τριγώνων είναι οι εξής: c2  c1 , αφού O1 είναι το περίκεντρο του τριγώνου ADE , και οι κύκλοι Euler c1  c3 . Επομένως ως προς την ομοιότητα των δύο τριγώνων θα έχουμε: ˆ  DAZ ˆ  CAY ˆ . X  c  c  Z  c  c . Επομένως προκύπτει ότι: BAX 1

2

2

3

Γ15. Έστω A1 , B1 τα ίχνη των υψών από τις κορυφές A, B , αντίστοιχα, ενός τριγώνου ABC με AC  BC . Ο κύκλος k περνάει από τα σημεία A1 , B1 και εφάπτεται της πλευράς AB στο σημείο D . Αν τα τρίγωνα ADA1 και BDB1 έχουν ίσα εμβαδά, να αποδείξετε ότι: A1 DB1  ACB . [Σερβία 2013] ΕΥΚΛΕΙΔΗΣ Β΄ 90 τ.2/20

-------------------------------------------------------------------------------- Μαθηματικοί Διαγωνισμοί – Μαθηματικές Ολυμπιάδες -----------------------------------------------------------------------------

Λύση Υποθέτουμε (χωρίς βλάβη της γενικότητας) ότι BC  AC . Οι ευθείες  και τέμνονται στο σημείο  (το  μεταξύ των  και  ). Από την ισότητα των εμβαδών των τριγώνων DA1 και BDB1 έχουμε:

AD PB1 . Από τη δύνα DB PA1

μη του σημείου  , ως προς το κύκλο k (σε συνδυασμό με το εγγράψιμο τετράπλευρο B1 A1 B ) έχουμε: Σχήμα 2

PD2  PA1  PB1  PA  PB . Από τις τελευταίες ισότητες, προκύπτουν οι α-

PD PA AD PB1 , από τις οποίες (στη συνέχεια), προκύπτουν οι παραλληλίες    PB PD DB PA1 ˆ B  ACˆB .  1 D // BC και A1 D // AC . Άρα 1 D 1

ναλογίες

Ασκήσεις για λύση 

Α23. Να προσδιορίσετε όλες τις συναρτήσεις f :    που είναι τέτοιες ώστε: f 1  0 και f  x   f  y    x  y  f  x  y  , για κάθε x, y  . Α24. Αν a, b, c είναι θετικοί πραγματικοί αριθμοί που ικανοποιούν την ισότητα a b b a 3  b 3  c 3  a 4  b 4  c 4 , να αποδείξετε ότι:  3  3 3 2  1. 2 3 3 2 3 a b c a b c a b c Α25. Αν a, b, c πραγματικοί αριθμοί με abc  0, να αποδείξετε ότι: a2 b2 c2   2 b2  c2 c2  a2 a 2  b2 A26. Ονομάζουμε τα πολυώνυμα A  x   an x n  ....  a1 x  a0 και B  x   bm x m  ....  b1 x  b0 , με anbm  0, όμοια, αν n  m και υπάρχει μία μετάθεση  του συνόλου 1, 2,..., n τέτοια ώστε να

ισχύει bi  a  i  , για κάθε i  1, 2,..., n . Αν P  x  και Q  x  είναι δύο όμοια πολυώνυμα με ακέραιους συντελεστές και P  2 4   32012 , βρείτε την ελάχιστη δυνατή τιμή του Q  32012  .

Γ16. Θεωρούμε οξυγώνιο τρίγωνο ABC με AB  AC . Έστω E το κέντρο του κύκλου Euler ˆ  x, ˆ  ACP του τριγώνου και P τυχόν σημείο του ευθυγράμμου τμήματος AE . Αν είναι ABP ˆ να αποδείξετε ότι: x  90  2  A Γ17. Θεωρούμε ισόπλευρο τρίγωνο ABC και σημείο της D πλευράς BC , διαφορετικό από τα άκρα της. Έστω I το κέντρο του παρεγεγραμμένου κύκλου του τριγώνου ABD απέναντι από την πλευρά AB και έστω J το κέντρο του παρεγεγραμμένου κύκλου του τριγώνου ACD απέναντι από την πλευρά AC . Έστω ότι οι περιγεγραμμένοι κύκλοι των τριγώνων AIB και AJC τέμνονται για δεύτερη φορά στο σημείο E . Να αποδείξετε ότι το σημείο A είναι το σημείο τομής των διχοτόμων του τριγώνου IEJ . Ν17. Βρείτε όλους τους θετικούς ακέραιους a, b που είναι τέτοιοι ώστε:

a

2

 b   a 2b  a  και  b 2  a   ab 2  b  .

Ν18. Να αποδείξετε ότι υπάρχουν άπειροι θετικοί ακέραιοι n τέτοιοι ώστε : n 2n  8 . ΕΥΚΛΕΙΔΗΣ Β΄ 90 τ.2/21

------------------------------------------------------------------------------------------------------------------------------------- Homo Mathematicus ----------------------------------------------------------------------------------------------------------------------------------

ΗΟΜΟ ΜΑΤHEMATICUS Η Homo Mathematicus είναι μια στήλη στο περιοδικό μας, με σκοπό την ανταλλαγή απόψεων και την ανάπτυξη προβληματισμού πάνω στα εξής θέματα: 1) Τι είναι τα Μαθηματικά, 2) Πρέπει ή όχι να διδάσκονται, 3) Ποιοι είναι οι κλάδοι των Μαθηματικών και ποιο το αντικείμενο του καθενός, 4) Ποιες είναι οι εφαρμογές τους, 5) Ποιες επιστήμες ή κλάδοι επιστημών απαιτούν καλή γνώση των Μαθηματικών για να μπορέσει κάποιος να τους σπουδάσει. Για τους συνεργάτες της στήλης: παράκληση! τα κείμενα της στήλης αυτής, ως προς το περιεχόμενό τους και ως προς το επίπεδό τους, θα πρέπει να είναι συμβιβαστά με τα ενδιαφέροντα και το επίπεδο κατανόησης από μέρους των παιδιών.

επιμέλεια: Γιάννης Κερασαρίδης Iα. "τι είναι τα Μαθηματικά;" Προλεγόμενα. Το υλικό το δανειστήκαμε από το κλασικό έργο «Grundlangen der Geometrie» του γίγαντα των Μαθηματικών David Hilbert. Είναι επιλεγμένες περικοπές. Είναι φανερό πως αυτό θα ολοκληρωθεί σε συνέχειες. Τα στοιχεία της Γεωμετρίας και οι πέντε ομάδες αξιωμάτων «Επεξήγηση. Θεωρούμε τρία διαφορετικά τα σημεία, οι ευθείες και τα επίπεδα αναφέρονται συστήματα από «όντα»: τα όντα του πρώτου σαν στοιχεία της Γεωμετρίας του Χώρου ή απλά συστήματος τα ονομάζουμε σημεία και θα τα ως στοιχεία του χώρου. συμβολίζουμε με A,B,C,…˙ του δεύτερου Θεωρούμε ότι, τα σημεία, οι ευθείες και τα επίπεδα συστήματος τα ονομάζουμε ευθείες και θα τα βρίσκονται σε ορισμένες αμοιβαίες σχέσεις και συμβολίζουμε με a,b,c,…˙ τα όντα του τρίτου δηλώνουμε αυτές τις σχέσεις με (κατάλληλες), συστήματος θα καλούμε επίπεδα και θα τα λέξεις, όπως: «κείνται», «μεταξύ», «ίσος», επισημαίνουμε νε α,β,γ,… «παράλληλος», «συνεχής»˙ η ακριβής και, για τους Τα σημεία χαρακτηρίζονται σαν στοιχεία της στόχους των Μαθηματικών, πλήρης περιγραφή Γραμμικής Γεωμετρίας˙ Τα σημεία κι οι ευθείες αυτών των σχέσεων γίνεται εφικτή μέσω των καλούνται στοιχεία της Επίπεδης Γεωμετρίας ενώ αξιωμάτων της Γεωμετρίας» [πηγή: D. Hilbert, «GRUNDLAGEN DER GEOMETRY», από την 7η γερμανική έκδοση (Leipzig, 1930), μετάφραση Σ. Παπαδόπουλος, εκδ. ΤΡΟΧΑΛΙΑ, Αθήνα 1995 ]

Ιβ. "Ευκλείδεια Γεωμετρία, αγάπη μου" ● σημεία μεταπολικά Δίνονται τα τρίγωνα ● σημεία ορθοπολικά Δίνονται τα τρίγωνα ΑΒΓ,ΔΕΖ. Αν α) από τις κορυφές του ΑΒΓ ΑΒΓ,ΔΕΖ. Αν α) από τις κορυφές του ΑΒΓ φέρουμε παράλληλες προς τις πλευρές του ΔΕΖ και φέρουμε κάθετες προς τις πλευρές του ΔΕΖ και αυτές συντρέχουν στο σημείο Μ, β) από τις αυτές συντρέχουν στο σημείο Μ, β) από τις κορυφές του ΔΕΖ φέρουμε παράλληλες προς τις κορυφές του ΔΕΖ φέρουμε κάθετες προς τις πλευρές του ΑΒΓ και αυτές συντρέχουν στο σημείο πλευρές του ΑΒΓ και αυτές συντρέχουν στο σημείο Ν, τότε τα σημεία Μ,Ν λέγονται μεταπολικά Ν, τότε τα σημεία Μ,Ν λέγονται ορθοπολικά σημεία των δύο τριγώνων σημεία των δύο τριγώνων σημεία μεταπολικά σημεία ορθοπολικά σημεία αντίστροφα

● σημεία αντίστροφα Δύο σημεία Α,Α1 θα λέμε (ΟΑ)(ΟΑ1)=ρ ότι είναι αντίστροφα το ένα του άλλου ως προς σημείωση αν συμβεί: α) το ρ>0 τότε το Ο πόλο σημείο Ο και δύναμη αντιστροφής τον βρίσκεται εκτός του τμήματος ΑΑ1, β) το ρ<0 τότε σταθερό αριθμό ρ, αν τα Α,Ο,Α1 είναι συνευθειακά το Ο βρίσκεται μεταξύ Α και Α1 και ισχύει ΙΙα. «Αυτό το ξέρατε;» α) πως συσχετίζεται το όνομα του Ernst Wilhelm Grebe με το "θεώρημα του Vecten"; β) τίνος είναι η διπλανή υπογραφή; [η απάντηση στο τέλος της στήλης] ΕΥΚΛΕΙΔΗΣ Β΄ 90 τ.2/22

------------------------------------------------------------------------------------------------------------------------------------- Homo Mathematicus ----------------------------------------------------------------------------------------------------------------------------------

ΙΙΙ. «οι συνεργάτες της στήλης γράφουν-ερωτούν» 1ο θέμα. Έννοιες για τις συναρτήσεις, του A.N. Kolmogorov Προλεγόμενα [διάβασε πρώτα την 2η από το "Ειδησούλες – Ειδήσεις"]. Το άρθρο αυτό εξηγεί τι εννοούν σήμερα οι μαθηματικοί όταν χρησιμοποιούν τη λέξη «συνάρτηση». Παρόλο που δεν απαιτεί εξειδικευμένες γνώσεις έξω από τα όρια της ύλης του λυκείου, θα πρέπει να προσέξετε την κάθε λέξη του. Επίσης, υποθέτει ότι κατανοείτε τις λέξεις «σύνολο» και «στοιχείο ενός συνόλου» και γνωρίζετε πώς να τις χειριστείτε [η μετάφραση είναι του, σημαντικού μαθηματικού, Μιχάλη Λάμπρου] Εισαγωγή «Όταν ερωτηθεί ένας μαθητής τι είναι χρησιμοποιήσουμε άλλες έννοιες. Στην συνάρτηση συχνά θα απαντήσει ότι «μια πραγματικότητα, μέχρι να κατανοήσουμε τη συνάρτηση μπορεί να δοθεί από έναν πίνακα, ένα σημασία κάποιων εννοιών δεν μπορούμε να γράφημα ή έναν τύπο». Αυτό, φυσικά, δεν είναι ξεκινήσουμε ούτε να διατυπώσουμε κανένα ορισμός. Από την άλλη πλευρά, οι μαθητές που ορισμό. Έτσι η έκθεση οποιασδήποτε μαθηματικής αποφεύγουν έναν σαφή γενικό ορισμό και θεωρίας αρχίζει με την αποδοχή ορισμένων προσπαθούν να περιγράψουν άμεσα τους τρόπους βασικών εννοιών που δεν ορίζονται. Αυτές στη προσδιορισμού μιας συνάρτησης δεν έχουν συνέχεια μπορούν να χρησιμοποιηθούν για τη απόλυτα άδικο. Τα μαθηματικά δεν μπορούν να διατύπωση των ορισμών επόμενων παραγόμενων ξεκινήσουν με ορισμούς. Όταν διατυπώνουμε τον εννοιών» ορισμό μιας έννοιας, είναι αναπόφευκτο να Πώς μπορούμε να εξηγήσουμε. «Πώς μπορούμε, λοιπόν, να εξηγήσουμε σε κάποιον άλλο τη σημασία των βασικών εννοιών; Δεν υπάρχει άλλος τρόπος παρά η παρουσίαση των πραγμάτων που πρέπει να οριστούν μέσω παραδειγμάτων και η εξαντλητική περιγραφή των χαρακτηριστικών ιδιοτήτων τους. Οι περιγραφές αυτές ίσως είναι λίγο ασαφείς σε μερικές λεπτομέρειες, ενώ στην αρχή μπορεί να μην είναι πλήρεις. Βαθμιαία όμως αποτυπώνουν με ικανοποιητική καθαρότητα τη σημασία της έννοιας. Αυτός είναι ο τρόπος με τον οποίο θα Παραδείγματα

προσεγγίσουμε την έννοια της συνάρτησης, θεωρώντας τη ως μια από τις βασικές μαθηματικές έννοιες που δεν ορίζονται τυπικά. [Αργότερα, βέβαια, θα υποστηρίξω ότι συνάρτηση είναι απλώς η απεικόνιση ενός συνόλου σε ένα άλλο σύνολο (του πεδίου ορισμού της στο πεδίο τιμών της). Εδώ όμως η λέξη «απεικόνιση» δεν είναι παρά ένα συνώνυμο της «συνάρτησης». Είναι δύο ονόματα για την ίδια έννοια. Και ο ορισμός της έννοιας που εκφράζει μια λέξη δεν μπορεί να αντικατασταθεί από την ερμηνεία της λέξης μέσω μιας άλλης.]»

«Παράδειγμα 1. 'Έστω ότι τα γράμματα χ και γ δηλώνουν πραγματικούς αριθμούς και ότι το σύμβολο του ριζικού « » δηλώνει την εξαγωγή της (θετικής) τετραγωνικής ρίζας. Τότε η ισότητα y=

1 - x2

(1)

σημαίνει ότι ισχύουν οι συνθήκες x2≤1, y≥0, x2+y2=1

(2)

Τα σημεία με συντεταγμένες που ικανοποιούν αυτές τις συνθήκες αποτελούν το ημικύκλιο του Σχήματος 1. Το Σχήμα 1 παρουσιάζει διαγραμματικά τα επόμενα γεγονότα που μπορείτε να αποδείξετε με καθαρά αλγεβρικό τρόπο. 1. Η εξίσωση (1), για κάθε χ που ικανοποιεί την -1<χ<1, (3) μας επιτρέπει να υπολογίσουμε το αντίστοιχο γ, το οποίο ικανοποιεί τις ανισότητες 0 < γ < 1. (4) » [η συνέχεια στο επόμενο τεύχος] 2ο θέμα. Πρόβλημα μεγίστου-ελαχίστου, του Παναγιώτη Οικονομάκου ΕΥΚΛΕΙΔΗΣ Β΄ 90 τ.2/23

------------------------------------------------------------------------------------------------------------------------------------- Homo Mathematicus ----------------------------------------------------------------------------------------------------------------------------------

Προλεγόμενα Ο φίλος της στήλης Παναγιώτης Οικονομάκος συνεχίζει να τροφοδοτεί τη στήλη με "έξυπνα" θέματα. Αυτή τη φορά ασχολείται με ένα πρόβλημα "ελαχίστου" εκφώνηση Μια ευθεία (ε) εφάπτεται σε σημείο Α μιας έλλειψης (με εστίες Ε1,Ε2). Δείξτε ότι το άθροισμα των αποστάσεων των σημείων της (ε), από τις εστίες της έλλειψης γίνεται ελάχιστο, όταν το τυχαίο σημείο συμπέσει με το Α. Όμως, από το τρίγωνο απόδειξη Η έλλειψη είναι κυρτό σχήμα˙ άρα η (ε) Α1ΜΕ2 έχουμε αφήνει όλη την έλλειψη προς το ίδιο ημιεπίπεδο. Α1Ε2<ΜΑ1+ΜΕ2 (2) Αυτό σημαίνει πως όλα τα σημεία της (ε) (εκτός Προσθέτουμε και στα δύο μέλη της (2) το Α1Ε1 από το Α), είναι εξωτερικά σημεία ως προς την οπότε έχουμε: Α1Ε1+ Α1Ε2< Α1Ε1+ ΜΑ1+ΜΕ2 ή έλλειψη. Θεωρούμε ένα τέτοιο σημείο Μ. Το ευθ. Α1Ε1+ Α1Ε2< ΜΕ1+ ΜΕ2 (3) τμήμα ΜΕ1, τέμνει την έλλειψη σε σημείο Α1. Τέλος από (1),(3) έχουμε ΑΕ1+ΑΕ2< ΜΕ1+ ΜΕ2 Επειδή το Α1 βρίσκεται πάνω στην έλλειψη, θα είναι: ΑΕ1+ΑΕ2=Α1Ε1+Α1Ε2 (1) 3ο θέμα. Μια περίληψη άρθρου του Αλκιβιάδη Τζελέπη με θέμα: «Το Πρόβλημα στα Μαθηματικά της Α΄ Λυκείου» Προλεγόμενα. Από το φίλο της στήλης Αλκιβιάδη Π.Π. Λυκείου της Ευαγγελικής Σχολής, με θέμα της Άλγεβρας στην Α΄ Τζελέπη (Αθήνα) λάβαμε και δημοσιεύουμε μια "Διδασκαλία περίληψη ενός άρθρου του με θέμα «Το Λυκείου. Πειραματισμοί και Πρακτικές", στα Πρόβλημα στα Μαθηματικά της Α΄ πλαίσια του Εργαστηρίου Άλγεβρας της Σχολής Λυκείου».Το άρθρο αποτελεί μία περίληψη της Ευαγγελικής εισήγησης που παρουσιάστηκε στην ημερίδα του (http://algebrateacherlab.blogspot.gr/) Περίληψη Το παρόν άρθρο αναφέρεται στον τρόπο εφαρμογής μίας πρότασης διδασκαλίας μαθηματικών προβλημάτων σε μαθητές της Α΄ Λυκείου, στο μάθημα της Άλγεβρας. Αφετηρία της διδασκαλίας αποτέλεσε η διαπραγμάτευση ενός ανοικτού προβλήματος σε ομάδες μέσα στην τάξη και η υποδειγματική προτυποποίηση ενός προβλήματος Φυσικής. Η ανάθεση εργασιών με

επιλεγμένα προβλήματα κατά τη διάρκεια της σχολικής χρονιάς και η παρουσίαση εργασιών από τους μαθητές, ανά τακτά χρονικά διαστήματα, έδωσαν τη δυνατότητα πραγματοποίησης συζητήσεων και ανταλλαγής επιχειρημάτων, ενώ παράλληλα συνεισέφεραν στην εμπέδωση των μαθηματικών εννοιών

1. Η χρησιμότητα των προβλημάτων Τα προβλήματα είναι χρήσιμο να διδάσκονται, γιατί ▪ Προάγουν τη συνεργατικότητα ▪ Εξάπτουν τη φαντασία ▪ Αναπτύσσουν τη μαθηματική σκέψη ▪ Βοηθούν στην εκμάθηση του πεδίου γνώσης και της μαθηματικής γλώσσας ▪ Αναδεικνύουν τη σχέση των μαθηματικών με άλλα επιστημονικά πεδία ▪ Αποτελούν έναν από τους καλύτερους τρόπους για να εισαχθεί μία νέα έννοια στα μαθηματικά ▪ Είναι κομβικής σημασίας στην κινητοποίηση των μαθητών, γιατί κίνητρα που έχουν ως βασικές συνιστώσες την επιθυμία για εξερεύνηση και την εργασία σε ομάδες υπηρετούνται ιδανικά μέσα από τα προβλήματα ▪ Δείχνουν τέλος έναν από τους λόγους για τους οποίους αξίζει να κάνουμε μαθηματικά 2. Η πρακτική της διδασκαλίας 1ο Βήμα: Ένα πρόβλημα ανοικτού τύπου, όπως: «Πόσες διαγώνιους έχει ένα πολύγωνο με ν κορυφές» Δημιουργούμε ανομοιογενείς ομάδες τριών ή τεσσάρων ατόμων στις οποίες παρουσιάζεται το πρόβλημα. Δε δίδεται καμία επεξήγηση ή βοήθεια από τον εκπαιδευτικό και παρέχεται εύλογο χρονικό διάστημα για τη διαπραγμάτευσή του.

Η κάθε ομάδα αναφέρει την δική της προσέγγιση στο πρόβλημα, γίνεται ανταλλαγή απόψεων και επιχειρημάτων με κατάληξη την ορθή λύση του. Αυτό που ενδιαφέρει κυρίως είναι η πορεία προς την επίλυση και όχι αν το αποτέλεσμα που έδωσε κάθε ομάδα είναι το σωστό.

ΕΥΚΛΕΙΔΗΣ Β΄ 90 τ.2/24

------------------------------------------------------------------------------------------------------------------------------------- Homo Mathematicus ----------------------------------------------------------------------------------------------------------------------------------

2ο Βήμα: Προτυποποίηση ενός προβλήματος Φυσικής Αφιερώνεται μία διδακτική ώρα για την επίλυση ανάλυση των δεδομένων και των αιτούμενων του ενός προβλήματος, δίνοντας ιδιαίτερη βαρύτητα προβλήματος, των προαπαιτούμενων γνώσεων από στη βήμα προς βήμα προσπάθεια προτυποποίησης. τον τομέα της φυσικής, τη μετάβαση από το Ένα πρόβλημα φυσικής, από την κίνηση για λεκτικό επίπεδο στο τυπικό μαθηματικό, μέχρι να παράδειγμα, είναι ενδεικτικό για τα ζητήματα που καταλήξουμε στην εξίσωση που μοντελοποιεί και θέλουμε να θίξουμε. Ζητήματα που αφορούν στην επιλύει το πρόβλημα. 3ο Βήμα: Ανάθεση εργασιών ανά δεκαπενθήμερο Ανατίθενται εργασίες ατομικές ή ομαδικές, σε πλούσια ιστορία των μαθηματικών, από εθελοντική βάση, με προβλήματα που διατρέχουν διαφορετικά επιστημονικά πεδία και αν είναι όλη την ύλη και συνδέουν τις καινούργιες έννοιες δυνατόν διαθεματικά. με τις προηγούμενες. Προβλήματα από την 4ο Βήμα: Η παρουσίαση των εργασιών στην τάξη: Οι ίδιοι οι μαθητές παρουσιάζουν τις εργασίες τους οι μαθητές για τα μαθηματικά, δημιουργείται στην τάξη και έτσι αναδεικνύεται η κίνητρο συνέχισης της μελέτης και επιπλέον υπευθυνότητα, αναγνωρίζεται και επιβραβεύεται η ελέγχεται η επίτευξη των στόχων της προσπάθεια τους. Δίνεται η ευκαιρία να μιλήσουν διδασκαλίας.» IV. Ειδησούλες – Ειδήσεις 1η. Η "σιδερόφρακτη" Άλγεβρα Είναι γνωστό πως ο Έκτορας Κακναβάτος (κατά κόσμον Γιώργος Κοντογιώργης), ήταν μαθηματικός και δίδασκε Μαθηματικά στην ιδιωτική εκπαίδευση και στα φροντιστήρια. Παράλληλα, εθεωρείτο για τη χώρα μας, ως αντιπροσωπευτικός εκπρόσωπος του υπερρεαλισμού (σουρεαλισμού) στην ποίηση, . Στην ποίησή του, διάχυτη είναι η μαθηματική σκέψη. Ο γράφων είχε την εύνοια της τύχης. να εργάζεται στα φροντιστήρια με τον Γ. Κοντογιώργη. Μετά από κάθε συζήτηση μαζί του 2η. Επέτειος για έναν "Μεγάλο" Με την ευκαιρία της επετείου των 26 χρόνων από το θάνατο του γίγαντα της μαθηματικής σκέψης, A.N. Kolmogorov (25/4/1903-20/10/1987), παραθέσαμε παραπάνω, ένα από δύο αποσπάσματα από ένα έργο του που είχε δημοσιευθεί στο περιοδικό 3η. Μαθηματικά και ποίηση Προ ημερών αποκτήσαμε, έστω και καθυστερημένα, ένα εξαιρετικά ενδιαφέρον βιβλίο με τίτλο «μαθηματικά και ποίηση, από τον Αρχιμήδη στον Ελύτη». Το βιβλίο αυτό είναι έργο του σημαντικού Θεσσαλονικιού μαθηματικού

έβγαινα "σοφότερος". Να ένα ποίημά του για την Άλγεβρα, γραμμένο με την υπερρεαλιστική τεχνοτροπία. ΑΛΓΕΒΡΑ Πέρα κατά τη δημοσιά φάνηκε πρώτα στήλη κουρνιαχτός ως τα μεσούρανα. Δεν άργησε πολύ. Ο δρόμος έφερνε το ποδοβολητό το χουγιατό της κλείνανε παράθυρα κατέβαιναν ρολά. Σιδηροντυμένη έμπαινε πια στην πόλη η εξίσωση.

QUANTUM, με τίτλο «Περί πεδίου τιμών…και ορισμού». (τόμ. 5, τεύχ. 5. Η μετάφραση ήταν του Μιχάλη Λάμπρου). «…Ο A.N. Kolmogorov ασχολήθηκε με τη συναρτησιακή ανάλυση, την πραγματική ανάλυση, την τοπολογία, τη μαθηματική λογική, τη θεωρία πιθανοτήτων και τα εφαρμοσμένα μαθηματικά (ιδίως με τη θεωρία διαταραχών, την κυβερνητική και τη θεωρία πληροφοριών)…» Στέφανου Μπαλή. Ο Σ.Μ. δίδαξε Μαθηματικά στη Μέση εκπαίδευση και στο πανεπιστήμιο Θεσσαλονίκης. Διαβάζοντάς το κατανόησα τη σχέση Μαθηματικών και Ποίησης.. Στο επόμενο τεύχος θα επανέλθουμε.

ΙΙβ. Απάντηση στο «Αυτό το ξέρατε;»

α) Πάνω στις πλευρές τριγώνου ΑΒΓ (και προς το εξωτερικό αυτού), κατασκευάζουμε τα τετράγωνα ΒΓΖΘ, ΑΓΗΙ, ΑΒΔΕ, με αντίστοιχα κέντρα Α1,Β1,Γ1. Αποδεικνύεται ότι οι ΑΑ1,ΒΒ1,ΓΓ1, διέρχονται από το ίδιο σημείο V. Ο Ernst Wilhelm Grebe ήταν ο πρώτος που ονόμασε το σημείο V, σημείο Vecten β) Αυτή η υπογραφή είναι του ημίθεου της μαθηματικής επιστήμης Leonhard Euler

ΕΥΚΛΕΙΔΗΣ Β΄ 90 τ.2/25

Α΄ΛΥΚΕΙΟΥ

Εξισώσεις – Ανισώσεις Λαζαρίδης Χρήστος

Το άρθρο μελετάται ευκολότερα, αν γνωρίζουμε πολύ καλά τα κεφάλαια 3 και 4 του σχολικού βιβλίου. Οι απλές περιπτώσεις επίλυσης εξισώσεων και ανισώσεων θεωρούνται γνωστές. Επειδή τα αντίστοιχα κεφάλαια δεν είναι δυνατόν να εξαντληθούν σε ένα απλό και σύντομο άρθρο, παρουσιάζονται ορισμένα στοιχεία Θεωρίας και είδη ασκήσεων που δεν αναφέρονται στο σχολικό. Η εκτενής εισαγωγή, οφείλεται στην προσπάθεια αποσαφήνισης των εννοιών. Επεξηγηματικό Σχόλιο: Για καλύτερη κατανόηση των παραδειγμάτων που ακολουθούν προτάσσουμε το εξής σχόλιο. Αν θεωρήσουμε δυο συναρτήσεις f: A→  και g: B→  με Α∩Β  , τότε λέμε ότι ορίζεται (έχει νόημα), η εξίσωση f(x) = g(x) και το σύνολο D = Α∩Β θα το λέμε σύνολο ορισμού της. Αν όμως Α∩Β =  , τότε θα λέμε ότι δεν ορίζεται (δεν έχει νόημα) τέτοια εξίσωση. Ειδικότερα, αν Α = , τότε δεν ορίζεται καν συνάρτηση f, πολύ μάλλον εξίσωση της μορφής f(x) = g(x), π.χ.  x 2  1  x . Ομοίως, αν Β = . Φυσικά και οι δυο αυτές περιπτώσεις, εντάσσονται στην περίπτωση Α∩Β = . Στο εξής λοιπόν όταν ζητάμε να λυθεί μια εξίσωση, ουσιαστικά εννοούμε να εξετασθεί αρχικά αν D = Α∩Β ≠ , και στη συνέχεια να ασχοληθούμε με τη διαδικασία επίλυσης. Προφανώς η επίλυση της εξίσωσης δεν απαιτεί την ισότητα των συναρτήσεων f, g η οποία ως γνωστόν ορίζεται μόνον όταν Α = Β, αλλά την εύρεση του συνόλου L={xD |f(x) = g(x)}, το οποίο ονομάζεται σύνολο λύσεων (ριζών) της εξίσωσης. Πρόκειται λοιπόν για μια «εν δυνάμει ισότητα», που θα έπρεπε να συμβολίζεται διαφορετικά, δηλαδή για έναν προτασιακό τύπο που δεν γνωρίζουμε για ποιες τιμές του x αληθεύει (καθίσταται αληθής πρόταση) και ζητάμε να τις προσδιορίσουμε.

Ο εκλεκτός μου φίλος, διαπρεπής συνάδελφος και συγγραφέας Αντώνης Κυριακόπουλος έχει εισηγηθεί τον πολύ επιτυχημένο κατά τη γνώμη μου συμβολισμό, f (x)  g(x) , μέχρι να κατανοηθεί πλήρως η έν;

νοια της εξίσωσης. Εξάλλου στην περίπτωση Α∩Β   αντί της έκφρασης: «Δεν ορίζεται εξίσωση» χρησιμοποιεί σαν ισοδύναμη την έκφραση: «Η εξίσωση έχει σύνολο ορισμού το κενό σύνολο ». Αν και συμφωνούμε στο ότι η περίπτωση D =  προφανώς διαφέρει από την περίπτωση (D ≠ , L = ) της αδύνατης εξίσωσης, απλώς για περιορισμό των περιπτώσεων εντάσσει και τις δυο στις αδύνατες εξισώσεις. Ανάλογα ορίζεται η αδύνατη ανίσωση καθώς και η ταυτοτική ανίσωση η οποία μεταπίπτει σε μόνιμη ανισότητα, όπως ακριβώς η ταυτοτική εξίσωση μεταπίπτει σε ταυτότητα στο D. Αποφεύγεται ο όρος αόριστη ανίσωση, αφού το σύνηθες είναι μια ανίσωση να έχει άπειρες λύσεις.

Γιώργος Σ. Τασσόπουλος

Εισαγωγή  Η επίλυση μιας εξίσωσης (ανίσωσης) αρχίζει με την εύρεση του συνόλου ορισμού D της εξίσωσης (ανίσωσης) και τελειώνει με την εύρεση του συνόλου λύσεων της L  D. Παράδειγμα 6 Να λύσετε την εξίσωση x   5 (1) . x Λύση Το σύνολο ορισμού είναι D  R * . Για κάθε x  A ,έχουμε: (1)  x 2  6  5x  x 2  5x  6  0   (x  2 η x  3) . Το σύνολο λύσεων είναι L  {2,3}

 Μια εξίσωση δεν ορίζεται, αν, και μόνο αν, D    Μία εξίσωση λέγεται αδύνατη, αν , και μόνο αν, ισχύει: ( D   και L   ) Παράδειγμα 1) Να λύσετε την εξίσωση x  1 

1 1 x

(1) 

Λύση Το σύνολο ορισμού D, προκύπτει από την επίλυση του παρακάτω συστήματος:  x 1 0 x 1    1 x  0 ,δηλαδή του x 1 που είναι αδύναx 1    1 x  0

ΕΥΚΛΕΙΔΗΣ Β΄ 90 τ.2/26

-------------------------------------------------------------------------------------------------------------------- Μαθηματικά για την Α΄ Λυκείου -----------------------------------------------------------------------------------------------------------------

το. Άρα, D   και συνεπώς τέτοια εξίσωση δεν ορίζεται. Αναλυτικότερα το πρώτο μέλος έχει σύνολο ορισμού το Α= [1, ) και το δεύτερο το Β= (,1) ,οπότε D = ΑΒ = . 2) Να λύσετε την εξίσωση x  2 

4  0 (1) . x 1

Λύση Το σύνολο ορισμού είναι D  R  1

Για κάθε x  D ,έχουμε: (1)  (x  1)(x  2)  4  0  x 2  x  2  0 . Η διακρίνουσα της είναι:   1  8  7  0 . Άρα L   , δηλαδή η (1) είναι αδύνατη.  Μία εξίσωση λέγεται ταυτοτική ,αν, και μόνο αν, ισχύει : D   και L  D Παράδειγμα Να λύσετε την εξίσωση x  5  5  x (1) . Λύση Το σύνολο ορισμού D, προκύπτει από την επίλυση του συστήματος: x  5  0 x  5 . Άρα D  5 . δηλαδη του   5  x  0 x  5 Για x = 5 προφανώς η (1) ισχύει. Άρα L  5  A ,δηλαδή η (1) είναι ταυτοτική.  Μία εξίσωση λέγεται αόριστη ή έχει άπειρες λύσεις αν, και μόνο αν, το L έχει άπειρα στοιχεία ( είναι απειροσύνολο) . Παράδειγμα 1) Να λύσετε την εξίσωση x   x (1) . Λύση D = R. Για κάθε x  D ,έχουμε: (1)  x  0 . Άρα L  (, 0] . Η (1) είναι αόριστη (και όχι ταυτοτική). 2) Να λύσετε την εξίσωση 2x x 5x  12 (1)   1 3x  6 2x  4 4  2x

Λύση D = R{2}. Για κάθε x  D ,έχουμε: 2x x 5x 12 (1)    1  3(x  2) 2(x  2) 2(x  2) 2x x 5x 12 (1)    x  2   3 2 2 (1)  4x  3x  6x 12  5x 12  0x  0   x  R  {2}   D  .

Η εξίσωση (1) είναι αόριστη ΚΑΙ ταυτοτική.

 Στην περίπτωση ανισώσεων έχουμε αντίστοιχους ορισμούς εκτός της περίπτωσης αόριστης ανίσωσης. Για την περίπτωση ταυτοτικής ανίσωσης χρησιμοποιείται και ο όρος μόνιμη ανισότητα. Παράδειγμα Να λύσετε την ανίσωση

1 1 x  24 (1)    2x 3x 6x 2

Λύση D  R * Για κάθε x  D ,έχουμε: 1 1 x  24 3x  2x  x  24 0 (1)    2  0  2x 3x 6x 6x2  24  6x2  0  x2  0  x  0 . Άρα, L = R*(= D) και επομένως, η ανίσωση (1) είναι μόνιμη ανισότητα . Η εξίσωση x v  α, v  N* ,α  R (1) D=R Ι) α > 0  ν άρτιος. Τότε: (1) (x  v α η x  v α)

 ν περιττός. Τότε: (1)  x  v α ΙΙ) α < 0  ν άρτιος. Τότε: η (1) είναι αδύνατη.  ν περιττός. Τότε: (1)  x   v  α III) α = 0. Τότε: (1)  x  0 Παραμετρικές εξισώσεις - ανισώσεις. Άσκηση 1 Να λύσετε την εξίσωση x7  1   λ(λ  1) (1) Λύση D=R. Για κάθε x  D έχουμε:

1  x 7   2    1

Αλλά:  λ2  λ  1  0 ,για κάθε λ  R , αφού Δ = 18 = 7 < 0 και α = 1 < 0. Για κάθε x  D λοιπόν, έχουμε: (1)  x7  λ2  λ 1  x  7 (λ2  λ 1)

 x   7 λ2  λ  1 . Άσκηση 2 Να λύσετε την εξίσωση x4  λ2  1 (1) Λύση D = R. Διακρίνουμε τις περιπτώσεις:  Αν λ2  1  0 ,δηλαδή, (λ  1 η λ  1) , τότε: (1)  (x  4 λ2  1 η x   4 λ2  1)  Αν λ2  1  0 ,δηλαδή 1  λ  1 , τότε η ανίσωση (1) είναι αδύνατη.

ΕΥΚΛΕΙΔΗΣ Β΄ 90 τ.2/27

-------------------------------------------------------------------------------------------------------------------- Μαθηματικά για την Α΄ Λυκείου -----------------------------------------------------------------------------------------------------------------

 Αν λ2  1  0 , δηλαδή (λ  1 η λ  1) , τότε: (1)  x  0 . Άσκηση 3 Να λυθεί η 4x 2  4λx  3λ  2  0, λ  R (1) . Λύση D = R. Η διακρίνουσα της εξίσωσης αυτής είναι: Δ  (4λ)2  4  4  (3λ  2)  16(λ2  3λ  2) Διακρίνουμε τις περιπτώσεις::  Αν Δ  0, δηλαδή (λ  1 ή λ  2) Τότε:

Λύση: D = R Πρώτος Τρόπος Για κάθε x  D ,έχουμε:

4λ  4 λ2 3λ  2 λ  λ2 3λ  2 x  8 2  Αν Δ < 0, δηλαδή 1 < λ < 2 Τότε: η εξίσωση (1) είναι Αδύνατη. Άσκηση 4 Να λύσετε τις ανισώσεις: α) λ2 (x  1)  (3λ  2)x  4, λ  R (1) β) 0, 25(x  1)  0,5x  4 (2) Λύση α) D = R. Για κάθε x  D ,έχουμε: (1)  λ2 x  λ2  3λx  2x  4 

Δεύτερος Τρόπος

(1)  x 

(λ2 3λ2)x  λ2 4 (λ1)(λ2)x (λ2)(λ2) (3)  Έστω ότι (λ1)(λ2) > 0, δηλαδή: λ2 (λ < 1 ή λ > 2). Τότε: (3)  x  . λ 1  Έστω ότι : (λ1)(λ2) < 0 δηλαδή: λ2 . 1 < λ < 2. Τότε: (3)  x  λ 1  Έστω ότι (λ1)(λ2)=0, δηλαδή: (λ=1 ή λ=2). Τότε: Αν λ=1 τότε: (1) 0x 3  x R , μόνιμη ανισότητα. Αν λ=2 τότε: (1)  0x  0 , αδύνατη. β) Η ανίσωση (2) προκύπτει από την (1), για λ=0,5 < 1,επομένως από τα προηγούμενα, έ0,5  2 2,5 χουμε:  2  x  x  x  5 . 0,5 1 0,5 Η εξίσωση f (x)  g(x) (1)

Για κάθε x  A , έχουμε:  g ( x)  0  g ( x)  0  (1)    2 2  f ( x)  g ( x)  f ( x)  g ( x)  g ( x)  0  g ( x)  0  (1)   η   f ( x)  g ( x)  f ( x)   g ( x) Άσκηση 5 Να λύσετε την εξίσωση x  3  3x  1 (1)

1  x  3x 1 0  (1)   3 2 2 (x 3)  (3x 1) x2 6x 9  9x2 6x 1 1 1    x  x   x 1 3  3 8x2  8 x 1 η x 1 3x  1  0  3x  1  0   (1)   η   x  3  3x  1  x  3  3x  1 1  1 1  1    x x  x x  3 η  3  3 η  3  x 1  2  2x 4x  4  x  1  x  1 Η εξίσωση

f (x)  g(x) (1)

Για κάθε x  D ,έχουμε:  g ( x)  0 (1)   2  f ( x)  g ( x) Αντίστοιχα για την ανίσωση

f (x)  g(x)

Άσκηση 6 Να λύσετε την εξίσωση x  2  x  1  x (1) Λύση D  {x  R | x  0 και x  2  x  0} Δεν είναι απαραίτητο το Α να παρουσιαστεί με αναγραφή των στοιχείων του, δηλαδή ως διάστημα ή ένωση διαστημάτων. Για κάθε x  D ,έχουμε:  1 x 0  x 1 (1)   2 x2 x (1 x) x2 x 1x2 x  x 1 , Αδύνατο. Άρα και η (1) αδύνατη.   x 1 Η ανίσωση f (x)  g(x) (1)

Για κάθε x  D ,έχουμε:  g ( x)  0  g ( x)  0  (1)   ή  2 2  f ( x)  g ( x)  f ( x)  g ( x)  (1)   g (x)  f (x)  g (x) . Παρατηρήστε ότι η ισοδυναμία x      x   ισχύει και για   0 , αφού τότε και τα δύο μέλη της καθίστανται ψευδείς προτάσεις για κάθε x   .

ΕΥΚΛΕΙΔΗΣ Β΄ 90 τ.2/28

-------------------------------------------------------------------------------------------------------------------- Μαθηματικά για την Α΄ Λυκείου -----------------------------------------------------------------------------------------------------------------

Άσκηση 7 Να λύσετε την ανίσωση: x 2  2  x 2  3 (1) . Λύση: D = R. Για κάθε x  D ,έχουμε: x 2  2 0

Άρα, (2)  3  x  1 η 1  x  3 xD

 (3  x  1 η 1  x  3) Άσκηση 10 x2  4  0 (1) x1

(1)  x  2  x  3  (x  2)  (x  3) 

Να λύσετε την ανίσωση

x 4  4x 2  4  x 4  6x 2  9  10x 2  5  0  1 1 2x 2  1  0   x 2 2 Άσκηση 8 Να λύσετε την ανίσωση x   x 2  2x (1) . Λύση: D = R. Πρώτος Τρόπος Για κάθε x  D ,έχουμε:   x 2  2x  0  x2  2x  0  2 (1)   2 2 2 2 3 4  x  (2x  x )  x  4x  4x  x

Λύση: D = R{1} Για κάθε x  D ,έχουμε: x  1  0 , οπότε:

2

2

2

2

2

2

0 x 2  x(x  2)  0   4  2 2 3 2  x  4x  3x  0  x (x  4x  3)  0  0 x 2  0 x 2    x0  x2  4x  3  0  x  1 η x  3  0  x  2  0  x  2  0  x 1  η   x 1  x  3 Δεύτερος Τρόπος (1)  x 2  2x  x   x 2  2x   x 2  2x  x  x 2  3x  0     2 2  x   x  2x  x x0 0  x  3  0  x 1  0  x  1 f (x) Η ανίσωση  0 (1) g(x) Για κάθε x  D , έχουμε: (1)  f (x)  g(x)  0 Άσκηση 9 x2  1 Να λύσετε την ανίσωση: 2  0 (1). x 9 Λύση D = R{3,3}. Για κάθε x  D ,έχουμε: (1)  (x 2  1)(x 2  9)  0  (x  1)(x  1)(x  3)(x  3)  0 (2)

(1)  x 2  4  0   x  2  x  2   0 xD

 2  x  2  2  x  1 η 1  x  2 . Γενικές Ασκήσεις Άσκηση 11 Να λύσετε την

7x10  3x2  2x  2  1(1) 7x10  5x2  x  1

Λύση D  {x  R | 7x10  5x 2  x  1  0} Για κάθε x  D ,έχουμε: (1)  7x10  3x 2  2x  2  7x10  5x 2  x  1 1 xD 1  2x2  x 1  0  (x  1 η x   )  x   2 2 (Δεκτή τιμή γιατί;) Παρατηρείστε, ότι το σύνολο ορισμού D δεν είναι εύκολο να παρουσιαστεί με αναγραφή. Με παρουσίαση του D με περιγραφή το πρόβλημα παρακάμπτεται. Ο έλεγχος των λύσεων αν ανήκουν ή όχι στο D, είναι απαραίτητος. Άσκηση 12 Να λύσετε την x  1  α  1,α  R (1) Λύση: D = R. Για κάθε x  D ,έχουμε: (1)  ( x  1  α  1 η x  1  α  1)  x  1  α  1 (2) η x  1  α  1 (3)

 Έστω ότι: α < 1, οπότε: α+1 < 0 και α1 < 0. Τότε, οι εξισώσεις (2) και (3) είναι αδύνατες. Άρα και η (1) είναι αδύνατη.  Έστω ότι: 1< α < 1, οπότε: α+1 > 0 και α1 < 0. Τότε, η (3) είναι αδύνατη οπότε: (1)  (2)  x 1  α 1 η x 1  (α 1)  (x  α  2 η x  α)  Έστω ότι: α > 1, οπότε α1 > 0 και α+1 > 0. Τότε, έχουμε: (1) x 1 α 1 η x 1(α 1) η x  1  α  1 η x  1  (α  1)   (x  α η x  α  2 η x  α  2 η x  α)

ΕΥΚΛΕΙΔΗΣ Β΄ 90 τ.2/29

-------------------------------------------------------------------------------------------------------------------- Μαθηματικά για την Α΄ Λυκείου -----------------------------------------------------------------------------------------------------------------

 Έστω ότι: α = 1. Τότε, α+1 = 0 και α1 = 2 < 0, οπότε: (1)  (2)  x  1  0  x  1  Έστω ότι: α = 1. Τότε, α1 = 0 και α+1 = 2 > 0, οπότε: (1)  x 1  2 η x 1  0) x  3 η x 1η x 1 α  0 Με α, β  0 , ισχύει: α  β  0   β  0 α  0 β  0

Με α, β  0 , ισχύει: α  β  0  

Εφαρμογές. Με ,   R , ισχύουν:  α  β  0  α  β  0.

Άσκηση 16 Να λύσετε την ανίσωση (x  1)2  3 x  1  2  0 (1)

 α 2  β 2  0  α  β  0.  α 2  β  0  α  β  0. Αν, επιπλέον α, β  0 και ν, μ  N* , ισχύει : v

Να υπολογίσετε το μ, έτσι ώστε f(x) > 0,για κάθε x  R και το g(x) να μην έχει πραγματικές ρίζες. Λύση Έχουμε: α1 = 1 > 0, α2 = 1  0 Οι διακρίνουσες των τριωνύμων f(x),g(x) είναι, αντίστοιχα: Δ1 = μ21 και Δ2 = μ2μ, οπότε: 1  0 f (x)  0,  κάθε x      g(x) δεν έχει ρίζες στο   2  0  2  1  0  1    1  2   0   1     0 0    1

α  μ β  0  α  β  0.

Λύση D = R. Για κάθε x  D ,έχουμε: 2 (1)  x  1  3 x 1  2  0 (2).

Άσκηση 13 Να λύσετε την εξίσωση:

Θέτουμε : x  1  t , οπότε αναγόμαστε στην

10x 5  x 2  8x  1  11 1  x 2  0 (1) Λύση D  {x  R |10x 5  x 2  8x  1  0 και 1  x 2  0} Για κάθε x  D ,έχουμε: 10x5 x2 8x 1 0 (1)  1x2  0 

Έτσι έχουμε:

9

10x5  x2  8x 1  0 10x5  x2  8x 1  0    x 1 x  1   4  0  0  0     x  1 (Δεκτή τιμή).  x  1 x  1 Άσκηση 14 Να λύσετε την ανίσωση x 2  5x  6  x 2  6x  8  0 . (1)

Λύση D=R Για κάθε x  D ,έχουμε: 2 x 5x 6  0 (x  2 ή x=3) (1)  2  x2 x 6x 8  0 (x  2 ή x=4) Άσκηση 15 1 4

Έστω τα τριώνυμα f (x)  x 2  μx  , g(x)  x 2  μx 

μ , όπου μ  R . 4

επίλυση τις ανίσωσης: t 2  3t  2  0 (3) Προφανώς:  3   t  1 t  2   0  1  t  2 .  x  1  1  (2)  1  x  1  2    x  1  2  x  1  1 ή x-1  -1  x  2 ή x  0    2  x  1  2  1  x  3   2  x  3 ή 1  x  0  .

Άσκηση 17 Θεωρούμε τα τριώνυμα: fv(x) = x2λx+v2, όπου λ  R με λ  2v , ν = 1,2,3,… Να αποδείξετε ότι η ανίσωση f1(x) f2(x)··· f2013(x) > 0 (1),είναι ταυτοτική. Λύση Για κάθε   N* ,η διακρίνουσα του τριώνυμου f  (x) είναι: Δ ν  λ2  4v 2  0 , διότι 2

λ  2v  λ  (2v) 2  λ2  4v 2 . Επειδή ο συντελεστής του x2 είναι α =1 > 0, συμπεραίνουμε ότι για κάθε   N* και για κάθε x  R , ισχύει: fv(x) > 0. Έτσι, η (1) ισχύει για κάθε x  R( Α) επομέ-

ΕΥΚΛΕΙΔΗΣ Β΄ 90 τ.2/30

-------------------------------------------------------------------------------------------------------------------- Μαθηματικά για την Α΄ Λυκείου -----------------------------------------------------------------------------------------------------------------

νως είναι ταυτοτική. Άσκηση 18 Να λύσετε την εξίσωση: x  1  x  1 (1) Λύση Το σύνολο ορισμού της εξίσωσης είναι το σύνολο λύσεων του συστήματος : x  1  0 x 1 , δηλαδή του    x0 x  0 Άρα, D = [1,+). Για κάθε x  D ,έχουμε: (*)

2

(1)  x  1  1  x  x 1  (1 x)2 

x 1  1 2 x  x  2 x  2  x  1  x  1. Αντιστρόφως. Όπως βρίσκομαι εύκολα, για x = 1, η εξίσωση (1) επαληθεύεται. Άρα, μοναδική λύση της εξίσωσης είναι 1 Σημειώσεις: Η επαλήθευση ότι ο αριθμός 1 είναι ρίζα της εξίσωσης περιλαμβάνεται στην διαδικασία επίλυσης της εξίσωσης. Διαφορετικά, αφού έχουμε χρησιμοποιήσει συνεπαγωγές και όχι ισοδυναμίες, δεν θα είμαστε βέβαιοι ότι πράγματι το 1 είναι ρίζα της εξίσωσης. Η ισοδυναμία διακόπτεται στο σημείο, όπου υπάρχει ο αστερίσκος (*). Για την ακρίβεια για κάθε α,β  R ισχύει: α = β  α2 = β2 αλλά δεν ισχύει πάντοτε: α2 = β2  α = β ?, όπως φαίνεται στο παρακάτω αντιπαράδειγμα (2)2 = 22, αλλά 2 ≠ 2. Άσκηση 19 Έστω το τριώνυμο f(x) = 4x24μx+6μ9, όπου μ  R . Να βρείτε το μ, έτσι να ισχύει: f(x)  0, για κάθε x  R . Λύση Στο τριώνυμο f(x) ο συντελεστής του x2 είναι α = 4 > 0 και η διακρίνουσά του είναι: Δ = 16μ216(6μ9) = 16(μ26μ+9) = 16(μ3)2 Έχουμε:  x  R)  (f (x)  0, για καθε Δ  0  16(μ  3) 2  0  μ  3 Άσκηση 20 Δίνεται το τριώνυμο: 4x 2  4λx+5λ, όπου λ  R . α) Να βρείτε τις τιμές του λ για τις οποίες, το τριώνυμο έχει δύο ρίζες πραγματικές και άνισες.

β) Να εξετάσετε αν υπάρχει τιμή του λ, για την οποία το τριώνυμο έχει δύο πραγματικές ρίζες x1, x2 με: x1+ x2= x1 x2 1. γ) Αν Α είναι ένα ενδεχόμενο ενός δειγματικού χώρου Ω και Α΄ το συμπληρωματικό του, να αποδείξετε ότι για κάθε xR ισχύει: 4x2  4P(A)x+5P(A) 4x2  4P(A΄)x+5P(A΄) 4x2  4P(Ω)x+5P(Ω)  0

Λύση α=4>0 Η διακρίνουσα του τριωνύμου είναι : Δ = 16λ216(5λ) =16(λ25λ)=16λ(λ5). α) Το τριώνυμο έχει δύο ρίζες πραγματικές και άνισες, αν και μόνο αν Δ>0. Αλλά: Δ > 0  16λ(λ5) > 0  (λ < 0 ή λ > 5). β) Το ζητούμενο συμβαίνει αν, και μόνο αν

  0 ()     x x x x 1 2 1 2  1   0    5   Αλλά, ()   4 5 1     4 4   0    5   0    5    , 5   4    1   4 που είναι αδύνατο. Άρα δεν υπάρχει τέτοια τιμή του λ. γ) Το γινόμενο αποτελείται από τρεις παράγοντες, που είναι της μορφής του τριωνύμου της άσκησης, όπου λ ένας από τους αριθμούς Ρ(Α), Ρ(Α΄), Ρ(Ω) αντίστοιχα. Επειδή   [0,1] , δηλαδή Δ  0 και α = 4 > 0, το γινόμενο αποτελείται από έναν μη αρνητικό παράγοντα και από δύο θετικούς. Άρα είναι μη αρνητικό.

Βιβλιογραφία:





“Για ένα κοινό τρόπο έκφρασης στα Μαθηματικά” Γιώργος Τασσόπουλος Ευκλείδης Β΄ Τεύχος 89 [Σελίδα 32] “Περί εξισώσεωνΠερί ανισώσεων” Αντώνης Κυριακόπουλος http://www.mathematica.gr/forum/ /viewtopicphp?f=60&t=24251 http://www.mathematica.gr/forum/ /viewtopicphp?f=60&t=31430

ΕΥΚΛΕΙΔΗΣ Β΄ 90 τ.2/31

Α΄ΛΥΚΕΙΟΥ

Παραλληλόγραμμα - Τραπέζια Ροδούλα Κιούφτη

ΠΑΡΑΛΛΗΛΟΓΡΑΜΜΑ-ΤΡΑΠΕΖΙΑ (ΟΡΙΣΜΟΣ-Ι∆ΙΟΤΗΤΕΣ-ΚΡΙΤΗΡΙΑ) ΠΑΡΑΛΛΗΛΟΓΡΑΜΜΟ

ΟΡΘΟΓΩΝΙΟ

ΡΟΜΒΟΣ

ΤΕΤΡΑΓΩΝΟ

ΤΡΑΠΕΖΙΟ ΙΣΟΣΚΕΛΕΣ ΤΡΑΠΕΖΙΟ

Σ Χ Η Μ Α Ο Ρ Ι Σ Μ Ο Σ ΠΛΕΥΡΕΣ

Οι απέναντι είναι ίσες και παράλληλες

ΓΩΝΙΕΣ

Οι απέναντι είναι ίσες, οι διαδοχικές παραπληρωματικές

ΔΙΑΓΩΝΙΟΙ

Ι Δ Ι Ο Τ Η Τ Ε Σ

Το τετράπλευρο που έχει τις απέναντι πλευρές του παράλληλες

Διχοτομούνται

Το παραλληλόγραμμο που έχει μία ορθή γωνία

Οι απέναντι είναι ίσες και παράλληλες

Όλες είναι ορθές

Το παραλληλόγραμμο που έχει δύο διαδοχικές πλευρές ίσες

Το παραλληλόγραμμο που είναι ορθογώνιο και ρόμβος

Οι απέναντι είναι παράλληλες και όλες είναι ίσες

Οι απέναντι είναι παράλληλες και όλες είναι ίσες

Οι απέναντι είναι ίσες, οι διαδοχικές παραπληρωματικές

 Το κυρτό τετράπλευρο που έχει δύο μόνο πλευρές του παράλληλες ΙΔΙΟΤΗΤΕΣ  Η διάμεσος του τραπεζίου είναι παράλληλη προς τις βάσεις του και ίση με το ημιαθροισμά τους

Α

Όλες είναι ορθές

Δ

Διχοτομούνται και είναι ίσες

Διχοτομούνται, διχοτομούν τις γωνίες του, τέμνονται κάθετα

Είναι ίσες, διχοτομούνται, διχοτομούν τις γωνίες του και τέμνονται κάθετα

Β

Γ

 Ισοσκελές είναι το τραπέζιο του οποίου οι μη παράλληλες πλευρές είναι ίσες. ΙΔΙΟΤΗΤΕΣ  Οι γωνίες που πρόσκεινται σε μια βάση είναι ίσες.  Οι διαγώνιοί του είναι ίσες.

Κ Ρ Ι Τ Η Ρ Ι Α

1. 2. 3. 4. 5.

ΔΕΙΧΝΩ ΟΤΙ: 1. Οι απέναντι πλευρές είναι παράλληλες 2. Οι απέναντι πλευρές είναι ίσες 3. Δύο απέναντι πλευρές είναι ίσες και παράλληλες 4. Δύο απέναντι γωνίες είναι ίσες 5. Οι διαγώνιοί του διχοτομούνται

ΔΕΙΧΝΩ ΟΤΙ: Είναι παραλληλόγραμμο και ένα από τα παρακάτω: 1.Έχει μία ορθή γωνία 2.Οι διαγώνιοί του είναι ίσες ή χωρίς να δείξω ότι είναι παρ/μο δείχνω: 1.Έχει τρεις ορθές γωνίες 2. Όλες οι γωνίες του είναι ίσες

ΔΕΙΧΝΩ ΟΤΙ: Είναι παραλληλόγραμμο και ένα από τα παρακάτω: 1.Δύο διαδοχικές πλευρές ίσες 2.Οι διαγώνιοί του τέμνονται κάθετα 3.μία διαγώνιός του διχοτομεί μία γωνία του ή χωρίς να δείξω ότι είναι παρ/μο δείχνω ότι όλες οι πλευρές του είναι ίσες

ΔΕΙΧΝΩ ΟΤΙ: Είναι ορθογώνιο και ρόμβος

ΕΡΩΤΗΣΕΙΣ ΣΩΣΤΟΥ-ΛΑΘΟΥΣ Κάθε διαγώνιος παραλληλογράμμου το χωρίζει σε δύο ίσα τρίγωνα. Οι διαγώνιες κάθε παραλληλογράμμου είναι άξονες συμμετρίας τους. Ένας ρόμβος με ίσες διαγώνιους είναι τετράγωνο. Οι διχοτόμοι δύο διαδοχικών γωνιών ενός παραλληλογράμμου τέμνονται κάθετα. Τα μέσα των πλευρών ενός ισοσκελούς τραπεζίου ορίζουν ρόμβο. ΕΥΚΛΕΙΔΗΣ Β΄ 90 τ.2/32

ΔΕΙΧΝΩ ΟΤΙ:  Για τραπέζιο: Ότι έχει δύο πλευρές παράλληλες  Για ισοσκελές: Ότι είναι τραπέζιο και: 1. Οι γωνίες που πρόσκεινται σε μια βάση είναι ίσες. 2. Οι διαγώνιοί του είναι ίσες.

-------------------------------------------------------------------------------------------------------------------- Μαθηματικά για την Α΄ Λυκείου -----------------------------------------------------------------------------------------------------------------

6. 7. 8. 9. 10. 11. 12. 13. 14. 15. 16. 17. 18. 19. 20. 21. 22. 23. 24. 25. 26. 27.

Ένα τετράπλευρο που έχει δύο απέναντι πλευρές του ίσες και παράλληλες είναι παραλληλόγραμμο. Οποιοδήποτε τετράπλευρο είναι τετράγωνο αν έχει μία γωνία ορθή και μία διαγώνιος διχοτομεί μια γωνία του. Ρόμβος λέγεται το τετράπλευρο που έχει όλες τις πλευρές ίσες μεταξύ τους. Οποιοδήποτε τετράπλευρο του οποίου οι διαγώνιες διχοτομούνται και είναι κάθετες είναι τετράγωνο. Αν ένας ρόμβος έχει ίσες διαγώνιους, τότε είναι τετράγωνο. Οποιοδήποτε τετράπλευρο έχει κάθετες διαγώνιες είναι ρόμβος. Οι διαγώνιοι του ισοσκελούς τραπέζιου είναι ίσοι. Ένα τετράπλευρο που έχει 3 γωνίες ορθές είναι ορθογώνιο. Όλες οι γωνίες κάθε ρόμβου είναι ίσες. Ένας ρόμβος με μια ορθή γωνία είναι τετράγωνο. Κάθε τετράγωνο είναι ρόμβος. Οι διαγώνιοι κάθε παραλληλογράμμου είναι ίσες. Τα μέσα των μη παραλλήλων πλευρών και των διαγωνίων τραπεζίου είναι συνευθειακά σημεία. Η διάμεσος κάθε ορθογωνίου τριγώνου που αντιστοιχεί στην υποτείνουσα ισούται με το διπλάσιο της υποτείνουσάς του. Σε κάθε ορθογώνιο τρίγωνο η διάμεσος προς την υποτείνουσα το χωρίζει σε δύο ισοσκελή τρίγωνα. Οι διαγώνιοι του τετραγώνου είναι κάθετες. Οι διαγώνιοι κάθε παραλληλογράμμου το χωρίζουν σε τέσσερα ίσα τρίγωνα. Το παραλληλόγραμμο που έχει δύο διαδοχικές γωνίες ίσες είναι ορθογώνιο. Κάθε τραπέζιο που έχει δύο απέναντι γωνίες παραπληρωματικές είναι ισοσκελές. Αν σε ορθογώνιο τρίγωνο ΑΒΓ(= 90°) είναι ΒΓ = 2ΑΒ τότε η γωνία Β είναι 30°. Αν οι διαγώνιες ενός ορθογωνίου είναι κάθετες, τότε είναι τετράγωνο. Σε κάθε τραπέζιο η ευθεία που ενώνει τα μέσα των δύο βάσεων είναι κάθετη σ’ αυτές.

ΑΣΚΗΣΕΙΣ Άσκηση 1η Επί ευθείας (ε). θεωρούμε τα διαδοχικά ίσα τμήματα: ΑΒ, ΒΓ, ΓΘ, ΘΔ. Κατασκευάζουμε, προς το ίδιο μέρος της (ε), τα ισόπλευρα τρίγωνα ΔΕΓ και ΓΖΒ και φέρνουμε το ύψος ΒΗ του τριγώνου ΓΖΒ και τη διάμεσο ΒΜ του τριγώνου ΒΖΑ. Να δειχθεί ότι: i Ε, Ζ, Α συνευθειακά. ii ΔΕΖΓ τραπέζιο. iii ΖΜΒΗ ορθογώνιο. iv



 ισόπλευρο. 

Αν ZK διχοτόμος του τριγώνου  και η προέκτασή της τέμνει την ΔΕ στο Λ, τότε ΔΛΖΒ ισοσκελές τραπέζιο. Απόδειξη i. Συγκρίνουμε τα τρίγωνα ΕΓΖ, ΕΘΓ. Έχουμε:        2               90 (1). ΕΓ κοινή, 1   2  60 ,   2 2      90 (2). Στο τρίγωνο ΓΖΑ έχουμε: ΒΖ διάμεσος και   2 v





i.

Άρα: (1), (2)     180  Ε, Ζ, Α συνευθειακά.

ii.

1   3  60  ΔΕ//ΓΖ  ΔΕΖΓ τραπέζιο, αφού οι ΔΓ, ΕΖ τέμνονται.













iii. ΒΖ=ΒΓ=ΑΒ   ισοσκελές και ΒΜ διάμεσος  ΒΜ ύψος  1  90  1    ΕΥΚΛΕΙΔΗΣ Β΄ 90 τ.2/33

-------------------------------------------------------------------------------------------------------------------- Μαθηματικά για την Α΄ Λυκείου -----------------------------------------------------------------------------------------------------------------

ΖΜΒΗ ορθογώνιο. 



iv. Από το ορθογώνιο τρίγωνο  έχουμε   90  60  30 . Εξάλλου ΘΖ διάμεσος του ορθογω          ισόπλευρο. νίου τριγώνου  και   30 . Άρα   2 



 ισόπλευρο, ΖΚ διχοτόμος  ΖΚ διάμεσος. Τα Κ, Ζ μέσα πλευρών στο   ΖΚ//ΑΘ  ˆ (γιατί;) Άρα ΔΛΖΒ τραπέζιο και      , οπότε ΖΛ//ΔΒ, ενώ ΔΕ, ΒΖ τέμνονται αφού ˆ 1   2 είναι ισοσκελές. Άσκηση 2η Δίνεται τρίγωνο ΑΒΓ με  και στην προέκταση της διχοτόμου ΑΔ θεωρούμε σημείο Κ τέτοιο ώστε ΔΚ=ΑΔ. Αν Ε σημείο της ΒΓ τέτοιο ώστε ΔΕ=ΔΒ, να δειχθεί ότι: i. Αν Λ το σημείο τομής των ΚΕ, ΑΓ και Μ το σημείο τομής των ΛΔ, ΑΒ τότε    . ii. ΑΛΚΜ ρόμβος. iii. Από τυχαία σημεία Π, Ρ (διαφορετικά των Α, Δ, Κ) των ΑΔ, ΔΚ φέρουμε κάθετες στην ΑΚ. Αν Η, Ν, Ο, Ξ τα σημεία τομής τους με τις πλευρές του ρόμβου, και  / /  τότε να δειχθεί ότι τα Ν, Η, Ο, Ξ είναι κορυφές ισοσκελούς τραπεζιού. Απόδειξη i. Στο τετράπλευρο ΑΕΚΒ οι διαγώνιοι διχοτομούνται. Άρα είναι παραλληλόγραμμο, οπότε: ΑΒ=ΚΕ. ˆ K ˆ A ˆ . Άρα A ˆ K ˆ και ΑΔ διχοτόμος  A ˆ και αφού Δ Αλλά ΑΕΚΒ παραλληλόγραμμο  A 1 1 1 2 2 1 μέσο του ΑΚ, θα είναι    . ii. Στο ΑΜΛ τρίγωνο η ΑΔ είναι ύψος και διχοτόμος. Άρα: ΔΜ=ΔΛ. Όμως ΑΔ=ΔΚ και   M , οπότε ΑΛΚΜ ρόμβος, αφού οι διαγώνιες διχοτομούνται κάθετα. v.









iii. Προφανώς ΝΗ//ΞΟ ως κάθετες στην ΑΚ. Εξάλλου 1  2, 1   2 . Άρα ΑΝ=ΑΗ και ΑΜ=ΑΛ, οπότε ΝΜ=ΗΛ. Επίσης ΚΞ=ΚΟ και ΚΜ=ΚΛ οπότε ΞΜ=ΟΛ. Συγκρίνουμε τώρα τα τρίγωνα ΝΜΞ, 







ΗΛΟ. Έχουν: ΝΜ=ΗΛ, ΜΞ=ΛΟ και    , οπότε:     ΝΞ=ΗΟ  ΝΗΟΞ ισοσκελές τραπέζιο. Προφανώς αν ΗΟ//ΑΚ τότε το τραπέζιο μεταπίπτει σε ορθογώνιο παραλληλόγραμμο. Άσκηση 3η Δίνεται ορθογώνιο ΑΒΓΔ με  . Από το σημείο Γ φέρουμε ευθεία (ε) κάθετη στην ΑΓ και έστω Ε, Ζ τα σημεία τομής της με τις ΑΒ, ΑΔ αντίστοιχα. Αν ΔΚ, ΒΛ τα ύψη των τριγώνων ΔΖΓ, ΒΓΕ αντίστοιχα, να δειχθεί ότι: 



i. Τα τρίγωνα , ΓΔΚ έχουν ίσες γωνίες. ii. Αν στην ΑΓ θεωρήσουμε τυχαίο σημείο Η και στην προέκτασή της πάρουμε σημείο Θ ώστε ΓΗ=ΓΘ, τότε ΚΗΛΘ ρόμβος. iii. ΒΔ=ΒΛ+ΔΚ. iv. Αν   2 και Ρ σημείο της ΒΛ ώστε ΒΛ=4ΡΛ τότε ΔΡΛΚ ορθογώνιο. v. Αν Σ το μέσο της ΒΕ τότε ΓΡ//ΛΣ. ΕΥΚΛΕΙΔΗΣ Β΄ 90 τ.2/34

-------------------------------------------------------------------------------------------------------------------- Μαθηματικά για την Α΄ Λυκείου -----------------------------------------------------------------------------------------------------------------

Απόδειξη 



ˆ .     90 Είναι ορθογώνια και ˆ 3  90  ˆ 1  ˆ 2   2 ˆ ˆ . οπότε και    i

1

ii Έχουμε: ΓΗ=ΓΘ και    . Αρκεί λοιπόν να δείξουμε ότι ΚΓ=ΚΛ. Φέρνουμε τη διαγώνιο ΔΒ του ΑΒΓΔ. Οι ΔΚ, ΑΓ, ΒΛ παράλληλες και χωρίζουν την ΔΒ σε δύο ίσα μέρη επομένως θα χωρίζουν και την ΚΛ σε δύο ίσα μέρη, δηλαδή ΚΓ=ΚΛ.     2                 . iii ΔΒΛΚ τραπέζιο    2 iv Έστω Μ, Ν τα μέσα των ΒΛ, ΓΛ αντίστοιχα. Συγκρίνουμε τα ορθογώνια τρίγωνα ΜΛΝ, ΚΔΓ. Έχουν:            και  2  2   2  3      ΚΔ=ΝΛ και ΚΓ=ΜΛ. Αλλά 2              . Επίσης ΚΔ//ΛΡ ως κάθετες στη ΖΕ οπότε ΔΡΛΚ ορθογώνιο. 2 2 4 (Σημείωση: Με χρήση ομοιότητας τριγώνων το συμπέρασμα θα ήταν πιο άμεσο: Από το i ερώτημα τα      1       2  2  4     . ) , ΓΔΚ είναι όμοια. Άρα    2 ˆ  ˆ  ˆ  ˆ (γιατί;), οπότε  / /  v Έχουμε  1 3 4 η Άσκηση 4 Έστω ΑΒΓ τυχαίο τρίγωνο. Στη ΒΓ θεωρούμε τα Δ, Ε ώστε ΒΔ=ΔΕ=ΔΓ. Αν ΓΜ διάμεσος του τριγώνου, Η το σημείο τομής της ΑΕ με την ΓΜ, Ζ το σημείο τομής της ΜΔ με την ΒΗ και προεκτείνουμε την ΜΔ κατά ΔΘ=2ΔΖ, να δειχθεί ότι: i.ΑΗΖΜ τραπέζιο. ii.ΒΖ=ΖΗ και ΜΗ=ΓΗ. iii.ΑΗ=3ΗΕ. iv.ΜΗΘΒ παραλληλόγραμμο. v.ΑΗΘΜ παραλληλόγραμμο. vi.ΖΗΓΘ τραπέζιο. Απόδειξη i. Στο τρίγωνο ΑΕΒ τα Μ, Δ είναι μέσα των πλευρών ΑΒ, ΒΕ αντίστοιχα. Άρα ΜΔ//ΑΕ,και οι ΑΜ, ΖΗ τέμνονται, οπότε ΑΗΖΜ τραπέζιο. ii. Στο τρίγωνο ΑΒΗ έχουμε ΜΖ//ΑΗ και Μ μέσο του ΑΒ. Άρα ΒΖ=ΖΗ. Όμοια στο ΜΓΔ έχουμε ΗΕ//ΜΔ και Ε μέσο του ΓΔ. Άρα ΜΗ=ΗΓ. iii. Αρκεί να δείξουμε ότι ΑΕ=4ΗΕ. Πράγματι: από το τρίγωνο ΑΒΕ, αφού Μ, Δ μέσα πλευρών έχουμε ΑΕ=2ΜΔ . Στο τρίγωνο ΓΜΔ, αφού Η, Ε μέσα πλευρών επίσης θα έχουμε ΜΔ=2ΗΕ. Τελικά ΑΕ=4ΗΕ. iv. Αρκεί να δείξουμε ότι ΜΖ=ΖΘ, δηλαδή ΜΖ=3ΖΔ, ή ΜΔ=4ΖΔ. Πράγματι ΜΔ=2ΗΕ=4ΖΔ (Μέσα πλευρών των τριγώνων ΔΓΜ, ΒΗΕ). v. Από ερωτήματα (i), (iv) έχουμε ΜΘ//ΑΗ και    2  2   . Άρα ΑΗΘΜ παραλληλόγραμμο. 2 vi. Στο τρίγωνο ΜΓΘ τα Ζ, Η μέσα των πλευρών ΜΘ, ΜΓ αντίστοιχα. Άρα ΖΗ//ΘΓ και οι ΖΘ, ΗΓ τέμνονται, οπότε ΖΗΓΘ τραπέζιο. Άσκηση 5η Θεωρούμε το τρίγωνο ΑΒΓ με ΒΓ=2ΑΒ και τη διάμεσο ΑΜ αυτού. Αν Ε, Ζ τα μέσα των ΒΜ, ΑΒ αντίστοιχα, να δειχθεί ότι: i. ΑΜΕΖ ισοσκελές τραπέζιο. 

ii. ΑΜ διχοτόμος της γωνίας  . iii. Αν προεκτείνουμε τις ΜΖ, ΑΕ κατά ΖΛ=ΕΚ=ΟΕ, τότε ΑΜΚΛ ορθογώνιο. iv. Αν η ΒΟ τέμνει την ΑΓ στο Ν, τότε ΛΑΝΒ ισοσκελές τραπέζιο. Απόδειξη ΕΥΚΛΕΙΔΗΣ Β΄ 90 τ.2/35

-------------------------------------------------------------------------------------------------------------------- Μαθηματικά για την Α΄ Λυκείου -----------------------------------------------------------------------------------------------------------------

i.

Στο τρίγωνο ΑΒΜ τα Ζ, Ε μέσα των ΑΒ, ΒΜ αντίστοιχα. Άρα ΖΕ//ΑΕ και αφού οι ΑΖ, ΕΜ τέμνο     . Τελικά το ΑΜΕΖ είναι ισοσκελές νται το ΑΜΕΖ είναι τραπέζιο. Επίσης έχουμε:   2 4 τραπέζιο. Εξάλλου στο ισοσκελές τρίγωνο ΑΒΓ η ΒΟ είναι η τρίτη διάμεσος, δηλαδή κάθετη στην ΑΜ. ii.









ΑΜΕΖ ισοσκελές τραπέζιο  1  1 . Αλλά 1   2 , ως εντός εναλλάξ αφού ΜΖ//ΑΓ, (ενώνει 





τα μέσα δύο πλευρών του  ), οπότε 1   2 . iii. ΑΟ =ΜΟ και ΟΕ=ΟΖ, επειδή ΑΜΕΖ ισοσκελές τραπέζιο. ΑΕ, ΜΖ διάμεσοι του τριγώνου ΑΒΜ. Άρα ΑΟ=2ΟΕ=ΟΚ και ΜΟ=2ΟΖ=ΟΛ. Τελικά οι διαγώνιοι του ΑΜΚΛ είναι ίσες και διχοτομούνται, οπότε είναι ορθογώνιο. iv. ΑΜΚΛ ορθογώνιο     . Προφανώς ΑΛ//ΒΝ ως κάθετες στην ΑΜ . Εξάλλου    



και 1   2  ΑΝ=ΑΟ. Αλλά ΒΛΑΟ είναι παραλληλόγραμμο (οι διαγώνιοι διχοτομούνται), οπότε ΑΟ=ΒΛ. Άρα και ΑΝ=ΒΛ. Δηλαδή το τραπέζιο ΛΑΝΒ είναι ισοσκελές. Άσκηση 6η 

Δίνεται ρόμβος ΑΒΓΔ κέντρου Ο, με   30 και ΑΕΒ ισόπλευρο τρίγωνο εκτός αυτού. Φέρνουμε τις ΔΕ, ΓΕ και έστω Ζ, Κ το σημείο τομής της ΓΕ με την ΑΒ, ΔΒ αντίστοιχα. Να δειχθεί ότι: ˆ . i. ΕΒ διχοτόμος της   ii. Αν Η το σημείο τομής της ΑΚ με την ΔΕ, τότε    και  2 iii. Γ, Β, Η είναι συνευθειακά. iv. ΑΗΒΔ τραπέζιο.   v. ΑΕΘΛ ισοσκελές τραπέζιο. vi.   . vii.   2 2 Απόδειξη i.









Προφανώς   30  60  90 και ΑΔ=ΑΕ  1  45   2  15 . Εξάλλου  ισοσκελές 













με   360     360  150  60  150  3  3  15 . Άρα  2  3 . ii.





Στο ρόμβο ΑΒΓΔ έχουμε 1   2  15 . Εξάλλου τα σημεία Α, Γ είναι συμμετρικά ως προς τη ΔΚ, 





οπότε 3  3  15 . Άρα   3  15  45 . Στο ορθογώνιο και ισοσκελές τρίγωνο ΔΑΕ η ΑΗ διχο . τόμος , οπότε και μεσοκάθετος του ΔΕ. Άρα   2 ˆ    ˆ , οπότε και μεσοκάθετος ˆ ( 150 ) η προέκταση της ΓΒ είναι διχοτόμος της ABE iii. Αφού AB του ΑΕ. Αλλά ΗΑ=ΗΕ οπότε το Η ανήκει στη ΓΒ δηλαδή Γ,Β,Η συνευθειακά. iv. Δείξαμε ότι    , δηλαδή ΒΗ//ΑΔ, οπότε: ΑΗΒΔ τραπέζιο. v.





Τα ορθογώνια τρίγωνα ΗΑΛ, ΗΕΘ είναι ίσα αφού ΗΑ=ΗΕ και 3  E 2  15 . Άρα ΗΛ=ΗΘ και 



ΑΛ=ΕΘ. Το τρίγωνο ΗΛΘ είναι ορθογώνιο και ισοσκελές οπότε 1  45  4 . Άρα ΛΘ//ΑΕ, δηλαδή ΑΕΘΛ τραπέζιο και αφού ΑΛ=ΕΘ θα είναι ισοσκελές. vi. Αρκεί να δείξουμε ότι ΓΖ=ΑΗ. Συγκρίνουμε τα τρίγωνα ΑΓΖ, ΑΓΗ. Έχουν: ΑΓ κοινή και οι προσκείμενες γωνίες στην ΑΓ είναι ίσες αφού σε καθένα από αυτά τα τρίγωνα η μία γωνία είναι 30ο και η άλλη 15ο Άρα τα τρίγωνα είναι ίσα, οπότε ΓΖ=ΑΗ. Παρατήρηση: Στα ίσα τρίγωνα ΑΓΗ, ΑΓΖ τα αντίστοιχα ύψη ΗΗ΄, ΖΖ΄ θα είναι ίσα και παράλληλα, οπότε ΗΖ//ΑΓ, δηλαδή ΑΓΖΗ ισοσκελές τραπέζιο. Αν μάλιστα θέσουμε ΑΗ=ΓΖ=λ, τότε ΄  ΄  

   , αφού     30 . 2



vii. Στο ορθογώνιο τρίγωνο ΟΓΚ έχουμε     30 , ως συμμετρικές προς την ΔΚ. Άρα    . 2 ΕΥΚΛΕΙΔΗΣ Β΄ 90 τ.2/36

---------------------------------------------------------------------------------------------------------------------- Μαθηματικά για τη Β΄ Λυκείου ------------------------------------------------------------------------------------------------------------------

B΄ΛΥΚΕΙΟΥ

Τριγωνομετρία – Πολυώνυμα Αργύρη Παναγώτα

ο

ΤΡΙΓΩΝΟΜΕΤΡΙΑ

ΘΕΜΑ 1 Δίνεται ορθογώνιο παραλληλόγραμμο με διαγώνιο ΑΓ= 2 .

Α. Να αποδείξετε οτι η περίμετρος του ορθογωνίου είναι : Π(θ)= 2 2(    ) Β. Αν ισχύει Π(θ)= 2 1   , να υπολογίσετε  την γωνία θ ( 0    ) 2 Λύση A. Έστω ΑΒ=x και ΒΓ=ψ τότε: x    x  2   2       2   2 Π(θ)= 2  x  2   = 2 2(  ) Β. Επίλυση τριγωνομετρικής εξίσωσης: 2 2(  ) = 2 1  2

 2 2(  )   (2 1   )2   

 4  2    2   2   4(1  )   4 1  2  4  4   8  4  0  4  2  1  0   0    00 ,  ί ή 1  2  1  0       2 6 ΘΕΜΑ 2ο 1 x Δίνεται η συνάρτηση f(x)=  x  x Α. Να αποδείξετε άτι f(x)= ημx, x  κπ .  17   Β. Να υπολογίσετε την τιμή f   .  4    Γ. Να λύσετε την εξίσωση : f(x)=   x   3  Λύση 1 x 1 x Α. f (x)      x x x x x

1   2 x  2 x   x x x 17   17    16    Β. f ( )           4 4   4   

        2       4       4      ( )   4  4  4  2        Γ. f (x)    x    x    x    3 3         ( x)    x    ( x)     x  3 6     5  (...)  x    ,  12 ΘΕΜΑ 3ο x Δίνεται η συνάρτηση f (x)  3    1 .  3 Α. Να υπολογίσετε την περίοδο Τ της f. B. Να παρουσιάσετε τον μονοτονία, την μέγιστη και της ελάχιστη τιμή της συνάρτησης σε πίνακα για διάστημα μίας περιόδου Τ, δηλαδή στο διάστημα [0,Τ]. Λύση 2 x Η περίοδος της f (x)     είναι    6 1 3 3 Τα διαστήματα που θα μελετηθεί η μονοτονία και τα ακρότατα της είναι τα εξής:          3   3  0, 4  ,  4 , 2  ,  2 , 4  ,  4 ,  . Τελικά βρίσκουμε:            3   3   9   9  f  o,  , f   ,3 ,f  3,   f   ,6  2  2 2   2  . ο

ΠΟΛΥΩΝΥΜΑ

ΘΕΜΑ 1 A. Δίνεται το πολυώνυμο P(x)=(x–1)2013–x20125x+3. α υπολογίσετε τον σταθερό του όρο και το άθροισμα των συντελεστών του. B. i) Αν ρ η ρίζα του πολυωνύμου q(x)=P(x)–x, να αποδείξετε ότι και το πολυώνυμο Q(x)=P(P(x))–x έχει ρίζα τον αριθμό ρ. ii) Να εξετάσετε αν ισχύει το αντίστροφο. Λύση A.To πολυώνυμο γράφεται στη μορφή: P(x)   v  x v   v 1  x v 1  ....  1  x   0 .

Είναι : 0 : v  0v  v1  0v1  ....  1  0  0  P(0)

ΕΥΚΛΕΙΔΗΣ Β΄ 90 τ.2/37

---------------------------------------------------------------------------------------------------------------------- Μαθηματικά για τη Β΄ Λυκείου ------------------------------------------------------------------------------------------------------------------

και  v  v 1  ...  1   0 :

(1)  P( )  (  1)3    (  1) 2  b  (  1) για

  v  1v   v 1  1v 1  ....  1  1   0  P(1) Άρα ο σταθερός όρος είναι το: P(0)  (1) 2013  02012  5  0  3  2 και άθροισμα των συνετελεστών του είναι το: P(1)  (0) 2013  12012  5  1  3  3 Β. i) Ο οριθμός ρ είναι ρίζα του q(x)=P(x)–x  q(ρ)=0  Ρ(ρ)=ρ Είναι: Q()  P(P())  P()  0  ρ ρίζα του Q(x). ii) Ζητείται να εξεταστεί αν ισχύει η συνεπαγωγή:Q(ρ)=0  q(ρ)=0 Ας αναζητήσουμε αρχικά πολυώνυμο Ρ(x) =αx+β για το οποίο να μην ισχύει η προηγούμενη συνεπαγωγή. Αρκεί για αυτό να είναι π.χ: (1)  2  1  1    2         2    , ή   ,ή  . ((1))  1  (2)  1  2    1       1 Τελικά:   . Άρα P(x)=–x+3 (μπορούν να βρε  3  θούν και άλλα πολυώνυμα με μεγαλύτερο βαθμό). ΘΕΜΑ 2ο Δίνεται το πολυώνυμο P(t) τέτοιο ώστε:

κάθε   R  P(x)  (x  1)3   (x 1)2  b  (x  1) για κάθε x R. Οπότε P(2)=1–α–b και P(0)=–1+α+b κ.τ.λ Β. Το πρόσημο του πολυωνύμου P(x):

P(x  1)  x 3    x 2  b  x , για κάθε x  R (1) Α. Να βρείτε τους συντελεστές α, b,ώστε το υπόλοιπο της διαίρεσης του πολυωνύμου P(x) με το x–2 να είναι 6 και η γραφική παράσταση της συνάρτησης ψ=P(x)να διέρχεται από την αρχή των αξόνων. Β. Για α=3 και b=–2 να λύσετε την ανίσωση:





P x2  2 3  x  3  0 Λύση Α. Πρέπει και αρκεί : P(2)=6 , P(0)=0 Α΄ τρόπος Για x=1 η (1) δίνει :Ρ(2)=–1+α+b και για x= –1 η (1) δίνει Ρ(0)=–1+α+b ,οπότε:  α=3 P(2)=6  1 b  6   b  5          b=-2  P(0)  0 1 b  0  b 1     3 2 2 Άρα : P(x  1)  x  3x  2x  x(x  3x  2)   x(x  1)(x  2) για κάθε x R (2). Αφού για κάθε x,  R ισχύει : x+1=ψ  x    1 θα έχουμε: (2)  P( )  (  1)    (  1) για κάθε   R  P(x)  (x  1)  x  (x  1) για κάθε

x  R  P(x)  x 3  x για κάθε x  R B΄ τρόπος Αφού για κάθε x,  R ισχύει  x    1 θα έχουμε:

Άρα Ρ(x)<0 x(x+1)(x–1)<0  x<–1 ή 0


 

2  2  x2  12  x  3  x  2 3  x  3       ()  0  1ή0 1  2  x 3   2      x  3  1 , αδύνατη   1ή0    1 x  3  x  3  2    ή 0  x  3 1   2     x  3 1 1 x  3 1 x  3     x  3 1, 3  3, 3 1  3 1  x  3  1 Θέμα 3ο Δίνεται το πολυώνυμο Ρ(χ): P(x)= x 3 -(5ημθ)  x 2 -2  x+2συν 2θ (1),   R Α. Να βρείτε τις τιμές που παίρνει η γωνία θ  R, ώστε το υπόλοιπο της διαίρεσης του πολυωνύμου Ρ(x) με το πολυώνυμο (x–1) είναι ίσο  με –2. Β. Αν επιπλέον ισχύει ότι     , τότε 2 να υπολογίσετε και τους άλλους τριγωνομετρικούς αριθμούς της γωνίας θ. Λύση A. Πρέπει και αρκεί P(1)=–2 (1). Έχουμε: (1) 13  (5) 12  2 1 2 2  2 









x+1=ψ











 



 2  1   2    5  1  0 

ημθ=ω

  2    5    3  0

22  5  3  0  

 ημθ=ω     1   ημθ=–3 3, ή       2 

2

(αδύνατη)

1  5    2   ή   2   ,    6 2 6 2 3 1 B. 22 1  2 1   2  2  

3  διότι συνθ<0 για     2 2 1 3       ,    3. 3   3

   

:



Επομένως η ανίσωση P x 2  12  x  3  0 

ΕΥΚΛΕΙΔΗΣ Β΄ 90 τ.2/38

ή

---------------------------------------------------------------------------------------------------------------------- Μαθηματικά για τη Β΄ Λυκείου ------------------------------------------------------------------------------------------------------------------

Εύρεση του υπολοίπου της διαίρεσης του πολυωνύμου Δ(x) με το διώνυμο δ  x   xκ  ρ χωρίς εκτέλεση διαίρεσης. Ευάγγελος Οικονομίδης

Έχουμε    και έστω     x  . Α. Αν κ=1 τότε το υπόλοιπο είναι υ=Δ(ρ) Β. Αν κ>1 τότε στο διαιρετέο Δ(x) θέτουμε όπου xκ το ρ και το πολυώνυμο που βρίσκουμε είναι το υπόλοιπο υ(x). Προηγουμένως στο Δ(x) κάθε μονώνυμο της μορφής βxλ, με λ κ το γράφουμε με τη μορφή   x      x    x  όπου κ, ν το πη

λίκο και το υπόλοιπο της διαίρεσης λ:κ αντιστοίχως. Προφανώς   0,1, 2,...,    1 . Παραδείγματα: 1) Αν  x 4x6 7x5 3x4 2x2 6x 5 και δ(x)=x4-3,

τότε έχουμε:  x 4 x4  x2 7 x4  x 3x4 2x2 6x 5, θέτουμε όπου x4 το 3 και βρίσκουμε   x   4  3  x 2  7  3  x  3  3  2x 2  6x  5   10x 2  15x  4 . Το αποτέλεσμα επαληθεύεται με εκτέλεση της διαίρεσης. 2) Αν  x  7x6  6x4 8x 10 και  x  x3  5, τότε  x 7 x3  6x3  x 8x 10 θέτουμε όπου x 3   5 2

το ίδιο υπόλοιπο και πηλίκα  x  ,   x  αντιστοίχως Παρατήρηση 2η : Το προηγούμενο υπόλοιπο θα μπορούσε να βρεθεί (με πιο επίπονη φυσικά διαδικασία) ως εξής: Έχουμε την ταυτότητα  3  3 x   x    x       x   x   , οπότε 2  2   

  3 3        2 2  απ’ όπου προκύπτουν τα α, β.  3 3      2 2    Αιτιολόγηση: Η αυστηρή αιτιολόγηση των παραπάνω συμπερασμάτων γίνεται ως εξής: Αν θεωρήσουμε δυο πολυώνυμα Α(x), Β(x)και ένα τρίτο μη μηδενικό πολυώνυμο δ(x), τότε τα Α(x), Β(x) θα λέγονται ισουπόλοιπα ως προς δ(x), όταν και μόνο οι διαιρέσεις Α(x):δ(x) και Β(x):δ(x) αφήνουν ίσα υπόλοιπα. Συμβολικά γράφουμε τότε:   x   B  x    x   ή απλώς   x   B  x  , όταν είναι σαφώς ορισμένο το δ(x).

και βρίσκουμε  x 7 5 6 5 x 8x 10 38x 185. Το αποτέλεσμα επαληθεύεται με εκτέλεση της διαίρεσης. 3) [εφαρμογή Άσκησης 9(β) Ευκλείδης Β΄ 74 τ.2/39] Αν   x   x 2010  x 2009  x 2008  1 και  x  x2 1,

Προφανώς ισοδυναμεί με Α(x)Β(x)=πολ δ(x). Εύκολα αποδεικνύεται ότι: Αν   x   B  x  και 1  x  B1  x

τότε  x  x2 

, 1   και    * . Έτσι σε κάθε πολυώνυμο f(x) μπορούμε να αντικαταστήσουμε κάποιο από τα πολυώνυμα που το απαρτίζουν (ως προσθετέοι ή ως παράγοντες) με άλλα ισουπόλοιπα αυτών, στοχεύοντας να υποβιβάσουμε το βαθμό, κάτω από το βαθμό του διαιρέτη. Εφαρμογή: Για να βρούμε το υπόλοιπο της διαίρεσης με το του πολυωνύμου f  x   x 5  2x 3  x  2

2

1005

 x2 

1004

 x  x2 

1004

1. Θέτουμε όπου

x2 το 1 και βρίσκουμε  x   1 1 x 1 1  x 1 . Αν δ(x)=αxκ+β με α0 εργαζόμαστε ομοίως  θέτοντας στο Δ(x) όπου xκ το   3 Παράδειγμα: Αν   x   5x  6x 2  4x  2 και 

 x  2x3  3 , τότε   x   5x2 x  6x2  4x  2 . Θέ3 και βρίσκουμε 2 15 23  x   x  9  4x  2  x 11 . Το αποτέλεσμα 2 2 επαληθεύεται με εκτέλεση της διαίρεσης. Παρατήρηση 1η : Από την ταυτότητα της διαίρεσης   x    x   x    x  παίρνουμε τουμε

όπου

x 2 

    x    x      x    x  , δηλαδή οι διαιρέ    σεις   x  :  x  και   x  :  x    αφήνουν  

τότε

    x   1  1  x     B  x   1  B1  x  ,

  x   1  x   B  x   B1  x  ,

   x   B  x  , όπου

  x   x 2  2x  1 , θα ακολουθήσουμε τη διαδικασία

που υποδείξαμε με τη συμφωνία τις σχέσεις (~) και (=) να τις γράφουμε για συντομία συνεχόμενες όπου ισχύουν. Έχουμε λοιπόν:   x   x 2  1  2x     x  , x 2  1  2x    x  

οπότε

και

f  x   x 1  2x   2x 1  2x   x  2  2

4x 3  8x 2  4x  2  4x 1  2x   8 1  2x   4x   8x 2  24x  10  8 1  2x   24x  10  40x  18

Το υπόλοιπο της διαίρεσης λοιπόν είναι   x   40x  18 . Μπορείτε εύκολα να επαληθεύσετε το αποτέλεσμα με εκτέλεση της διαίρεσης. Πλήρη αντιμετώπιση του θέματος μπορείτε να βρείτε στο βιβλίο του Θ.Ν. Καζαντζή, Πολυώνυμα, έκδοση 1977. Γιώργος Σ. Τασσόπουλος

ΕΥΚΛΕΙΔΗΣ Β΄ 90 τ.2/39

---------------------------------------------------------------------------------------------------------------------- Μαθηματικά για τη Β΄ Λυκείου ------------------------------------------------------------------------------------------------------------------

Θεώρημα ρητών ριζών Καμπούκος Κυριάκος [20 Πρότυπο Πειραματικό Γενικό Λύκειο Αθηνών] Υπενθυμίζουμε ότι ένας μη μηδενικός ακέραιος λέμε ότι διαιρεί τον ακέραιο αν και μόνο αν υπάρχει ακέραιος τέτοιος ώστε . Επίσης δύο ακέραιοι , λέγονται πρώτοι μεταξύ τους αν και μόνο αν ο μέγιστος κοινός διαιρέτης τους είναι ίσος με το 1. Ισχύει η εξής πρόταση: Αν ο ακέραιος και οι , είναι πρώτοι μεταξύ τους, τότε ο διαιρεί τον . διαιρεί το γινόμενο Θεώρημα 1 (Θεώρημα ρητών ριζών) Υποθέτουμε ότι , ,⋯, είναι ακέραιοι αριθμοί και ότι ο ρητός αριθμός με , πρώτους ⋯ και ο q διαιρεί τον

μεταξύ τους, είναι ρίζα της πολυωνυμικής εξίσωσης Όπου 1, 0. Τότε ο p διαιρεί το σταθερό όρο



είναι ρίζα της (1) έχουμε:

Απόδειξη Επειδή ο

0 (1) και ο . 0.

⋯ 0, οπότε ⋯ . Από την τελευταία ισότητα, επειδή οι , , καθώς και οι p, qν είναι πρώτοι μεταξύ τους, έπεται ότι ο διαιρεί το σταθερό όρο . Με ανάλογο επιχείρημα αποδεικνύουμε ότι ο διαιρεί τον . Παρατήρηση Σύμφωνα με το παραπάνω Θεώρημα, οι μόνες ρητές ρίζες της εξίσωσης (1) είναι οι αριθμοί της μορφής , όπου είναι διαιρέτης του σταθερού όρου και ο είναι διαιρέτης του . Πολλαπλασιάζοντας με

παίρνουμε

Εφαρμογή 1 Αν ο μη αρνητικός ακέραιος δεν είναι τετράγωνο ακεραίου, τότε ο √ είναι άρρητος. Απόδειξη Προφανώς ο αριθμός √ είναι ρίζα της πολυωνυμικής εξίσωσης 0 (2) η οποία έχει ακέραιους συντελεστές. Ας υποθέσουμε ότι ο ρητός ,όπου , είναι πρώτοι μεταξύ τους είναι ρίζα της εξίσωσης (2). Τότε 1 ή 1, οπότε ή κατά συνέπεια , το οποίο είναι άτοπο, διότι ο δεν είναι τετράγωνο ακεραίου. Επομένως ο √ είναι άρρητος. Εφαρμογή 2 Ο αριθμός √5 είναι άρρητος. 5 0 (3) Απόδειξη Ο αριθμός √5 είναι ρίζα της πολυωνυμικής εξίσωσης η οποία έχει ακέραιους συντελεστές. Σύμφωνα με το Θεώρημα (1) οι ρητές ρίζες της εξίσωσης (3) είναι οι αριθμοί 1, 5. Όμως επειδή κανένας από αυτούς δεν είναι ρίζα της (3) συμπεραίνουμε ότι ο √5 είναι άρρητος. Εφαρμογή 3 Ο αριθμός √3 √2 είναι άρρητος. 5 2√6 , απ’ όπου προκύπτει 5 Απόδειξη Υψώνοντας τον στο τετράγωνο παίρνουμε 24 , το οποίο σημαίνει ότι ο είναι ρίζα της πολυωνυμικής εξίσωσης με ακέραιους συντελεστές 10 1 0 (4) Πιθανές ρίζες της εξίσωσης (4) είναι οι αριθμοί 1. Προφανώς κανένας από αυτούς δεν είναι ρίζα της εξίσωσης (4), το οποίο σημαίνει ότι ο αριθμός √3 √2 είναι άρρητος. Εφαρμογή 4 Ο αριθμός 3 √2 είναι άρρητος. Απόδειξη Υψώνοντας στον κύβο παίρνουμε 3 √2, οπότε 3 2, το οποίο σημαίνει ότι ο 6 7 0 (5) αριθμός είναι ρίζα της πολυωνυμικής εξίσωσης με ακέραιους συντελεστές Πιθανές ρητές ρίζες της (5) είναι οι αριθμοί 1, 7. Με επαλήθευση βρίσκουμε ότι κανένας από τους παραπάνω αριθμούς δεν είναι ρίζα της εξίσωσης (5), κατά συνέπεια ο είναι άρρητος. 5

Εφαρμογή 5 Η εξίσωση √

1

0 (6) έχει μοναδική ρίζα τον αριθμό



. Παρατηρούμε ότι



, ο οποίος είναι άρρητος.

Απόδειξη Θέτουμε Επίσης 10

5

, δηλαδή



1

και 5 10

10 5

10

5

.

ΕΥΚΛΕΙΔΗΣ Β΄ 90 τ.2/40

, , οπότε

1 και 1

5

1.

---------------------------------------------------------------------------------------------------------------------- Μαθηματικά για τη Β΄ Λυκείου ------------------------------------------------------------------------------------------------------------------

Από την ταυτότητα 3 3 , έπεται 3 , δηλαδή 3 . Επομένως 1 10 5 3 , δηλαδή 5 5 1 0. Άρα ο είναι ρίζα της εξίσωσης (6). Εύκολα αποδεικνύεται ότι η συνάρτηση 5 5 1 , ∈ είναι γνησίως αύξουσα, οπότε ο είναι μοναδική ρίζα της εξίσωσης (6). Σύμφωνα με το Θεώρημα 1, οι πιθανές ρητές ρίζες της (6) είναι οι αριθμοί 1. Όμως κανένας από αυτούς δεν είναι ρίζα της (6), το οποίο σημαίνει ότι ο είναι άρρητος. Εφαρμογή 6 Ο αριθμός √2 √3 είναι άρρητος. Απόδειξη Προφανώς έχουμε: α  3  3 2 . Υψώνοντας και τα δύο μέλη της ισότητας √3 √2 , 3 √3 9 3√3 2, οπότε 9 2 3 1 √3. στον κύβο παίρνουμε: 9 2 3 1 √3 παίρνουμε: Αν υψώσουμε στο τετράγωνο και τα δύο μέλη της σχέσης 9 4 27 36 23 0, το οποίο σημαίνει ότι το είναι ρίζα της πολυωνυμικής 9 4 27 36 23 0 (7), που έχει ακέραιους συντελεστές. εξίσωσης Σύμφωνα με το Θεώρημα 1 οι μόνες ρητές ρίζες της εξίσωσης (7) είναι οι αριθμοί 1 και 23. Όμως κανένας από αυτούς δεν είναι ρίζα της (7), γεγονός το οποίο συνεπάγεται ότι ο είναι άρρητος. Συντομότερα: Αν υποθέσουμε ότι ο α είναι ρητός, τότε επειδή το οποίο είναι άτοπο διότι ο √3 είναι άρρητος (Εφαρμογή 1), ενώ ο

1

0, προκύπτει √3

,

είναι ρητός. Επομένως ο

είναι άρρητος. Με ανάλογο τρόπο μπορούμε να εργαστούμε στις εφαρμογές (3), (4). Βιβλιογραφία: 1) Kenneth A. Ross, Elementary Analysis: The Theory of Calculus, Springer–Verlag 2) George Pedrick, A First Course in Analysis, Springer–Verlag. 3) Ανδρεαδάκης Σ., Κατσαργύρης Β., Παπασταυρίδης Σ., Πολύζος Γ., Σβέρκος Α., Άλγεβρα Β΄ Λυκείου, ΙΤΥΕ “Διόφαντος”. 4) Καμπούκος Κ., Τασσόπουλος Γ., Μαθηματική Ανάλυση Ι, Εκδόσεις: Πελεκάνος

Υπολογισμός της τιμής ενός πολυωνύμου P(x) για x=α±  . Διονύσης Γιάνναρος Στον παρόν άρθρο παρουσιάζουμε ένα τρόπο υπολογισμού της τιμής ενός πολυωνύμου για x=α±  , όπου αQ, βQ+*, αλλά

 Q+*. Έστω P(x) = ανxν + αν–1xν–1 +…. + α1x +α0 ένα πολυώνυμο με

ρητούς συντελεστές και φ(x) το πολυώνυμο 2ου βαθμού με ρίζες τους αριθμούς α+  και α–  . Αυτό είναι: Φ(x)= [x – (α+  )][x–( α–  )] = (x–α)2 – β = x2–2αx +α2 – β. Αν το P(x) διαιρεθεί με το φ(x), θα έχουμε την ταυτότητα: P(x)=φ(x)π(x)+(κx+λ) (1). Επειδή ο διαιρέτης είναι β΄ βαθμού, το υπόλοιπο της διαίρεσης θα είναι πολυώνυμο α΄ βαθμού ή μηδενικό. Από τη σχέση (1) για x= α±  έχουμε: Ρ(α±  )= φ(α±  )π(α±  )+ κ(α±  )+λ ή επειδή φ(α±  )=0, Ρ(α±  )= κ(α±  )+λ. Εφαρμογές: 1.

2.

Έστω Ρ(x)=x7–3x6–2x5+2x4–x2 +5x – 4 και x = 2+

2 . Έχουμε: φ(x)=x2–4x+2, π(x)=x5+x4–1 και υ(x)=x–2. Άρα η αριθμητική τιμή του πολυωνύμου για x = 2+ 2 είναι: P(2+ 2 ) = υ(2+ 2 )= 2+ 2 –2 = 2 . x 7  2x 6  2x5  x  1 για x=1– 3 . Να βρεθεί η τιμή του κλάσματος 8 x  2x 7  2x 6  x  1 x 5 ( x 2  2 x  2)  x  1 και επειδή γιαx= 1– 3 το Έχουμε: φ(x) = x2–2x–2. Το κλάσμα γράφεται: 6 2 x ( x  2 x  2)  x  1

x2–2x–2 μηδενίζεται συμπεραίνουμε πως η τιμή του για x=1– 3 θα είναι ίση με ΕΥΚΛΕΙΔΗΣ Β΄ 90 τ.2/41

 3  3

 1.

---------------------------------------------------------------------------------------------------------------------- Μαθηματικά για τη Β΄ Λυκείου ------------------------------------------------------------------------------------------------------------------

B΄ΛΥΚΕΙΟΥ

ΕΜΒΑ∆Α ΕΥΘΥΓΡΑΜΜΩΝ ΣΧΗΜΑΤΩΝ Καρδαμίτσης Σπύρος – Σωτηρόπουλος Νίκος

Στην αρχαιότητα η έννοια του εμβαδού (όχι βέβαια με το σημερινό ορισμό του) αποτελούσε αντικείμενο μελέτης και έρευνας. Η σημερινή έννοια ισεμβαδικά σχήματα στα Στοιχεία του Ευκλείδη σημαίνει ισοδιαχωρισιμα σχήματα και αυτό επιτυγχάνεται πολλές φορές με εντυπωσιακό τρόπο. Πολλοί Έλληνες μαθηματικοί ανέπτυξαν σημαντικές θεωρίες και τεχνικές μεθόδους για το σκοπό αυτό, με αποκορύφωμα τον Αρχιμήδη που έφθασε μέχρι τον τετραγωνισμό παραβολικού χωρίου, δηλαδή στις παρυφές του ολοκληρωτικού Λογισμού. Ερωτήσεις σωστού– λάθους Να χαρακτηρίσετε τις παρακάτω προτάσεις ως σωστές (Σ) ή λάθος (Λ). 1. Αν οι γωνίες Α και Δ δύο οποιονδήποτε τριγώνων ΑΒΓ και ΔΕΖ είναι συμπληρωματικές τότε:

(AB)          

2. Αν ΑΒΓ είναι ισόπλευρο τρίγωνο πλευράς α και ΔΕΖ τυχαίο τρίγωνο με γωνία Ε = 120ο, τότε ισχύει η σχέση

()     ( ) 2

3. Το εμβαδό οποιουδήποτε τετραπλεύρου ΑΒΓΔ με κάθετες διαγώνιους είναι (ΑΒΓΔ) = 4. Όλα τα ισόπλευρα τρίγωνα με πλευρά α έχουν εμβαδό ίσο με  2

1    2

3 16

5. 6. 7. 8. 9.

Δύο οποιαδήποτε τρίγωνα έχουν ίσα εμβαδά αν και μόνο αν είναι ίσα. Δύο οποιαδήποτε ισοδύναμα τρίγωνα είναι και ίσα Υπάρχουν ισοδύναμα τρίγωνα που είναι και ίσα Δύο οποιαδήποτε ίσα τρίγωνα είναι ισοδύναμα. Σε οποιοδήποτε τετράπλευρο ΑΒΓΔ, αν σημειώσουμε με Μ το μέσο της διαγωνίου του ΒΔ, σχηματίζονται τα τετράπλευρα ΑΜΓΔ και ΑΜΓΒ που είναι ισοδύναμα. 10. Όλα τα ισόπλευρα τρίγωνα είναι ισοδύναμα. 11. Το ευθύγραμμο τμήμα που συνδέει τα μέσα των δύο βάσεων κάθε τραπεζίου το διαιρεί σε δύο ισοδύναμα τραπέζια. 12. Δύο οποιαδήποτε τετράγωνα με ίσα εμβαδά είναι ίσα. 13. Κάθε τετράγωνο με πλευρά α και διαγώνιο δ έχει εμβαδό ίσο με δ2. 14. Κάθε τρίγωνο χωρίζεται από τη διάμεσο του σε δύο τρίγωνα ισοδύναμα. 15. Δύο οποιαδήποτε ισοδύναμα ορθογώνια τρίγωνα είναι ίσα. ΑΠΑΝΤΗΣΕΙΣ [1 – Λ, 2 – Σ, 3 – Σ, 4 – Σ 5 – Λ, 6 – Λ, 7 – Σ, 8 – Σ, 9 – Λ. 10 – Λ, 11 – Σ , 12 – Σ, 13 – Λ ,14 – Σ, 15 – Λ]

Ασκήσεις

Άσκηση 1η Ένα ισόπλευρο τρίγωνο ΒΕΖ είναι εγγεγραμμένο σε τετράγωνο πλευράς α, όπως φαίνεται στο παρακάτω σχήμα. Να υπολογίσετε το εμβαδό του τριγώνου ΒΕΖ ως συνάρτηση της πλευράς α του τετραγώνου. ΛΥΣΗ Επειδή ΑΒ = ΒΓ = α και ΒΕ = ΒΖ αφού το τρίγωνο ΒΕΖ είναι ισοσκελές έχουμε ότι τα ορθογώνια τρίγωνα ΑΒΕ και ΒΓΖ είναι ίσα. Αν ονομάσουμε ΑΕ = ΖΓ = β, τότε είναι και ΔΕ=ΔΖ=α–β με β<α. Στο ορθογώνιο τρίγωνο ΑΕΒ έχουμε: x2 = α2 + β2 και από το ορθογώνιο τρίγωνο ΔΕΖ έχουμε:

x2 = (α – β)2 + (α – β)2 = 2(α – β)2 από όπου προκύπτει: α2 + β2 = 2(α – β)2  β2 – 4αβ + α2 = 0, άρα β = (2 – 3 )α αφού β < α.

ΕΥΚΛΕΙΔΗΣ Β΄ 90 τ.2/42

-------------------------------------------------------------------------------------------------------------------- Μαθηματικά για την Β΄ Λυκείου -----------------------------------------------------------------------------------------------------------------

Ανάλογα προκύπτει ότι (Α Μ Γ )  Α Μ . (2) Αλ-

Τότε είναι x2 = α2 + β2 = α2 +(2– 3 )2α2 = = 4α2( 2– 3 ) και το εμβαδό του τριγώνου ΒΕΖ

x2 3 3 είναι: (ΒΕΖ)=  42 (2  3)  2 (2 3 3) 4 4 Β΄ Τρόπος: x x Αν ΕΖ=x, τότε  , οπότε AE    2 2 2

x   2 και AB  AE  BE        x  2  2

2

2

2

x 2  2 2x  4 2  0  x  



6 2



Σχόλια:



Για την κατασκευή του τριγώνου ΒΕΖ αρκεί εκατέρωθεν της ΒΔ να σχηματίσουμε τις γωνίες

(Μ Δ Γ )

λά (1), (2)

ΜΔ

(A M B ) (Α Μ Γ )   (Β Μ Δ ) (Μ Δ Γ )

2 2 m2 n2   2  ( M B  )  m 2   (ΒΜΔ)  n 2 2 m  (ΑΒΓ) = m2 + n2 +  2 + n2

Άσκηση 3η Στο τετράπλευρο ΑΒΓΔ τα Ε, Ζ είναι τα μέσα των πλευρών του ΑΒ και ΔΓ αντίστοιχα. Συμβολίζουμε με Ε1=(AΘΖ), Ε2=(ΖΒΗ), Ε3=(ΕΗΓ) και Ε4 = (ΔΘΕ). Να δείξετε ότι: Ε1 + Ε3 = Ε2 + Ε4

ˆ   ˆ = 30 . 



Μπορούμε πλέον να υπολογίσουμε και τους τριγωνομετρικούς αριθμούς της γωνίας των 15ο χωρίς την χρήση τύπων της τριγωνομετρίας. π.χ. 15 

  x 



 6 2





6 2 4

η

Άσκηση 2 Δίνεται τρίγωνο ΑΒΓ και σημείο Δ της πλευράς ΒΓ τέτοιο ώστε: ΒΔ = κ·ΔΓ, με κ>0. Αν Μ τυχαίο σημείο της ΑΔ να δείξετε ότι: α) (ΑΔΓ) = 1 (ΑΒΓ). κ 1

β) Αν (ΑΜΒ) = m2, (AMΓ) = n2 και (ΓΜΔ) =  2 , 0, να υπολογιστεί το εμβαδό του όπου m, n,    τριγώνου ΑΒΓ ως συνάρτηση των m, n, και  . ΛΥΣΗ α) Τα τρίγωνα ΑΔΓ και ΑΒΓ έχουν κοινό ύψος,

(ΑΔΓ)    = = =  (ΑΒΓ)          1  Επομένως (ΑΔΓ) = (ΑΒΓ). (κ  1) κ 1

οπότε:

Παρατήρηση: Αν κ = 1 τότε η ΑΔ είναι διάμεσος του τριγώνου ΑΒΓ και (ΑΔΓ) =

1 (ΑΒΓ), που είναι η 2

γνωστή πρόταση.

β) Επίσης τα τρίγωνα ΑΜΒ και ΒΜΔ έχουν κοι-

νό ύψος, οπότε:

(ΑΜΒ) ΑΜ  . (1) (ΒΜΔ) ΜΔ

ΛΥΣΗ Αρκεί Ε1 + Ε3 +(ΖΗΕΘ) = Ε2 + Ε4 + (ΖΗΕΘ), δηλαδή (ΑΖΓΕ) = (ΒΖΔΕ). ράγματι με βάση τη γνωστή πρόταση «η διάμεσος κάθε τριγώνου το χωρίζει σε δύο ισοδύναμα τρίγωνα» έχουμε: ΑΕΖ)=(ΒΕΖ), στο τρίγωνο ΑΕΒ και (ΓΖΕ)=(ΔΖΕ) στο τρίγωνο ΔΖΓ. ελικά με πρόσθεση κατά μέλη παίρνουμε: (ΑΖΓΕ) = (ΒΖΔΕ). Άσκηση 4η Θεωρούμε οξυγώνιο τρίγωνο ΑΒΓ και το περιγγεγραμμένο του κύκλο κέντρου Ο. Έστω Α΄. Β΄, Γ΄ τα αντιδιαμετρικά των κορυφών του Α, Β, Γ αντίστοιχα. Να αποδείξετε ότι: α) Αν Η το ορθόκεντρο του τριγώνου ΑΒΓ, τότε τα τετράπλευρα ΗΒΑ΄Γ, ΗΓΒ΄Α και ΗΑô είναι παραλληλόγραμμα. β) (ΑΒ΄ΓΑ΄ΒΓ΄) = 2(ΑΒΓ) ΛΥΣΗ α) Τα τμήματα ΒΗ και Α΄Γ είναι παράλληλα αφού είναι κάθετα στο τμήμα ΑΓ, καθόσον ΑΑ΄ είναι διάμετρος. Για τον ίδιο λόγο τα τμήματα ΓΗ και Α΄Β είναι παράλληλα. Επομένως το τετράπλευρο ΗΒΑ΄Γ είναι παραλληλόγραμμο. Ανάλογα αποδεικνύεται ότι και τα τετράπλευρα αυτά ΗΓΒ΄Α και ΗΑΓΑ΄Β είναι παραλληλόγραμμα. β) Με δεδομένο ότι κάθε διαγώνιος παραλληλογράμμου το χωρίζει σε δύο ίσα και κατά συνέπεια ισοδύναμα τρίγωνα έχουμε: (ΑΒ΄ΓΑ΄ΒΓ΄) =

ΕΥΚΛΕΙΔΗΣ Β΄ 90 τ.2/43

-------------------------------------------------------------------------------------------------------------------- Μαθηματικά για την Β΄ Λυκείου -----------------------------------------------------------------------------------------------------------------

= (ΑΒΓ) + (ΑΒ΄Γ) + (ΓΑ΄Β) + (ΒΓΆ) = = (ΑΒΓ) + (ΑΗΓ) + (ΓΗΒ) + (ΒΗΑ) = = (ΑΒΓ) + (ΑΒΓ) = 2(ΑΒΓ)

Το τετράπλευρο ΚΛΜΝ είναι παραλληλόγραμ8 μο και (ΚΛΜΝ) = (ΑΒΓΔ) 9 ΛΥΣΗ BE΄  1    ΄ / /  και τα τρίγωνα ΒΕ΄Ζ   3  BE΄Z    1 2  1 . και ΒΑΓ είναι όμοια άρα     3  9

Όμοια ΘΗ΄// ΑΓ, ΗΖ΄//ΔΒ, ΕΘ΄//ΑΔ. Συνεπώς το τετράπλευρο ΚΛΜΝ είναι παραλληλόγραμμο. Εύκολα βρίσκουμε ότι ΚΕΕ΄=ΒΙΕ΄, ΛΖΖ΄=ΒΙΖ κ.λ.π. Άρα (ΚΛΜΝ) = =(ΕΕ΄ΖΖ΄ΗΗ΄ΘΘ΄)+(ΒΙΕ΄)+(ΒΙΖ)+(ΔΤΗ΄) + (ΔΤΘ) = = (ΕΕ΄ΖΖ΄ΗΗ΄ΘΘ΄)+(ΒΕ΄Ζ)+(ΔΗ΄Θ) = Β΄ τρόπος (Άμεση απόδειξη)

ˆ   ˆ  180  ( )  R  R  1  A΄OB

( ΄)

1 9

1 9

=(ΑΒΓΔ)–(ΑΘΈ)–(ΓΖΉ)= ()  ()  () =

R R

(ΑΟΒ) = (ΑΌΒ). Ομοίως (ΒΟΓ) = (Β΄ΟΓ) και (ΓΟΑ) = (ΓΌΑ) οπότε με πρόσθεση κατά μέλη προκύπτει το ζητούμενο. Άσκηση 5η Δίνεται τρίγωνο ΑΒΓ με ΑΒ < ΑΓ εμβαδού 10 τετραγωνικών μονάδων και η διχοτόμος του ΑΔ. Πάνω στην πλευρά ΑΓ παίρνουμε σημείο Κ τέτοιο ώστε ΑΚ = ΑΒ. Αν η ΒΚ τέμνει την διχοτόμο της γωνίας Α στο σημείο Μ. Να υπολογίσετε το εμβαδό του τριγώνου ΑΜΓ. ΛΥΣΗ Στο ισοσκελές τρίγωνο ΑΒΚ η ΑΜ είναι διχοτόμος οπότε και διάμεσος. Άρα (ΑΜΚ) = (ΑΒΜ), ομοίως (ΚΜΓ) = (ΒΜΓ). Άρα (ΑΒΓ) = (ΑΒΚ) +(ΒΓΚ) = 2 [(ΑΜΚ) + (ΚΜΓ)]= 10 2(ΑΜΓ)(ΑΜΓ)=  5 . 2

1 8 ( )  ( )  ( ) 9 9

Άσκηση 7η Δίνεται παραλληλόγραμμο ΑΒΓΔ. Επάνω στις πλευρές του ΑΒ και ΓΔ παίρνουμε τα σημεία Ε και Ζ τέτοια ώστε ΑΕ = ΓΖ = 1/3 ΑΒ και επάνω στις πλευρές του ΑΔ και ΒΓ τα σημεία Η και Θ τέτοια ώστε ΒΘ = ΔΗ = 1/3 ΒΓ. Τα τμήματα ΑΖ, ΒΗ, ΓΕ και ΔΘ τεμνόμενα ανά δύο σχηματίζουν τετράπλευρο ΜΝΡΚ.

(ΒΗΔΘ) 13 = . (ΜΝΡΚ) 3 β) Να δείξετε ότι (ΑΒΓΔ) = 13 (ΜΝΡΚ).

α) Να δείξετε ότι:

ΛΥΣΗ Άσκηση 6η Δίνεται τετράπλευρο ΑΒΓΔ και τα ζεύγη σημείων Ε, Ε΄ Ζ, Ζ΄, Η, Η΄ , Θ, Θ΄ που τριχοτομούν τις πλευρές ΑΒ, ΒΓ, ΓΔ και ΔΑ του τετραπλεύρου. Τα ευθύγραμμα τμήματα Ε΄Ζ, Ζ΄Η, Η΄Θ και Θ΄Ε προεκτεινόμενα σχηματίζουν νέο τετράπλευρο ΚΛΜΝ. Να δείξετε ότι: ΕΥΚΛΕΙΔΗΣ Β΄ 90 τ.2/44

-------------------------------------------------------------------------------------------------------------------- Μαθηματικά για την Β΄ Λυκείου -----------------------------------------------------------------------------------------------------------------

α) Έχουμε ΒΘ = // ΔΗ και ΕΑ//= ΓΖ, οπότε τα τετράπλευρα ΒΗΔΘ και ΑΕΓΖ είναι παραλληλόγραμμα. Επομένως και το ΜΝΡΚ είναι επίσης παραλληλόγραμμο. Τα παραλληλόγραμμα ΒΗΔΘ και ΜΝΡΚ έχουν κοινό ύψος, οπότε

()          (K)  

      1  1 2   1      2 KH 2 13  3 2  3 2  (1) Προφανώς ΘΝ=ΚΗ  3 3   1 από την ισότητα των ΓΘΝ, ΑΚΗ και     2 β) Τα παραλληλόγραμμα ΒΗΔΘ και ΜΝΡΚ έχουν 1

κοινό ύψος, οπότε

( )    3 (2). Πολ() 

Άσκηση 9η α) Έστω ΑΒΓΔ το κυρτό τετράπλευρο και Κ, Λ, Μ, και Ν τα μέσα των πλευρών του ΑΒ, ΒΓ, ΓΔ και ΔΑ αντίστοιχα. Αν Ο είναι το σημείο τομής των ΚΜ και ΛΝ, να δείξετε ότι: (ΑΚΟΝ) + (ΓΛΟΜ) = (ΒΚΟΛ) + (ΔΜΟΝ) β) Δίνεται τυχαίο κυρτό τετράπλευρο. Κάθε του πλευρά την χωρίζουμε σε τέσσερα ίσα τμήματα. Γραμμοσκιάζουμε εναλλάξ τα σχηματιζόμενα τετράπλευρα. Να αποδείξετε ότι το εμβαδόν του σκιασμένου χωρίου ισούται με το εμβαδόν του μη σκιασμένου (η στρεβλή σκακιέρα) ΛΥΣΗ α) Το τετράπλευρο ΚΛΜΝ είναι παραλληλόγραμμο, οπότε το Ο είναι κοινό μέσο των ΚΜ και ΛΝ . Στο τρίγωνο ΑΟΒ η ΚΟ είναι διάμεσος, οπότε (ΑΟΚ) = (ΒΟΚ). Ανάλογα έχουμε (ΒΟΛ)=(ΓΟΛ), (ΓΟΜ)=(ΜΟΔ), (ΔΟΝ)=(ΑΟΝ)

λαπλασιάζοντας κατά μέλη τις (1), (2) βρίσκουμε:

 AB 

(K)

 13 , δηλαδή (ΑΒΓΔ)=13 (ΜΝΡΚ).

Άσκηση 8η Δίνεται τραπέζιο ΑΒΓΔ με ΑΔ // ΒΓ και Ε το μέσο της ΑΒ. α) Να δείξετε ότι (ΓΕΔ) =

1 (ΑΒΓΔ) 2

β) Αν τα τμήματα ΕΓ και ΕΔ τέμνουν τη ΖΘ στα σημεία Κ και Λ, να δείξετε ότι: (ΓΚΛΔ) = (ΕΒΖΚ) + (ΑΘΛΕ) ΛΥΣΗ α) Αρκεί (ΒΓΕ)+ (ΑΔΕ)=

1 ( ) . Πράγματι: 2

Επομένως παίρνουμε: (ΑΚΟΝ) + (ΓΛΟΜ) = = (ΑΟΚ) + (ΑΟΝ) + (ΓΟΛ) + (ΓΟΜ) = = (ΒΟΚ) + (ΔΟΝ) + (ΒΟΛ) + (ΔΟΜ) = =(ΒΟΚ)+(ΒΟΛ)+(ΔΟΝ)+(ΔΟΜ)=(ΒΚΟΛ)+(ΔΜΟΝ) β) Σύμφωνα με το προηγούμενοι ερώτημα το Γ3 είναι το μέσο των τμημάτων Α3Ε3 και Γ1Γ5.

Αν συμβολίσουμε με h το ύψος του τραπεζίου, έχουμε: (ΒΓΕ)+ (ΑΔΕ) = =

1 h 1 h B   A  = 2 2 2 2

B  A 1  h  ( ) . 4 2

Στη συνέχεια διαπιστώνουμε ομοίως ότι τα Γ2 , Γ4 είναι τα μέσα των Γ1Γ3, Α2Ε2 και Γ3Γ5, Α4Ε4 αντιστοίχως, οπότε και τα Β3 , Β2, Β4 καθώς και Δ3, Δ2, Δ4 είναι μέσα αντιστοίχων τμημάτων. Σύμφωνα με το προηγούμενο ερώτημα από το Δ2 έχουμε: β) Αρκεί (ΓΚΛΔ)+(ΕΚΛ)=(ΕΒΖΚ)+(ΑΘΛΕ)+(ΕΚΛ), x1 + x4 = y1 + y4, από το Δ4, x2 + x3 = y2 + y3 , …. 1 Ομοίως από το Β2 έχουμε x5 +x8 =y5+ y8 και από ή (ΓΕΔ) = (ΑΒΖΘ), ή (ΑΒΓΔ) = (ΑΒΖΘ). το Β4 x6 +x7 =y6+ y7 . με πρόσθεση κατά μέλη έ2 χουμε: x1 + x2 + …+ x8 = y1 + y2 + …+ y8 δηλαδή Πράγματι: το εμβαδόν του σκιασμένου χωρίου ισούται με το 1 A   A   h h (ΑΒΓΔ). (ΑΒΖΘ)= εμβαδόν του μη σκιασμένου. 2 4 2 ΕΥΚΛΕΙΔΗΣ Β΄ 90 τ.2/45

-------------------------------------------------------------------------------------------------------------------- Μαθηματικά για την Β΄ Λυκείου -----------------------------------------------------------------------------------------------------------------

Β΄ΛΥΚΕΙΟΥ

ΕΥΘΕΙΑ ΚΑΙ ΚΥΚΛΟΣ (εχθροί ή φίλοι;) Του Κώστα Βακαλόπουλου

Στο άρθρο που ακολουθεί παραθέτουμε μια σειρά από ασκήσεις στις οποίες συνυπάρχουν άλλοτε αρμονικά και άλλοτε ανταγωνιστικά οι δύο βασικές γραμμές της ευθείας και του κύκλου. Σας καλούμε να παρακολουθήσετε τον αγώνα τους και στο τέλος να δώσετε το έπαθλο στη γραμμή που νομίζετε ότι κέρδισε στα σημεία! ΑΣΚΗΣΗ 1 Θεωρούμε τα σημεία M  2λ  1, 3  λ  , λ   . Α) Να βρεθεί ο γεωμετρικός τόπος (c) των σημείων Μ. Β) Να βρεθεί το σημείο Α του (c) που απέχει από την αρχή των αξόνων Ο την ελάχιστη απόσταση καθώς και η ελάχιστη αυτή απόσταση. Γ) Αν Β το σημείο που τέμνει ο (c) τον άξονα x΄x να βρεθεί το εμβαδόν του τριγώνου ΟΑΒ. Δ) Να βρεθεί ο γεωμετρικός τόπος των σημείων Ν του επιπέδου για τα οποία το εμβαδόν του τριγώνου ΝΑΒ ισούται με 20. ΛΥΣΗ Α) Έστω Μ τυχαίο σημείο του ζητούμενου γ.τ. Τότε x M  2λ  1 (1) και y M  3  λ (2) για κάποιο λ   .

ΠΑΡΑΤΗΡΗΣΗ Σε μια άσκηση που ζητείται ο γ.τ. των σημείων Μ που πληρούν μια ιδιότητα Ι, αφού ανακαλύψουμε τη γραμμή  c  πάνω στην οποία κινείται το σημείο Μ πρέπει, αντίστροφα, να ελέγξουμε αν κάθε σημείο της γραμμής  c  έχει την ιδιότητα Ι. Πολλές φορές κατά το στάδιο της λύσης μπορούμε να χρησιμοποιήσουμε ισοδυναμίες, οπότε έχουμε συγχρόνως το ευθύ και το αντίστροφο. Έτσι το παραπάνω ερώτημα θα μπορούσε να M  x 0 , y0  αντιμετωπισθεί ως εξής: Έστω σημείο του επιπέδου και

 c

ο γ.τ. Τότε

M   c   Υπάρχει λ   ώστε x 0  2λ  1 και y 0  3  λ  Υπάρχει λ   ώστε λ 

x0 1  3  y0  x0  2y0  5  0 . 2 Άρα ο γ.τ. είναι η ευθεία (ε): x  2y  5  0 . και λ  3  y0 

Β) Το σημείο της ευθείας που έχει τη μικρότερη απόσταση από την αρχή των αξόνων είναι το σημείο τομής της ευθείας (ε) με τη κάθετη ευθεία (η) από το Ο στην (ε). Η ευθεία (η) έχει εξίσωση:

y  2x αφού: ε  η  λε  λη  1 λη 

Έχουμε: (2)  λ  3  y M . Οπότε: (1)  x M  2y M  5  0 (Ι)  M   ε  , όπου (ε) η ευθεία με εξίσωση: x  2y  5  0 . Αντίστροφα, Έστω M  x M , y M   (ε) . Τότε προφανώς υπάρχει λ   ώστε x M  2λ  1

xM 1 . 2 Τότε: M   ε    2λ  1  2y M  5  0 

x0 1 2

Λύνοντας το σύστημα:

1  2. 1 2

 y  2x    x  2y  5  0 x 1 ...   y  2 βρίσκουμε ότι το ζητούμενο σημείο είναι το A 1, 2  .

Πρόκειται για το λ 

Η απόσταση του Ο  0, 0  από την ευθεία (ε) είναι:

yM  3  λ .

5  5 . 5 12  22 Γ) Αντικαθιστώντας y  0 στην εξίσωση της

Άρα το Μ θα έχει συντεταγμένες της μορφής  2λ  1,3  λ  .

d

0  20 5



ευθείας (ε) έχουμε x  5 . Άρα είναι: B  5, 0  .





Επίσης OA  1, 2  , OB   5, 0  και ΕΥΚΛΕΙΔΗΣ Β΄ 90 τ.2/46

-------------------------------------------------------------------------------------------------------------------- Μαθηματικά για την Β΄ Λυκείου -----------------------------------------------------------------------------------------------------------------

  1 2 det OA, OB   10 . 5 0 1 Άρα:  OAB    10  5 . 2 Δ) Έστω N  x 0 , y0  τυχαίο σημείο του  επιπέδου. Τότε έχουμε: AN   x 0  1, y 0  2  ,    x 1 y0  2 AB   4, 2  και det AN,AB  0  4 2









...  2  x 0  2y0  5 . Αν (c ) ο ζητούμενος γ.τ., τότε: Ν  (c)   ABN   20 

1  2(x 0  2y 0  5)  20  x 0  2y 0  5  20 2 ...  x 0  2y0  25  0 ή x 0  2y0  15  0 . ΄Αρα ο γ.τ. είναι η ένωση των ευθειών:

 ε1  : x  2y  25  0 και

 ε 2  : x  2y  15  0 . ΑΣΚΗΣΗ 2 Θωρούμε τα σημεία:

Μ Α)





2  ημθ  2, 2  συνθ  2 , θ   .

(Ιδιότητα Ι). Όμως για κάθε θ   2

 x 2 ημ2θ  συν2θ  1. Άρα:  M   2 

 x M  2    yM  2  2

 y 2    M   1  2 

 2  M   c  , όπου (c)

o κύκλος με εξίσωση:

 x  2   y  2

Αντίστροφα: Έστω Μ

σημείο του κύκλου (c),

2

2

 2.

τότε:  x M  2    y M  2   2  2

2

2

2

xM  2  x M  2   yM  2       1 . Έστω α  2   2  2  y 2 και β  M . Επειδή 2 α 2  β 2  1 το σημείο N  β, α  απέχει από την αρχή των αξόνων απόσταση ίση με 1. Άρα είναι σημείο του τριγωνομετρικού κύκλου. Έστω θ μια από τις γωνίες με τελική πλευρά την ημιευθεία ΟΝ. Σύμφωνα με τον ορισμό των τριγωνομετρικών αριθμών ισχύει:

   y M  2 x 2  , M         2 2 yM  2  συνθ  2 . Άρα για κάθε σημείο Μ του xM  2  ημθ 2  παραπάνω κύκλου υπάρχει θ   ώστε να έχει συντεταγμένες, τις:

Να βρεθεί ο γ.τ. (c) των σημείων Μ .

2

ισχύει: 2





2  ημθ  2, 2  συνθ  2 .

Επομένως ο γ.τ. των σημείων Μ είναι ο κύκλος  c  με κέντρο Κ  2, 2  και ακτίνα ρ  2 .

Β) Αν x  y  μ , μ   (1) παραμετρική οικογένεια ευθειών του επιπέδου να βρείτε για ποιες τιμές του μ   οι ευθείες που παριστάνει η εξίσωση (1)

Β) Γεωμετρική λύση:

i) τέμνουν τον (c) ii) εφάπτονται στον (c) iii) είναι εξωτερικές του (c) .

του κέντρου του κύκλου από την ευθεία με την ακτίνα του κύκλου. Συγκεκριμένα.

Λύση Α) Έστω

M  x M , yM 

ζητούμενου γ.τ. Τότε

τυχαίο

σημείο

του

xM  2 2

και



Αν d  K, ε   ρ η ευθεία τέμνει τον κύκλο,



δηλαδή έχει με αυτόν δύο κοινά σημεία Αν d  K, ε   ρ η ευθεία εφάπτεται του



κύκλου, δηλαδή έχει με αυτόν ένα κοινό σημείο. Αν d  K, ε   ρ η ευθεία είναι εξωτερική του

xM  2ημθ2 (1) και

yM  2  συνθ  2 (2).

(1)  ημθ 

Ως γνωστόν η σχετική θέση ευθείας ε και κύκλου  Κ,ρ  εξαρτάται από τη σχέση της απόστασης

(2)  συνθ 

yM  2 , 2

κύκλου, δηλαδή δεν έχει με αυτόν κοινά σημεία.

ΕΥΚΛΕΙΔΗΣ Β΄ 90 τ.2/47

-------------------------------------------------------------------------------------------------------------------- Μαθηματικά για την Β΄ Λυκείου -----------------------------------------------------------------------------------------------------------------



Η εξίσωση της ευθείας είναι: x  y  μ  0

d  Κ, ε   i)

μ 2

2  2  μ 2



μ 2

 2  μ  2  2  μ  2

Επομένως αν 2  μ  2 τότε η ευθείες που παριστάνει η (1) τέμνουν τον κύκλο.

μ

 2  μ  2  μ  2 Επομένως αν 2 μ  2 τότε η ευθείες που παριστάνει η (1)

ii)

εφάπτονται του κύκλου. iii)

μ 2

 2  μ  2  μ  2 ή μ  2

Επομένως αν μ  2 ή μ  2 τότε η ευθείες που παριστάνει η (1) είναι εξωτερικές του κύκλου. Αλγεβρική λύση: Τα σημεία τομής των ευθειών που παριστάνει η (1) με τον κύκλο  c  έχουν συντεταγμένες τις λύσεις

 x  2 2   y  2 2  2 του συστήματος:   x  yμ  0   x 2  y 2  4x  4y  6  0 (2)  yμx (3)  Η εξίσωση (2) λόγω της (3) γίνεται:

x 2   μ  x   4x  4  μ  x   6  0  ...  2

2x 2  2  μ  4  x  μ 2  4μ  6  0

 

μ  2 ή μ  2 τότε η ευθείες που αν παριστάνει η (1) είναι εξωτερικές του κύκλου. ΑΣΚΗΣΗ 3 Δίνονται οι εξισώσεις: (λ  2)x  λy  1  0 , (λ  5)x  2y  2  0 , λ   . Α) Να δείξετε ότι οι παραπάνω εξισώσεις παριστάνουν ευθείες. Β) Να βρείτε τη σχετική θέση των ευθειών  ε1  ,  ε2  για κάθε πραγματική τιμή του λ Γ) Για ποιες τιμές του λ οι παραπάνω ευθείες είναι κάθετες. Δ) Για ποιες τιμές του λ οι παραπάνω ευθείες είναι παράλληλες. Για τη μικρότερη από τις τιμές αυτές να βρείτε την εξίσωση της μεσοπαράλληλης αυτών Ε) Να βρεθεί η ευθεία (ε΄) που τέμνει τις ευθείες  ε1  ,  ε 2  που προκύπτουν για λ  1 στα σημεία Α και Β έτσι ώστε το σημείο M  4, 2  να είναι μέσο του ΑΒ. Λύση Α) Προφανώς (2,)   0,0 και ( 5,2)   0,0 , για κάθε    . Άρα και οι δύο εξισώσεις παριστάνουν ευθείες  Β) Προφανώς  ε1  / /v   λ, λ  2  και



 ε 2  / /u   2, λ  5

(4).

Αν Δ  0 τότε η εξίσωση (4) έχει δύο ρίζες οπότε οι ευθείες τέμνουν τον κύκλο. Αν Δ  0 τότε η εξίσωση (4) έχει μια διπλή ρίζα οπότε οι ευθείες εφάπτονται του κύκλου. Αν Δ  0 τότε η εξίσωση (4) δεν έχει ρίζες (πραγματικές) οπότε οι ευθείες είναι εξωτερικές του κύκλου.

και  2  τέμνονται. Αν   1 τότε οι ευθείες είναι  1  : 3x  y  1  0 και   2  : 6x  3y  2  0 ,



δηλαδή   2  : 3x  y  1  0 , οπότε ταυτίζονται. 

i) 4 μ 2  4  0  2  μ  2 Επομένως

 1  : 2x  4y  1 ,



2

2



αν

2  μ  2 τότε η ευθείες που παριστάνει η (1) τέμνουν τον κύκλο.





ii) 4 μ 2  4  0  μ  2 .

Επομένως

αν

μ  2 τότε η ευθείες που παριστάνει η (1) εφάπτονται του κύκλου τον κύκλο.



  λ λ  2 det  v,u    λ2  3λ  4  2 λ  5  1λ λ 4 , οπότε: det  v,u  0  1 ή 4.   Αν   1 και   4 τα διανύσματα v και  u δεν είναι παράλληλα. Άρα και οι ευθείες  1 

Δ  4  μ  4  8  μ  4μ  6  ...  4  μ  4 2

 

και v, u  0

Έχουμε:

Έστω Δ η διακρίνουσα της εξίσωσης (4). 



iii) 4 μ 2  4  0  μ  2 ή μ  2 . Επομένως

και

Αν

  4

 2  : y  

τότε δηλαδή

οι

ευθείες

είναι:

 1  : y  

1 1 x 2 4

1 x  1 , οπότε είναι παράλληλες 2

με την Ευκλείδεια έννοια, αφού έχουν τον ίδιο συντελεστή διεύθυνσης και τέμνουν τον άξονα y΄y

 

1 4

στα διαφορετικά σημεία  0,     0,1 . ΕΥΚΛΕΙΔΗΣ Β΄ 90 τ.2/48

-------------------------------------------------------------------------------------------------------------------- Μαθηματικά για την Β΄ Λυκείου -----------------------------------------------------------------------------------------------------------------

Σημείωση: Η διερεύνηση της σχετικής θέσης των δύο ευθειών μπορεί να γίνει αν θεωρήσουμε το σύστημα:

 (  2)x  y  1 και το διερευνήσουμε με τη  (  5)x  2y  2 μέθοδο των οριζουσών. Οι ορίζουσες του παραπάνω συστήματος είναι: D   2  3  4      1   4  ,

D x  2    1 , D y    1 . Οπότε:  Αν   1 και   4 το σύστημα έχει μοναδική λύση την:

 Dx Dy   2 1  , ,    D D  λ4 λ4

 x, y   

οπότε οι ευθείες τέμνονται και μάλιστα στο σημείο με συντεταγμένες τη παραπάνω μοναδική λύση του συστήματος. 

 3x  y  1 Αν   1 το σύστημα γίνεται:  6x  2y  2 3x  y  1 που προφανώς έχει δηλαδή:  3x  y  1 άπειρες

 



 

Δ) Ομοίως  1  / /  2   v/ /u  det v,u  0 

  1 ή   4 . Για   4 έχουμε τις ευθείες 1  1  : x  2y   0 ,  2  : x  2y  2  0 . 2 Η μεσσοπαράλληλη ευθεία των ( 1 ) και ( 2 ) θα 1 και θα έχει συντελεστή διεύθυνσης λ   2 διέρχεται από το μέσο οποιουδήποτε τμήματος που έχει άκρα στις ευθείες ( 1 ) και ( 2 ). Οι παραπάνω ευθείες ( 1 ) και ( 2 ) τέμνουν τον άξονα x΄x στα σημεία

 1  Α  ,0  2 

και

Β  2, 0  αντίστοιχα.

Οπότε το μέσο Μ του τμήματος ΑΒ είναι ασφαλώς ένα σημείο από το οποίο θα διέλθει η μεσοπαράλληλη που αναζητούμε. ‘Όμως

3  Μ  , 0  . Άρα η ζητούμενη εξίσωση είναι: 4  1 3 y  0    x   , δηλαδή : 4x  8y  3  0 . 2 4

κάθε ζεύγος:  x, y    x,1  3x  , x   οπότε οι ευθείες



λύσεις

ταυτίζονται.   4 Αν

το

σύστημα

γίνεται:

1   2x  4y  1  x  2y   , δηλαδή:  2 που   x  2y  2  x  2y  2 προφανώς είναι αδύνατο οπότε οι ευθείες είναι παράλληλες με την Ευκλείδεια έννοια. Γ) Στο ερώτημα αυτό, θα χρησιμοποιήσουμε   πάλι τα διανύσματα v και u του ερωτήματος Α) που είναι παράλληλα στις ευθείες 1 και 2 . Η αναφορά στα διανύσματα αυτά γίνεται γιατί η συνθήκη καθετότητας δύο διανυσμάτων μέσω του εσωτερικού γινομένου τους δεν απαιτεί κανένα περιορισμό. Αντιθέτως η συνθήκη καθετότητας μέσω συντελεστών διεύθυνσης απαιτεί διερεύνηση για τη περίπτωση που τα διανύσματα είναι παράλληλα στον άξονα y΄y. Έχουμε προφανώς        v, u  0 , οπότε:  1    2   v  u  v  u  0 

 ,   2    2,   5  0  2λ   λ  2  λ  5   0  λ2  9λ  10  0 λ

9  41 . 2

ΓΕΝΙΚΑ Αν 1 και 2

δυο

παράλληλες

ευθείες

με

εξισώσεις:  1  : x  y  1  0 και

  2  : x   y   2  0 .

Εύκολα αποδεικνύεται

ότι η εξίσωση της μεσοπαράλληλης ε των δυο ευθειών είναι:    : x   y 

1   2 0 2

  1 προκύπτουν οι ευθείες  1  : x  y  1  0 και  2  : 2x  y  1  0 .

Δ) Για

Έστω Α και Β τα σημεία στα οποία η ευθεία ε΄ τέμνει τις  1  και   2  . Αν x A , x B οι τετμημένες και y A , y B οι τεταγμένες των σημείων

ΕΥΚΛΕΙΔΗΣ Β΄ 90 τ.2/49

-------------------------------------------------------------------------------------------------------------------- Μαθηματικά για την Β΄ Λυκείου -----------------------------------------------------------------------------------------------------------------

Α, Β θα ισχύει:

xA  xB  4  x A  x B  8 (1) 2

yA  yB  2  y A  y B  4 (2) 2 x A  yA  1  0  yA  x A  1 (3)

Α) Αν Β το συμμετρικό ενός σημείου Α ως προς ευθεία (ε) και A     τότε η ευθεία (ε) είναι μεσοκάθετος του ευθυγράμμου τμήματος ΑΒ και αντιστρόφως.

και

2x B  y B  1  0  y B  1  2x B (4) Αντικαθιστώντας στη (2) τα y A , y B από τις (3), (4) βρίσκουμε x A  x B  4 και τελικά A  4,3

B  4, 7  . Επομένως η ζητούμενη ευθεία είναι η κατακόρυφος ευθεία με εξίσωση: x  4

και

Προφανώς A     , αφού x   y   3  4  0

Έστω B  ,   με B  A το ζητούμενο σημείο





οπότε AB     6,   1  0 . Πρέπει και αρκεί το μέσο Μ του ΑΒ να ανήκει στην (ε) και το τμήμα ΑΒ να είναι κάθετο στην ευθεία (ε). Όμως

   6  1  ,   , οπότε: 2   2   6  1        3  0      1 (1). 2 2 Επειδή ο συντελεστής διεύθυνσης της (ε) είναι 1  0 για να είναι      πρέπει και αρκεί το ΑΒ να έχει συντελεστή διεύθυνσης δηλαδή   6 και         1 (2). Αλλά  2  

 1 1  1      5 (3) 6

Λύνοντας το σύστημα των (1) και (3) έχουμε:   3 και   2 . Άρα το ζητούμενο σημείο είναι το B  2,3 . Συντομότερα:

Έχουμε:



   / /v  1,1

  AB  0 , οπότε            v    v    0 

και

 1   6   1   1  0      5 κ.λ.π. Σημείωση: Το παραπάνω ερώτημα λύνεται και γεωμετρικά ως εξής: Αν Θ το σημείο τομής των  1  και   2 

Β) Έστω Δ και Ε τα σημεία που η ευθεία ε τέμνει τον κύκλο (c). Αν Η η προβολή του Β στην ε, το τρίγωνο ΒΗΔ είναι ορθογώνιο. Από το πυθαγόρειο θεώρημα έχουμε:  2   2   2 (3).

και Ν το συμμετρικό του Θ ως προς Μ τότε τα Α, Β είναι κορυφές του παραλληλογράμμου ΘΑΝΒ οπότε προσδιορίζονται. ΑΣΚΗΣΗ 4 Δίνεται η ευθεία (ε): x  y  3  0 και το σημείο

A  6, 1 .

Α) Να βρεθεί το συμμετρικό σημείο Β του Α ως προς την ευθεία (ε). Β) Να βρείτε την εξίσωση του κύκλου  c  που έχει κέντρο το Β αποκόπτει από την ευθεία ε τμήμα με μήκος 4 2 . Λύση ΕΥΚΛΕΙΔΗΣ Β΄ 90 τ.2/50

-------------------------------------------------------------------------------------------------------------------- Μαθηματικά για την Β΄ Λυκείου -----------------------------------------------------------------------------------------------------------------

233

Όμως,   d(B, ) 

12   1



2

4 2 2 2

    2

και   2 2 . Οπότε : 2  2 2  2 2

2

  2  16    4 . Άρα ο κύκλος έχει ακτίνα 4 και εξίσωση:  x  2    y  3   16 . 2

2

ΑΣΚΗΣΗ 5 Δίνεται η εξίσωση:

x 2  y 2  2x  2  1    y  1  2  0

διέρχονται όλοι οι κύκλοι που παριστάνει η εξίσωση (1). Γ) Μια ευθεία εφάπτεται σε ένα κύκλο αν και μόνο αν το κέντρο του κύκλου απέχει από την ευθεία όσο η ακτίνα του. Έχουμε,

d  K,   

  (1  )  1 12  (1) 2



2 2

  2 

(1),

Άρα όλοι οι κύκλοι που παριστάνει η (1) εφάπτονται στην ε.

Α) Να αποδειχθεί ότι η (1) παριστάνει κύκλο για κάθε λ    Β) Να αποδειχθεί ότι όλοι οι κύκλοι που παριστάνει η (1) διέρχονται από σταθερό σημείο Γ) Να αποδειχθεί ότι οι παραπάνω κύκλοι εφάπτονται στην ευθεία (ε) με εξίσωση:

Δ) Για   1 και για   2 προκύπτουν οι κύκλοι ( c1 ) και ( c2 ) με κέντρα K1  1, 2  ,

λ   .

xy1 0 Δ) Αν ( c1 ) και ( c2 ) οι κύκλοι που προκύπτουν από την (1) για λ  1 και λ  2 αντίστοιχα, να

δείξετε ότι η εφαπτόμενες από το σημείο Α  4, 5  στον κύκλο ( c1 ) εφάπτονται και στον

K 2  2, 1 και ακτίνες 1  2 και  2  2 2 αντίστοιχα. Προφανώς η εφαπτόμενη του κύκλου ( c1 ) από το Α δεν είναι κατακόρυφη (γιατί;)

Έστω y  5    x  4  δηλαδή

x  y  4  5  0 η εξίσωση της ζητούμενης ευθείας η. Αρκεί να αποδείξουμε ότι:

d  K1 ,    1  d  K 2 ,    2

κύκλο ( c2 ). Λύση Α) Αν   2 ,   2 1    και   1  2 τότε

 2  2  4   2    2 1      4 1  2   ...  8 2  0 . Άρα η (1) παριστάνει κύκλο με    κέντρο    ,     ,   1 και ακτίνα 2  2 2

2

8 2    2 για κάθε     . 2 Β) Ζητάμε σημείο  x 0 , y 0  , τέτοιο ώστε

x 0 2  y0 2  2x 0  2 1    y0  1  2  0 δηλαδή

x02   y0 1  2  x0  y0  1  0 2

Πράγματι: (2)



για κάθε    . Για να έχει η εξίσωση (2) (ως προς λ) άπειρες λύσεις (τουλάχιστον δύο) πρέπει και αρκεί: ισχύει για

2  x 0 2   y 0  1  0 που προφανώς   x 0  y 0  1  0

 x 0 , y0    0,1 . Άρα υπάρχει ζεύγος

 x 0 , y0    0,1

για το οποίο η εξίσωση (2) 

αληθεύει για κάθε    . Το ζεύγος αυτό είναι οι συντεταγμένες του σημείου από το οποίο

d  K1 ,   

  2  4  5

d  K 2 ,  

2  1  4  5

  (1) 2

(2)

d  K1 ,    1 

6  6 2  1

2

2  1 3  3 2  1 (3)

 

3  3 2  1 6  6 2  1

 2

 2 2  d  K 2 ,    2 .

ΕΥΚΛΕΙΔΗΣ Β΄ 90 τ.2/51

(2) (3) Άρα:

-------------------------------------------------------------------------------------------------------------------- Μαθηματικά για την Β΄ Λυκείου -----------------------------------------------------------------------------------------------------------------

αβ ≥ αβ (α, β>0) 2

Μια άσκηση του σχολικού βιβλίου και οι συνέπειες της στη βασική ανισότητα

Διονύσης Γιάνναρος

Θα ασχοληθούμε με την άσκηση 4 σελίδα 28 του σχολικού βιβλίου κατεύθυνσης Β΄ Λυκείου την οποία αναδιατυπώνουμε ως εξής: Θεωρούμε ορθογώνιο τρίγωνο ΟΔΕ ( ˆ 1ορθή, O ΟΔ<ΟΕ) και φέρουμε το ύψος ΟΑ και τη διάμεσο της ΟΒ. Θέτουμε ΔΑ =α, ΑΕ=β και αν Μ είναι εσωτερικό MA κ σημείο του ΑΒ, ώστε = , τότε MB λ    λ  ΟΑ  κ  ΟΒ ΟΜ  . Το παραπάνω συμπέρασμα λκ το θεωρούμε γνωστό όπως επίσης και τα: ΟΑ= αβ αβ (ύψος προς υποτείνουσα), ΟΒ = 2  (διάμεσος προς υποτείνουσα), α<β. Είναι: OA ·       2  ΟΒ = OA · OA = OA =α·β OB = OA · προβΟΑ και   2  2 λ  ΟΑ κ  ΟΒ  2    OA < OM < OB ή OA < < OB ή λκ      2 λ2 ΟΑ2  2 λ κ ΟΑ  ΟΒ  μ2 ΟΒ2  2 OA < < OB 2  λ  κ 2   1  α β  OA < λ2 αβ  2λκαβ  κ 2   < OB και λκ  2 

αβ 2 αβ κ  κ λ . λ αβ αβ 2 Αν στην (1) αντικαταστήσουμε το κ βρίσκουμε : 2αβ(α  β) αβ αβ < < . Από την άσκηση 7 2 α β έχουμε:

(γενικές ασκήσεις σχολικού βιβλίου γεωμετρίας σελ.88) και συνέπειές της έχουμε: i) Αν το Μ είναι το σημείο επαφής της πλευράς ΑΒ με τον εγγεγραμμένο κύκλο στο τρίγωνο ΟΑΒ τότε ΑΜ= τ-ΟΒ και ΜΒ = τ- ΟΑ και ii) Αν Μ είναι το σημείο επαφής της ΑΒ με τον παρεγγεγραμμένο κύκλο (Ιο ,ρο) τότε : ΑΜ= τ-ΟΑ και ΜΒ = τ- ΟΒ. ΟΑ  ΑΒ  ΟΒ  ΟΑ  Είναι όμως :τ-ΟΑ= 2 ΑΒ  ΟΒ  ΟΑ ΔΒ  ΔΑ  ΟΒ  ΟΑ   2 2 α β α β α   αβ β  αβ β ( β  α) 2 2 .   2 2 2 α( β  α ) . Με όμοιο τρόπο είναι τ-ΟΒ  2 Με βάση αυτά τα συμπεράσματα έχουμε τις παρακάτω εφαρμογές της σχέσης (1). 3η) Αν το Μ είναι το σημείο επαφής με τον εγγεγραμμένο κύκλο, τότε α( β  α) ΜΑ τ  ΟΑ α 2    ΜΒ τ  ΟΒ β( β  α) β

2

2

αβ <

αβ 1 αβ λ2 αβ  2λκαβ  κ 2   < 2 λκ  2 

Εφαρμογές 1η) Αν Μ μέσο του ΑΒ, τότε λ = μ, οπότε η (1) μετά από αντικαταστάσεις και απλοποιήσεις γίνεται

αβ <

a 2  14aβ  β 2 α  β < 2 4 

Αν ΟΜ διχοτόμος της γωνίας Α Ο Β ,τότε από  ΟΑ ΜΑ ΟΑ κ θεώρημα διχοτόμου έχουμε     . ΜΒ ΟΒ ΟΒ λ Είναι όμως

  α  β ΟΑ  αβ και ΟΒ  2

οπότε

κ α α  κ λ . λ β β Με αντικατάσταση του κ στην (1) βρίσκουμε

Οπότε

αβ 

α(α  β) 2  8αβ αβ  4αβ 2

<

αβ . 2

2( α  β ) 4η) Αν Μ είναι το σημείο επαφής της ΑΒ με τον παρεγγεγραμμένο κύκλο (Ιο ,ρο) τότε : ΜΑ α κ α   ,άρα κ   λ . Η σχέση (1) τώρα ΜΒ β λ β γίνεται

αβ 

β(α  β) 2  8αβ αβ  4α 2β

ΕΥΚΛΕΙΔΗΣ Β΄ 90 τ.2/52

2( α  β )

<

αβ . 2

------------------------------------------------------------------------------------------------------------------------- Μαθηματικά για την Γ΄ Τάξη ----------------------------------------------------------------------------------------------------------------------

Γ΄ΛΥΚΕΙΟΥ

Ασκήσεις Στατιστικής Φίλιππος Σερέφογλου και Βασίλης Καρκάνης

Άσκηση 1 Σε μια χώρα που αντιμετωπίζει οικονομικά προβλήματα οι μισθοί των 400 υπαλλήλων μιας εταιρίας του δημοσίου ακολουθούν περίπου κανονική κατανομή με μέση τιμή 1000€ και τυπική απόκλιση 100€. α) Να βρείτε τα χρήματα που διαθέτει ετησίως η εταιρία για τους μισθούς των υπαλλήλων της. β) Πόσοι υπάλληλοι παίρνουν μισθό πάνω από 1200€. γ) Αν το όριο φτώχιας για την εν λόγω χώρα είναι 900€ να βρεθεί ο αριθμός των υπαλλήλων που ζει κάτω από το όριο της φτώχιας. δ) Η διοίκηση της εταιρείας αποφασίζει αρχικά να μειώσει τους μισθούς όλων των υπαλλήλων κατά 10% και στη συνέχεια να τους αφαιρέσει το μηνιαίο επίδομα παραγωγικότητας που τους χορηγεί. Μετά την εφαρμογή των μέτρων αυτών ο μέσος μηνιαίος μισθός των υπαλλήλων γίνεται 850€. Να βρείτε πόσο ήταν το επίδομα και ποια είναι η νέα τυπική απόκλιση των μισθών . ε) Μια ομάδα εξωτερικών οικονομικών συμβούλων προτείνει στην εταιρία να ακυρώσει τα προηγούμενα μέτρα και να απολύσει 40 υπαλλήλους με μισθό 1.100 € και 25 υπαλλήλους με μισθό1.200 €. Να εξετάσετε από τις δυο μειώσεις του μηνιαίου μισθολογικού κόστους ποια είναι η μεγαλύτερη. Λύση Έχουμε: x = 1000€, s = 100€ και ν=400 ο αριθμός των υπαλλήλων. α) Είναι: (12 μήνες)·(400 υπάλληλοι)· x = =12·400·1000=4.800.000€. β) Πάνω από 1200€ παίρνει το 2,5% των υπαλλήλων δηλαδή 2,5%·400=10 υπάλληλοι. γ) Κάτω από το όριο της φτώχιας που είναι 900€ ζει το 16% δηλαδή 16%·400=64 υπάλληλοι. δ) Ο νέος μέσος μισθός έγινε y =850€. Όμως y = 0,9 x -λ  850=0,9·1000-λ  λ=50 οπότε το επίδομα που αφαιρέθηκε ήταν 50€. ε) Το μηνιαίο κόστος με τα μέτρα που πάρθηκαν είναι:(400 υπάλληλοι)· y =400·850 = 340.000€. Αν γίνουν οι απολύσεις όπως προτείνουν οι σύμβουλοι το αρχικό μηνιαίο μισθολογικό κόστος που είναι: 400·1000=400.000€ θα μειωθεί κατά 40·1100=44.000€ λόγω της απόλυσης των 40 υπαλλήλων με μισθό 1100€ και κατά 25·1200=30.000€ λόγω της απόλυσης των 20 υπαλλήλων με μισθό 1200€. Άρα συνολικά η μείωση θα είναι: 44.000+30.000= 74.000€ με αποτέλεσμα το μηνιαίο μισθολογικό κόστος της εταιρείας να περιοριστεί στα: 400.000-74.000=326.000€.

Άσκηση 2 Η βαθμολογία ενός τυχαίου δείγματος 100 μαθητών της Α’ Λυκείου που πήραν μέρος στο μαθηματικό διαγωνισμό «Θαλής» κυμάνθηκε από 0-20. Οι βαθμοί ταξινομήθηκαν σε 5 κλάσεις ίσου πλάτους και στη συνέχεια κατασκευάστηκε το ιστόγραμμα συχνοτήτων και σχετικών συχνοτήτων επί τοις εκατό καθώς και το κυκλικό διάγραμμα της κατανομής. Στα παραπάνω παρατηρήθηκε ότι: 1) Στο ιστόγραμμα συχνοτήτων το εμβαδό του ορθογωνίου της 1ης κλάσης ισούται με 10 2) Στο ιστόγραμμα συχνοτήτων επί τοις εκατό το ύψος του ορθογωνίου της 4ης κλάσης είναι 20 3) Στο κυκλικό διάγραμμα συχνοτήτων το τόξο που αντιστοιχεί στην 3η και 4η κλάση ισούται με 216ο. 4) Οι μαθητές της 2ης κλάσης είναι πενταπλάσιοι από τους μαθητές της 5ης κλάσης α) Για την κατανομή της βαθμολογίας να κατασκευάσετε και να συμπληρώσετε πίνακα συχνοτήτων, απολύτων και αθροιστικών. β) Να γίνει το ιστόγραμμα και το πολύγωνο των αθροιστικών σχετικών συχνοτήτων επί τοις εκατό και να βρείτε την διάμεσο της κατανομής. γ) Να βρείτε την μέση τιμή της κατανομής. δ) Να εκτιμήσετε το δείγμα ως προς την ομοιογένεια. Λύση R 20 Έχουμε: ν=100, R=20-0=20 και c= = =4 άρα  5 οι κλάσεις είναι: [0,4), [4,8), [8,12), [12,16), [16,20]. Λόγω του (1) είναι ν1=10. Λόγω του (2) είναι f 4 % =20 άρα f 4 =0,2. Λόγω του (3) είναι:α3,4=2160  3600·( f3 + f 4 ) = =2160  …  f3 =0,4. Λόγω του (4) είναι:ν2=5ν5 και ισχύει: νi = ν· f  οπότε: ν3=40, ν4=20, ν5=5 και ν2=25 οπότε:

ΕΥΚΛΕΙΔΗΣ Β΄ 90 τ.2/53

------------------------------------------------------------------------------------------------------------------------- Μαθηματικά για την Γ΄ Τάξη ----------------------------------------------------------------------------------------------------------------------

α) f x Κλάσεις xi νi fi Νi Fi i Fi% i· xi2·νi [-) νi % 0-4 2 10 0,10 10 0,10 10 10 20 40 4-8 6 25 0,25 30 0,35 25 35 150 900 8-12 10 40 0,40 70 0,75 40 75 400 4000 12-16 14 20 0,20 90 0,95 20 95 280 3920 16-20 18 5 0,15100 1 5 100 90 1620 ΣΥΝΟΛΟ 100 1 100 94010480

β)

τά 20% και στη συνέχεια κατά 12 μονάδες το δείγμα θα εξακολουθήσει να είναι ομοιογενές; Να δικαιολογήσετε την απάντησή σας. Λύση α)  ά x  R f (x)  2xx  2s s ά f (x)  0  xx  s  x  x x 0 s f   x   0  xx  s  x   f   x   0  x s s s   x  ώ f    ,  , f    ,  x x x   s ά ά ό έά  x    x s β) Είναι CV   0.08  0.1 άρα το δείγμα είναι x ομοιογενές. Η κατανομή των τιμών του δείγματος φαίνεται στο παρακάτω σχήμα:

 

Από το παραπάνω σχήμα έχουμε: ΑΔ  ΒΕ  ΓΖ    (Θ.Θ.). Επίσης: ΔΗ  ΕΘ  ΖΙ( Θ.Θ)    οπότε:

 





γ) Είναι: x =





   i·

i

1 δ) Έχουμε: s 2  

8 12  8 i

=



50  35 75  35

 ...    9,5  

940 =9,4. 100

x

2 

  - x

 

2

οπότε από τον

10480 9,42=16,44 άρα 100 s 4,05 = =0,43=43% s= 16, 44 =4,05 οπότε CV= x 9, 4 άρα το δείγμα δεν είναι ομοιογενές. Άσκηση 3 Ένα δείγμα παρατηρήσεων μεταβλητής  ακολουθεί κανονική κατανομή όπου το 16% των παρατηρήσεων παίρνουν τιμές μικρότερες από 23 . Επίσης δίνεται η συνάρτηση f  x   xx 2  2sx  2014, x  R , όπου

πίνακα του (α) είναι: s 2 =

x,s αντιστοίχως η μέση τιμή και η τυπική απόκλιση

του παραπάνω δείγματος με x  0 . α) Να μελετήσετε τη συνάρτηση f ως προς τη μονοτονία και τα ακρότατα β) Αν η f παρουσιάζει ολικό ελάχιστο για x  0.08 να αποδείξετε ότι το δείγμα είναι ομοιογενές και να βρείτε τις τιμές x,s γ) Αν 190 παρατηρήσεις βρίσκονται στο διάστημα  25, 29 να βρείτε το μέγεθος του δείγματος. δ) Αν κάθε παρατήρηση του δείγματος μειωθεί κα-

Από τα δεδομένα έχουμε: s x  s  23 (1)   0,08  s  0.08x (2) . Από x  2

1  x  0.08x  23    x  25  ό (2) s  0.08  25  2 γ) Στο διάστημα  25, 29 περιέχεται το 34%  13,5%  47,5% των παρατηρήσεων άρα αν  είναι το πλήθος των παρατηρήσεων τότε: 47.5 19000 190000    190      400 100 47.5 475 δ) Οι παρατηρήσεις x i , i  1, 2,, 400 γίνονται yi  0.8x i  12 , i  1, 2,, 400 με μέση τιμή

y  0.8x  12  0.8  25  12  8 και τυπική απόκλιsy 1.6 ση sy  0.8s  0.8  2  1.6 ό CVy    0.2  0.1 y 8 και άρα το δείγμα μετά τις μεταβολές δεν είναι ομοιογενές. Άσκηση 4 Ένα δείγμα  ανδρών εξετάστηκε ως προς το σωματικό βάρος σε kgr και τα δεδομένα ομαδοποιήθηκαν σε 7 κλάσεις σύμφωνα με τον παρακάτω πίνακα:

ΕΥΚΛΕΙΔΗΣ Β΄ 90 τ.2/54

------------------------------------------------------------------------------------------------------------------------- Μαθηματικά για την Γ΄ Τάξη ---------------------------------------------------------------------------------------------------------------------Κλάσεις Κεντρικές βάρους τιμές σε kgr xi

, , , , ,  75,  ,



i  i fi % Fi % i x i   x i  x



2

40

77,5 540

Σύνολο

Δίνεται ότι στο κυκλικό διάγραμμα συχνοτήτων και για τις 5 πρώτες κλάσεις αντιστοιχεί τομέας γωνίας 16 324ο. Επίσης: 65  51 , 2  3  3 2   6 και η 13 διάμεσος του δείγματος είναι   70 kgr . α) Να συμπληρώσετε τον πίνακα με τη χρήση υπολογιστή τσέπης β) Να βρείτε τον συντελεστή μεταβλητότητας του δείγματος και να εξετάσετε αν είναι ομοιογενές. Λύση α) Είναι 324o  360o F5  F5  0,9,f 5  F5  F4  0.9  0.775  0.125   Επίσης 65  5v1  6 5  5 1  6f 5  5f1    6 f1   f5  0.15  F1 ,f2  F2  F1  0.4  0.15  0.25. 5 2 3 16  16 Ακόμη 23  32  6  3  2   6    13  13 16 3 2F3  3F2   F6 άρα F3   0.4  0.6 , 13 2 3  13 f3  F3  F2  0.6  0.4  0.2, F6   0.4  0.975, 16 f 6  F6  F5  0.975  0.9  0.075 ,

f 7  F7  F6  1  0.975  0.025 ,

 

f4  1   f1  f2  f3  f4  f5  f6   1  0.825  0.175 .  Επιπλέον  

 5 540   600 άρα 1  f1    90 , F5 0.9

 2  f 2    150 , 3  f 3    120 , 4  f4   0.175 600 105, 5  f5   0.125  600  75 , 6  f6   0.075  600  45, 7  f7   0.025 600 15 . Επειδή F2 %  40, F3 %  60 και οι παρατηρήσεις θεωρούνται ομοιόμορφα κατανεμημένες στις κλάc σεις έχουμε ότι 3  70 άρα 75  70  2c   2 37920 5c 10  c  2 οπότε x   63.2 και ο πίνα600

κας συμπληρώνεται ως εξής: Κλάσεις Κεντρικές βάρους τιμές σε kgr i

 65, 67   67, 69   69, 71   71, 73   73 , 75   75 , 77   77, 79 



i  i fi % Fi % i x i   x i  x

66

90 90

15

15 5940

705.6

68

150240 25

40 10200

3456

70

120360 20

60 8400

5548.8

72

105465 17.5 77.5 3240

8131.2

74

75 540 12.5

76

45 585 7.5 97.5 3420

7372.8

78

15 600 2.5

3285.6 37248

Σύνολο

90 5550 100 1170 37920



2

8748

β) Είναι s2 

 x  1

ν

i

i 1

οπότε CV 



2

 x νi  7.88 63.2

37248 600

 62.08 , s  62.08  7.88

 0.12  0.1 άρα το δείγμα δεν εί-

ναι ομοιογενές. Άσκηση 5 Σε κάθε άτομο ενός δείγματος από  άρρενες μετρήθηκε η συγκέντρωση του ουρικού οξέος σε mg/100ml. Τα δεδομένα ομαδοποιήθηκαν σε κλάσεις δεδομένων. Το πολύγωνο των σχετικών συχνοτήτων f i % έχει διαδοχικές κορυφές τα σημεία:   2.8 , 0  ,   x  , y  ,   x  , y  ,   x  , y   ,

  x  , y  ,   x  , y   ,   x  , y    x  , y   ,   6,0  . Ισχύει ότι: 

x x , η τεταγ-

μένη y ισούται με το τοπικό μέγιστο της συνάρτη5x σης f  x   2 , η τεταγμένη y ισούται με το όριο x 1 5x  10x  15 2

lim 32 x 3

3

x  5  64

, η κορυφή Δ ανήκει στην εφαπτό-

μενη  της Cg με g  x   5ex x στο σημείο στο ο2

ποίο τέμνει τον άξονα yy ,η τεταγμένη y ισούται με τη διάμεσο των 24 πρώτων διαδοχικών όρων της γεωμετρικής προόδου    με 1  5 1011 και

  10 και η μέση τιμή της κατανομής είναι x  4.58 . α) Να κατασκευάσετε το ιστόγραμμα και το πολύγωνο των σχετικών συχνοτήτων f i % β) Τα άτομα με συγκέντρωση ουρικού οξέος πάνω από 5mg/100ml θα κληθούν για περαιτέρω εξετάσεις. Αν το πολύγωνο συχνοτήτων έχει εμβαδό 40 να βρείτε τον αριθμό των ατόμων αυτών Λύση α) Αν c το πλάτος καθεμιάς από τις 9 κλάσεις τότε 2.8  8c  6  c  0.4 άρα αν  το κάτω άκρο της 1ης κλάσης με μηδενική συχνότητα τότε έχου-

ΕΥΚΛΕΙΔΗΣ Β΄ 90 τ.2/55

------------------------------------------------------------------------------------------------------------------------- Μαθηματικά για την Γ΄ Τάξη ----------------------------------------------------------------------------------------------------------------------

    0.8  2.8  2  5.6  0.8    2.4 2 οπότε οι 9 κλάσεις είναι οι εξής:  2.6,3 , 3 ,3.4  , 3.4,3.8  , 3.8, 4.2  ,  4.2, 4.6  ,

με ότι

 4.6,5 , 5,5.4  , 5.4,5.8 , 5.8,6.2 

με αντίστοιχες κεντρικές τιμές 2.8,3.2,3.6, 4, 4.4, 4.8,5.2,5.6,6 .  Η πρώτη και η τελευταία έχουν μηδενική συχνότητα, ενώ οι ενδιάμεσες αφορούν τα δεδομένα του δείγματος και έχουν αντίστοιχες συχνότητες i και σχετικές συχνότητες f i όπου i  1, 2,3, 4,5,6,7 . xR είναι Επίσης για κάθε 2 2 5 x 1  5x  2x 5x2  5 10x2 5 x 1 f  x    2 2 2  x2 1  x2 1  x2 1 άρα f   x   0  x2 1  0  x2  1  x  1 ή x  1  και

f   x   0  x   , 1  1,   , f   x   0

 x   1,1 οπότε η f παρουσιάζει τοπικό μέγι5 στο για x 1 το f  1  και τοπικό ελάχιστο για 2 5 x 1 το f 1  , κατά συνέπεια 2 y  f1 %  f  1  2.5 . Ακόμη

5x 2  10x  15  5  x 2  2x  3  και το τριώνυμο

x 2  2x  3 έχει διακρίνουσα   4  12  16  0 24 24  3 , x2   1 οπότε και ρίζες x1  2 2

lim x3

5  x 2  2x  3  5x 2  10x  15   lim 3 32 3 x  5  64 x  3 32 x52

 5  32 lim x3 

5

lim 32

x  3  x  1



 x  3  x  1 







3

x  5

x5

3

 x  5 2

lim   x  1   x  5  32 5

3



3

3

2

 23 x  5  4

2

 23 x  5  4



 23 x  5  4  

x3



5 4 32



3



64  2  3 8  4 

12  13 5  1011  1011  5  10111012   2 2 5 1  10  55    27.5  y   f 4 % . 2 2 Επιπλέον  x x     0  y z  y  0  

y z  y  f 5 %  f 6 % και f1 %  f 2 %  f 3 %  f 4 %  f 5 %  f 6 %  f 7 %  100  2.5  7.5  15  27.5  2f 5 %  f 7 %  100  f 7 %  47.5  2f 5 % (1) ενώ επίσης ισχύει ότι x  4.58  f1x1  f 2 x 2  f3 x 3  f 4 x 4  f5 x 5  f 6 x 6  f 7 x 7  4.58  f1 %  x1  f 2 %  x 2  f 3 %  x 3  f 4 %  x 4  f 5 %  x 5  f 6 %  x 6  f 7 %  x 7  458  2.5  3.2  7.5  3.6  15  4  27.5  4.4  ... 1

...  4.8  f 5 %  5.2  f 6 %  5.6f 7 %  458 

f5 %  f6 %

 

8  27  60  121  10f 5 %  5.6  47.5  2f 5 %  

 458  216  10f5 %  266  11.2f 5 %  458  1.2f 5 %  24  f 5 %  20  f 6 % και από (1) έχουμε ότι f 7 %  47.5  40  7.5 Το ιστόγραμμα και το πολύγωνο σχετικών % συχνοτήτων είναι το εξής:



 23

x-3

x3





1  5  1011 και λόγο   10 τότε



 

5   4  4  4   7.5  8

 y   f 2 % .  Είναι g  0   5 και επιπλέον για κάθε x  R είναι g  x   51  2x  ex

2

x

, συνεπώς

g  0  5 και η εξίσωση της εφαπτόμενης  της Cg στο σημείο  0,g  0  είναι y  5  5x  y  5x  5

οπότε   y  5x  5  y  5  4  5  15  f3 % . Αν  είναι η διάμεσος των 24 πρώτων της γεωμετρικής προόδου     με πρώτο όρο

 

β) Το εμβαδό πολυγώνου συχνοτήτων είναι ίσο με το μέγεθος του άρα   40 οπότε τα ζητούμενα άτομα είναι  6   7  40  f 6  f 7  

40  0.2  0.075   40  0.275  11 Σχόλιο: Η παρακάτω άσκηση είναι μια παραλλαγή της άσκησης 7 σελ.63 τεύχος 82 του περιοδικού Ευκλείδης Β' που είχε δημιουργηθεί από τους συναδέλφους Μητσιάδη Γιώργο και Ζησόπουλο Γιώργο

Άσκηση 6 Η προϋπηρεσία των εργαζομένων μιας εταιρίας σε έτη έχει ομαδοποιηθεί σε 4 κλάσεις ίσου πλάτους με εύρος R  16 .Η μέση τιμή των κέντρων των κλάσεων είναι ίση με 8 ,η γωνία του κυκλικού διαγράμματος που αντιστοιχεί στις 3 πρώτες κλάσεις

ΕΥΚΛΕΙΔΗΣ Β΄ 90 τ.2/56

------------------------------------------------------------------------------------------------------------------------- Μαθηματικά για την Γ΄ Τάξη ----------------------------------------------------------------------------------------------------------------------

είναι 216ο και για τις αντίστοιχες αθροιστικές συ  χνότητες ισχύει ότι 1  2  3 . 3 6 α) Να βρείτε τα άκρα και τα κέντρα των κλάσεων β) Να βρείτε τις σχετικές συχνότητες των κλάσεων

6,14 

και το ποσοστό των εργαζομένων που έχουν προϋπηρεσία εντός του διαστήματος αυτού f% f% είναι 2  f3 % 4  10%  30%  20%  60% 2 2 ε) Οι νέες σχετικές συχνότητες των κλάσεων που 4 2 θα προκύψουν μετά την απόλυση όσων εργαζόμεγ) Να αποδείξετε ότι S2   x i 2 f i  x όπου s 2 , x νων έχουν λιγότερα από 4 έτη προϋπηρεσίας (δηi 1 η διακύμανση και η μέση τιμή των κλάσεων αντί- λαδή μετά την εξάλειψη της 1ης κλάσης) είναι: i1 στοιχα και x i τα κέντρα των κλάσεων και κατόπιν i1 f f 0.2 2  να βρείτε τους αριθμούς x , s  ν  i1 , i 1,2,3 συνεπώς f1  2  fi   ,  1  f 1  0.1 9 ν-ν 1-f 1 1 1 1 δ) Να βρείτε το ποσοστό των εργαζομένων που 1ν έχουν προϋπηρεσία από x  s έως x  s χρόνια f f 0.3 1  0.4 4 θεωρώντας ότι σε κάθε κλάση οι παρατηρήσεις f2  3  άρα ο  , f3  4   1 f1 1 0.1 3 1  f1 1  0.1 9 κατανέμονται ομοιόμορφα ε) Αν η διοίκηση της εταιρίας απολύσει όσους ερ- νέος μέσος χρόνος προϋπηρεσίας είναι 2 1 4 98 γαζόμενους έχουν προϋπηρεσία λιγότερο από 4 y  x2f1  x3f2  x4f3 6 10 14  , y 10.89 έτη. έτη να βρείτε το νέο μέσο χρόνο προϋπηρεσίας 9 3 9 9 Λύση Άσκηση 7 α) Αν c το πλάτος κάθε κλάσης και x1 , x 2 , x 3 , x 4 Η βαθμολογία (με άριστα το 20) των εξεταζομένων σε ένα τεστ δεξιοτήτων κατά τη διαδικασία R 16 αντίστοιχα τα κέντρα τους τότε c    4 και επιλογής του προσωπικού μιας εταιρίας ομαδο4 4 ποιήθηκε σε 4 κλάσεις. Οι βαθμολογίες έχουν εύx x x x xi  x1  4  i  1 , i  2,3,4 οπότε 1 2 3 4  8 ρος R  8 και τα κέντρα x i των κλάσεων έχουν 4 διάμεσο 16.Η μέση βαθμολογία στο τεστ ήταν  x1  x1  4  x1  8  x1  12  32  4x1  8  x1  2 15,4 και η γωνία του κυκλικού διαγράμματος συοπότε το κάτω και το πάνω άκρο της 1ης κλάσης χνοτήτων που αντιστοιχεί στις δύο πρώτες κλάσεις ο c c αείναι αντίστοιχα τα x1  0,x1   4 άρα οι 4 κλά- είναι 216 . Επίσης ισχύει ότι στο ιστόγραμμα 2 2 θροιστικών συχνοτήτων το ορθογώνιο της 3ης κλά17 σεις αντίστοιχα είναι οι:  0,4 , 4 ,8 ,8,12 ,12,16 του εμβαδού του σης έχει εμβαδό ίσο με τα 20 β) Αν  το μέγεθος του δείγματος τότε: ης 216o  360o  f1  f 2  f 3   f1  f 2  f 3  0.6(1) , ορθογωνίου της 4 κλάσης. α) Να προσδιορίσετε τα άκρα και τις κεντρικές τι     μές των κλάσεων 1  2  3  1  2  3  3 6 3 6  β) Να κατασκευάσετε το ιστόγραμμα και το πολύf f f F F f f γωνο των fi % F1  2  3  f1  1 2  1 2 3 (2) 3 6 3 6 γ) Σύμφωνα με τον κανονισμό της εταιρίας στην Από την σχέση (2) έχουμε ότι f1  f 2  f 3  6f1 και επόμενη φάση της διαδικασίας περνούν όσοι έχουν αντικαθιστώντας στην (1) έχουμε ότι σκορ πάνω από 16/20. Αν στην συγκεκριμένη πε6f1  0.6  f1  0.1 οπότε και πάλι από την σχέση ρίπτωση πέρασαν στην επόμενη φάση 64 άτομα να βρείτε το πλήθος των ατόμων που διαγωνίστηκαν. 0.1  f 2  f 2  0.2 άρα από Λύση (2) έχουμε ότι 0.1  3 R 8 α) Το πλάτος c των κλάσεων είναι c    2 . την (1) έχουμε ότι 0.1  0.2  f 3  0.6  f 3  0.3 . 4 4 Τέλος ισχύει ότι f1  f 2  f 3  f 4  1  Αν x1  x 2  x1  2  x 3  x1  4  x 4  x1  6 εί0.1  0.2  0.3  f 4  1  f 4  0.4 ναι τα αντίστοιχα κέντρα των κλάσεων και  η  2 x  x3 γ) Είναι: s2 xi2fi x όμως x  x1f1  x2f2  x3f3  x4f4   2  διάμεσός τους τότε 2 i1 2x  6  0.1  2  0.2  6  0.3 10  0.4 14  10 16  1  x1 13 οπότε x2  15,x3  17,x4  19 . 2 2 2 2 2 2 s  0.1  2  0.2  6  0.3  10  0.4  14  10  16 2 Το κάτω άκρο της 1ης κλάσης είναι ίσο με οπότε s  16  4 δ) Το διάστημα που μας ενδιαφέρει είναι το ΕΥΚΛΕΙΔΗΣ Β΄ 90 τ.2/57

------------------------------------------------------------------------------------------------------------------------- Μαθηματικά για την Γ΄ Τάξη ----------------------------------------------------------------------------------------------------------------------

c  13  1  12 οπότε οι κλάσεις είναι οι ακό2 λουθες: 12,14  , 14,16  , 16,18  , 18, 20  x1 

β) Είναι: f1x1  f 2 x 2  f 3 x 3  f 4 x 4  x  15.4  13f1  15f 2  17f 3  19f 4  15.4 (1),

216o  360o  f1  f 2   f1  f 2  0.6

(2),

17 17 4  3    20 20  3 17 17   F3   f1  f 2  f 3  0.85  20 20 f1  f 2  f 3  f 4  1 3 

(3),

τών να γίνει ομοιογενές. Δίνεται ότι: 2      t   i   1   s 2    t i2   i 1     i 1      

Λύση α) Για κάθε x  0 ισχύει ότι (…) f   x   lnx  x2  4x  5 . f   x   0  x  1 ή x  5 Το πρόσημο της f  , η μονοτονία και τα ακρότατα της f φαίνονται στον παρακάτω πίνακα:

(4).

 2

 3  0.6  f3  0.85  f3  0.25 .  3

 4   0.85  f 4  1  f 4  0.15 . Συνεπώς  2

1 13f1  15  0.6  f1   17  0.25  19  0.15  15.4

 

  2f1  0.7  f1  0.35 ό f 2  0.6  0.35  0.25  3xi2  xi   Το ιστόγραμμα και το πολύγωνο των σχετικών συ5 β) Είναι: y  5  i 1 χνοτήτων f i % έχει ως εξής: 



x

3 i1 

2 i



 x  5   xi 2  i 1

   xi      1    x i2   i 1    i 1    

 

γ) Έχουμε: 64   f 3  f 4    64  0.4    160 Άσκηση 8. Έστω η συνάρτηση: x3 f  x   1  3ln x   x 2  2ln x  1  5x  ln x  1 9 α) Να μελετήσετε τη συνάρτηση ως προς τη μονοτονία και τα ακρότατα β) Οι παρατηρήσεις x1 ,x2 ,,x έχουν διακύμανση S2x ίση με τη θέση τοπικού ελαχίστου της συνάρτησης f . Οι παρατηρήσεις yi  3xi2  xi , i 1,2,, έχουν μέση τιμή y ίση με τη θέση τοπικού μεγίστου της συνάρτησης f . Να βρείτε το συντελεστή μεταβολής των παρατηρήσεων x1 ,x2 ,,x αν xi  0 για κάθε   1,2,,  . Είναι το δείγμα των παρατηρήσεων αυτών ομοιογενές; γ) Να βρείτε τη μικρότερη τιμή της θετικής σταθεράς  που πρέπει να προσθέσουμε σε κάθε παρατήρηση x i ώστε το δείγμα των παρατηρήσεων αυ-

1







 x 5 3

  x

2

2

 (1) και

 x 5 3

    1   



x i 1

2 i

S2x  1

 x   

2



2

   0  x  5  3x  3  0 

1 5 1 5 2 1 , x   6 6 3 από τις οποίες η δεύτερη απορρίπτεται διότι x  0 εφ’όσον x i  0 για κάθε   1,2,,  . Κατά συνέS πεια x  1 , Sx  1 οπότε CVx  x  1  0.1 οπότε το x δείγμα των παρατηρήσεων x1 ,x2 ,,x δεν είναι ομοιογενές. zi  xi  ό   0 γ) Ακόμη έχουμε: 2

3x  x  2  0 με ρίζες x 

 i 1,2,....,ν άρα z  x      1 , Sz  Sx  1 οπότε το δείγμα των παρατηρήσεων z1 , z 2 ,, z  είναι ομοιογενές όταν και μόνο όταν: 1 1 10     9 . Κατά συνέπεια η CVz  0.1    1 10 μικρότερη τιμή της σταθεράς  που πρέπει να προσθέσουμε σε κάθε παρατήρηση x i ώστε το δείγμα των παρατηρήσεων αυτών να γίνει ομοιογενές είναι 9 .

ΕΥΚΛΕΙΔΗΣ Β΄ 90 τ.2/58

Γ΄ΛΥΚΕΙΟΥ

Όριο, συνέχεια, παράγωγος Αντωνόπουλος Νίκος, Αργυράκης Δημήτρης, Λουριδάς Γιάννης

ΘΕΜΑ 1 Έστω f :[0, + )   μια συνεχής συνάρτηση η οποία για κάθε x  0 ικανοποιεί την ι-



ρεί να υπολογισθεί και από την ισότητα (2). β. Η συνάρτηση f είναι συνεχής στο διάστημα [0,  ) και παραγωγίσιμη στο (0,  ) με



σότητα e-x 2f(x) + e-x = x - f 2 (x) , (1) και επιπλέον ισχύει e2f(2)+1>0. α. Να αποδείξετε ότι η συνάρτηση φ :[0, + )   , με φ(x) = f(x) + e-x , για κάθε x  0 , διατηρεί σταθερό πρόσημο στο διάστημα (0, +  ) και ότι

f (x) = x  e  x , για κάθε x  0 . β. Να μελετήσετε τη συνάρτηση f ως προς τη μονοτονία και να αποδείξετε ότι: 6f(7) + 7f(6) i. f(13) > 13 ii. Η εξίσωση f(x)=0 έχει μοναδική ρίζα στο διάστημα (0, 1). γ. Έστω ότι g, h:    είναι δυο συναρτήσεις ώστε για κάθε x  0 να ισχύει g(h(x)) = 2f(x) + h(x) . Να αποδείξετε ότι: i. Η συνάρτηση h αντιστρέφεται. ii. Η γραφική παράσταση της g, έχει κοινό σημείο με την ευθεία y=x. Λύση α. Για κάθε x  0 έχουμε:

(1)  2f(x)e-x + e-2x = x - f 2 (x)  f 2 (x)  2f(x)e x  e2x  x  (f(x)  e  x ) 2  x  (φ(x)) 2  x , (2) Αν θεωρήσουμε ότι η φ δεν διατηρεί σταθερό πρόσημο στο διάστημα (0,  ) , τότε ως συνεχής θα έχει σημείο μηδενισμού. Αυτό σημαίνει ότι υπάρχει xo>0 ώστε φ(xo)=0. Με x=xo η (2) οδηγεί στο άτοπο 0  x o . Επομένως, η φ διατηρεί σταθερό πρόσημο στο (0, ) . Επιπλέον, 1 φ(2)  f(2)  e2  2  e2 φ(2)  0 οπότε φ(x)>0 για e κάθε x>0. Έτσι, για κάθε x>0 έχουμε:

φ(x)  x  f (x)  ex  x  f (x)  x  e x Επιπλέον, η f είναι συνεχής στο μηδέν, οπότε

f (0)  lim f (x)  1 x0

Άρα, για κάθε x  0 είναι f (x)  x  e x που είναι το ζητούμενο. Σημείωση: Προφανώς η τιμή της f στο xo=0 μπο-

f (x) 

1 2 x

 e x

Επειδή f (x)  0 για κάθε x  (0,  ) , η συνάρτηση είναι γνησίως αύξουσα στο πεδίο ορισμού της. i. Από την μονοτονία της f προκύπτει ότι: f (13)  f(7) και f(13)>f(6), Οπότε 6f(13)>6f(7) και 7f(13)>7f(6) απ’ όπου έχουμε: 6 f (7)  7 f(6) 13f (13)  6 f (7)  7 f (6)  f(13)  13 που είναι το ζητούμενο. ii. Η f είναι συνεχής στο διάστημα [0, 1] και 1 ισχύουν: f(0)= -1<0 και f(1)=1- >0 e οπότε απ’ το θ. Bolzano συνάγουμε ότι η εξίσωση f(x)=0 έχει (πραγματική) ρίζα ρ που περιέχεται στο διάστημα (0, 1). Η μοναδικότητα της προκύπτει από τη γνήσια μονοτονία της f. γ. i. Για κάθε x1, x2  [0,  ) με h(x1)=h(x2) έχουμε: h(x1)=h(x2)g(h(x1))=g(h(x2))  2f(x1)+h(x1)=2f(x2)+h(x2)  2f(x1)=2f(x2)  f(x1)=f(x2)  x1=x2 διότι η f ως γνησίως μονότονη είναι «1-1». Άρα, η συνάρτηση h αντιστρέφεται. ii. Για την ρίζα ρ της εξίσωσης f(x)=0 του ερωτήματος (βii), έχουμε: g(h(ρ))=2f(ρ)+h(ρ)  g(h(ρ))=h(ρ), απ’ όπου συνάγουμε ότι ο αριθμός α=h(ρ) είναι ρίζα της εξίσωσης g(x)=x. Έτσι, το σημείο M(α, g(α)) με τετμημένη α=h(ρ) είναι κοινό σημείο της Cg με την ευθεία y=x. ΘΕΜΑ 2 Έστω f:    μια συνάρτηση η οποία για κάθε x   ικανοποιεί τη σχέση f 3 (x) + 2f(x) = x + 1 (1) α. Να αποδείξετε ότι η f είναι γνησίως αύξουσα. β. Να βρείτε την αντίστροφη της f. γ. Να αποδείξετε ότι η f έχει όριο στο xo=2 και να το προσδιορίσετε. δ. Να αποδείξετε ότι η εξίσωση f -1 (x) = 0 έχει μοναδική λύση και αυτή περιέχεται στο διά1 1 στημα  ,  . 3 2

ΕΥΚΛΕΙΔΗΣ Β΄ 90 τ.2/59

------------------------------------------------------------------------------------------------------------------------- Μαθηματικά για την Γ΄ Τάξη ---------------------------------------------------------------------------------------------------------------------

ε. Αν για τους πραγματικούς αριθμούς α, β με α 2 - αβ + β 2 α = να αποδείξετε αβ  0 ισχύει 2 α + αβ + β 2 β 1 α 1 ότι < < . 3 β 2 Λύση α. Έστω ότι η συνάρτηση f δεν είναι γνησίως αύξουσα. Αυτό σημαίνει ότι υπάρχουν πραγματικοί x1
f 3 (x1 )  f 3 (x 2 ) και 2f (x1 )  2f(x 2 ) , οπότε f 3 (x1 )  2f(x1 )  f 3 (x 2 )  2f(x 2 )  x1  1  x 2  1  x1  x 2 άτοπο. Άρα, η f είναι γνησίως αύξουσα στο  . β. Η f ως γνησίως μονότονη είναι «1-1» οπότε αντιστρέφεται. Προκειμένου να βρούμε την f-1 απαιτείται πρώτα ο προσδιορισμός του συνόλου τιμών της f, που θα είναι το πεδίο ορισμού της αντίστροφης. Θα αποδείξουμε ότι το σύνολο τιμών της f είναι το  . Προς τούτο, αρκεί να δείξουμε ότι για κάθε y   , η εξίσωση f(x)=y, έχει λύση στο  . Έστω ότι υπάρχει x΄  ώστε f(x΄)=y (2). Αν x=x΄, τότε: 1  f 3(x΄)  2f(x΄)  x΄1 (3),  2

οπότε (3) y  2 y  x΄  1  x΄  y  2y  1 (Μοναδική πιθανή ρίζα). Θα αποδείξουμε ότι για x΄  y3  2y  1 , ισχύει f(x΄)=y. Πράγματι έχουμε 3

3

λόγω της (1): f 3 (x΄)  2f(x΄)  x΄  1

f 3(x΄)  2f(x΄)  y3  2y g(f(x΄))  g(y) f(x΄)  y όπου g(x)=x3+2x, συνάρτηση που προφανώς είναι «1-1». Αν θέσουμε f-1(x) στο x, η δοσμένη ισότητα γίνεται: [f(f 1 (x))]3  2f (f 1 (x))  f 1 (x)  1 1

 f (x)  x  2 x  1, για κάθε x   . 3

γ. Για κάθε x κοντά στο xo=2, έχουμε:

f 3 (x)  2f(x)  x  1  f 3 (x)  2f(x)  3  x  2  (f(x)  1)(f 2 (x)  f(x)  3)  x  2 | f(x)  1|| f 2 (x)  f(x)  3|| x  2 | | x 2 | | f(x)  1| 2 | x  2 | f (x)  f(x)  3

(διότι f2(x)+f(x)+3>1)   | x  2| f(x) 1 | x  2|

  | x  2 | 1  f(x) | x  2 | 1 και lim( | x  2 | 1)  1  lim(| x  2 | 1) οπότε, x 2

x 2

από το κριτήριο παρεμβολής, συμπεραίνουμε υπάρχει το όριο της f στο xο=2 και limf(x)  1 x 2

δ. Η f-1 είναι συνεχής στο  ως πολυωνυμική,

1 1  οπότε είναι συνεχής και στο διάστημα  ,  . 3 2 1 1 2 1  18  27 8 Επιπλέον f 1 ( )   1   0 3 27 3 27 27 1 1 2 1 f 1 ( )    1   0 2 8 2 8 οπότε, απ’ το θ. Bolzano συμπεραίνουμε ότι η εξίσωση f -1(x)=0 έχει, τουλάχιστον μια, λύση στο 1 1  1 2  ,  . Επίσης f (x)  3x  2  0 , για κάθε 3 2   x   οπότε η συνάρτηση f 1 είναι γνησίως αύξουσα στο  και κατά συνέπεια η λύση που πε1 1 ριέχεται στο διάστημα  ,  είναι η μοναδική 3 2 πραγματική λύση της εξίσωσης. α2 α  1 α 2 - αβ + β 2 α α β2 β δ. Έχουμε: 2  =  2 2 α α α + αβ + β β β  1 2 β β





α t 2 β



t  t 1  t  t3  t 2  t  t 2  t 1  0 t2  t 1

1 1   διότι ο αριθμός t είναι η (μοναδική) ρίζα της εξί1 α 1 που σωσης του (γ) ερωτήματος. Άρα, < < 3 β 2 είναι το ζητούμενο. ΘΕΜΑ 3 Έστω f: μια συνεχής συνάρτηση, με

 t 3  2t  1  0  f 1 (t)  0  t   ,  3 2

f(x)  0, για κάθε x   . α. Αν για κάθε x[0, 1] ισχύει f(x)[0, 1] και επx πf(x) να αποδείξετε ότι: πιπλέον f(συν ) = συν 2 2 πx - x είναι γνησίως i. Η συνάρτηση g(x)= συν 2 φθίνουσα στο [0, 1] πx ii. Υπάρχει μοναδικό xo(0, 1) ώστε συν o =xo 2 iii. f(xo)=x0 β. Να αποδείξετε ότι για οποιοδήποτε α>0 και 1 β(0, 1), υπάρχει ξ[β, ] ώστε β 1 1 2f(ξ)  αf( ) + f(β) β α Λύση α. i. Για κάθε x1, x2[0, 1] έχουμε:

ΕΥΚΛΕΙΔΗΣ Β΄ 90 τ.2/60

------------------------------------------------------------------------------------------------------------------------- Μαθηματικά για την Γ΄ Τάξη ---------------------------------------------------------------------------------------------------------------------

πx1 πx2 π πx πx     1   2 2 2 2 2 2 και - x1>- x2 οπότε πx πx  1  x1   2  x 2  g(x1 )  g(x2 ) 2 2 Άρα η g είναι γνησίως φθίνουσα στο [0, 1]. ii. Αρκεί η g να έχει μοναδική ρίζα (0, 1). Πράγματι: η g είναι συνεχής στο [0, 1] και  g(0)  1  0  1  g(1)  0  1  1 οπότε εφαρμόζεται γι’ αυτήν το Θ. Bolzano στο [0,1]. Άρα, υπάρχει xo(0, 1), το οποίο, λόγω της μονοτονίας της g είναι μοναδικό, ώστε g(xo)=0. πf(x o ) πx   iii. Έχουμε: συν = f  συν o   f (x o ) 2 2   Αυτό σημαίνει ότι ο αριθμός f(xo) είναι ρίζα της πx = x η οποία όπως προεξίσωσης συν 2 αναφέραμε έχει μοναδική λύση τον αριθμό xo. Άρα, f(xo)=xo. 1 ] οπότε λαμβάνει β. Η f είναι συνεχής στο [β, β σ’ αυτό μέγιστη και ελάχιστη τιμή. Αν M, m είναι η μέγιστη και η ελάχιστη τιμή αντίστοιχα, τότε 1 έχουμε: m  f ( )   και m  f ()    1 οπότε α  m  α  f ( )  α    1 1 1 και  m   f ()    α α α 1 1  1  1 Άρα,  α+   m  α  f( )+  f(β)   α+    β α  α  α 1 1 α  f( )+  f(β) β α  m  1 α+ α Από τα παραπάνω προκύπτει ότι ο αριθμός 1 1 α  f( )+  f(β) β α 1 α+ α περιέχεται στο σύνολο τιμών της f, οπότε υπάρχει 1 1 α  f( )+  f(β) 1 β α (1). ξ[β, ] ώστε f ()  1 β α+ α 2 Από την προφανή σχέση α +1  2α που ισχύει για x1  x2  0 

1 1 1  κάθε α   , με α>0 έχουμε α+  2  1 α 2 α+ α 1 1 αf( )+ f(β) β α οπότε (1)  f ()  2 1 1  2f(ξ)  αf( )+ f(β) που είναι το ζητούμενο. β α ΘΕΜΑ 4 x

Δίνεται η συνάρτηση f (x) = x + e - 1 , x   . α. Να αποδείξετε ότι η f αντιστρέφεται. x

β. Να λύσετε την εξίσωση e = 1 - x . γ. Να λύσετε την ανίσωση f

-1

f(x) - x + 1 > 1

δ. Θεωρούμε τη συνεχή και γνησίως αύξουσα συνάρτηση g :    , της οποίας η γραφική παράσταση Cg δι-έρχεται από την αρχή των αξόνων. Να αποδείξετε ότι η εξίσωση:

(f  g)(x) - f (1 - x

2013

)=0

(Ι)

έχει μία τουλάχιστον λύση στο (0 , 1) Λύση α. Για τυχαία x1 , x 2  R με x1  x 2 έχουμε

e

x1

e

x2

e

x1

 x1  e

και με πρόσθεση κατά μέλη προκύπτει x2

 x2 .

x

Επομένως e 1  x1    e

x2

 x 2  ,

δηλαδή f (x1 )  f (x 2 ) . Άρα η f είναι γνησίως αύξουσα στο  , οπότε είναι και «1–1», δηλαδή αντιστρέφεται. x

x

β. Είναι: e =1-x  e +x-1=0

f(x)=0 f(x)=f (0)  x=0, (1) αφού η f είναι «1–1». γ. Αφού η f είναι γνησίως αύξουσα έχουμε:  

f

1

 f (x)-x+1 >1  f  f -1  f (x)-x+1 >f (1)  

 f (x)  x  1  f (1)  f (x)  x  1  e x

x

 x  e 1 x 1  e  e  e  x 1 δ. Είναι:

 I   f (g(x))=f (1-x2013 ) 

g(x)  1  x

2013

 g(x)  x

2013

1  0

επειδή η f είναι «1–1». Θεωρούμε τη συνάρτηση 2013

h(x)  g(x)  x  1, x   Για τη συνάρτηση h ισχύουν: Η h είναι συνεχής στο  0 , 1 , ως αποτέλεσμα

ΕΥΚΛΕΙΔΗΣ Β΄ 90 τ.2/61

------------------------------------------------------------------------------------------------------------------------- Μαθηματικά για την Γ΄ Τάξη ---------------------------------------------------------------------------------------------------------------------

πράξεων συνεχών συναρτήσεων. h(0)  g(0)  1  1  0 , ( g(0)  0 , αφού η Cg διέρχεται από την αρχή των αξόνων) και

h (1)  g(1)  1  1  g(1)  g(0)  0

(α-

φού η συνάρτηση g είναι γνησίως αύξουσα). Ικανοποιούνται λοιπόν οι προϋποθέσεις του Θεωρήματος Bolzano. Άρα η εξίσωση h(x)  0 , οπότε και η ισοδυναμή της (Ι) έχει μία τουλάχιστον ρίζα στο  0 , 1 . ΘΕΜΑ 5 Θεωρούμε τις συναρτήσεις f ,g :    για τις οποίες ισχύουν τα παρακάτω:  Έχουν σύνολο τιμών το  ,

 x  f  f (x)   f (x),  ά x   (1) , και   g  f (x)    g  f (x)   x,  ά x   (2) 3

α. Να βρείτε τον τύπο της συνάρτησης f β. Να αποδείξετε ότι η g αντιστρέφεται και να βρείτε την g 1 . γ. Να λύσετε την ανίσωση g(x)  x  1, x   δ. Αν α, β   με β>0 και α+β+2<0, τότε να αποδείξετε ότι η εξίσωση g g 1 (x)  x    2  1 έχει ακριβώς τρεις





πραγματικές ρίζες. Λύση α. Από την (1) για κάθε x έχουμε: x  f(x) και

f  f(x)  f(x) . Έτσι έχουμε f(y)  y,  ά y  , διότι f ()   . Άρα f (x)  x,  ά x  (3) β. Από (2), (3) έχουμε:

g  x 

3

 g  x   x,  ά x  (4)

Για κάθε x1 , x 2  έχουμε: g(x1 )  g(x 2 )   g(x1 )    g(x 2 )  3

3 (4)

  g(x1 )   g(x1 )   g(x 2 )   g(x 2 )  x1  x 2 3

3

Άρα η g είναι συνάρτηση 1-1, οπότε αντιστρέφεται και το πεδίο ορισμού της g 1 είναι το g()   . Για να βρούμε τον τύπο της g 1 θέτουμε στην (4), όπου x το g 1 (x) και έχουμε:

g g

1



(x)   g  g 1 (x)   g 1 (x) 3

 g 1 (x)  x 3  x,  ά x  για κάθε x1 , x 2  έχουμε: x1  x2 x  x  x13  x1  x 23  x 2  g 1  x1   g 1  x 2 

[Διαφορετικά, διότι

 g  x   ΄  3x 1

2

g 1 

g(x)  x  1  g 1  g(x)   g 1  x  1  x  (x  1)3  x  1  (x  1)3  1  (x  1)3  (1)3  x  1  1  x  2

δ. Είναι: g  g 1 (x)  x    2   1

 g1 (x)  x  2  g1 (1)  x3  x  x  β  2  2  x3  (1  )x  β  0 (5) Θεωρούμε τη συνάρτηση h(x)  x3  (1  )x  β, x  Είναι:

im h(x)  im  x 3  (1  )x  β   im x 3  

x 

x 

x 

Άρα υπάρχει γ<0 τέτοιο, ώστε h(  )  0 Επίσης, h(0)    0 , από υπόθεση, h(1)      2  0 , από υπόθεση και

im h(x)  im  x 3  (1  )x  β   im x 3  

x 

x 

x 

Άρα υπάρχει δ>1 τέτοιο, ώστε h()  0 Η h ως πολυωνυμική είναι συνεχής στα διαστήματα [γ, 0], [0, 1], [1, δ] και είναι h(  )  h(0)  0 , h(0)  h(1)  0 και h(1)  h()  0 Δηλαδή, η συνάρτηση h ικανοποιεί τις προϋποθέσεις του θεωρήματος Bolzano στα διαστήματα [γ, 0], [0, 1], [1, δ], οπότε η εξίσωση h(x)  0 έχει τρεις τουλάχιστον ρίζες, μία τουλάχιστον σε καθένα από τα διαστήματα (γ,0)  (,0) , (0,1), (1,δ)  (1,  ) και επειδή η εξίσωση h(x)  0 έχει τρεις το πολύ πραγματικές ρίζες, ως πολυωνυμική εξίσωση τρίτου βαθμού, θα έχει τελικά ακριβώς τρεις πραγματικές ρίζες. Λόγω της (5) θα έχει ακριβώς τρεις πραγματικές ρίζες και η εξίσωση g  g 1 (x)  x    2   1 , ως ισοδύναμη της h(x)  0 ΘΕΜΑ 6 Έστω συνάρτηση f : (0,   )   για την οποία ισχύει: x  2  f (x)  (x  1)(x  nx)  1, για κάθε x  (0,   ), (1) (με την αριστερή ισότητα να ισχύει μόνο για μια τιμή του x). α. Να βρείτε την εξίσωση της εφαπτομένης της γραφικής παράστασης της f στο σημείο (1,f(1)) β. Να βρείτε τα όρια: lim g  x  ,lim h  x  , όπου x 1

γ. Η συνάρτηση g 1 είναι γνησίως αύξουσα, διότι 3 1

κάθε x  ] Είναι:

3 2

 1  0, για

g  x 

h  x 

ΕΥΚΛΕΙΔΗΣ Β΄ 90 τ.2/62

x 1

xf (x)  x  3  1 , και x-1   f (x)  1 2

2

 x-1  f 2 (x)  x 2  4x  4 

------------------------------------------------------------------------------------------------------------------------- Μαθηματικά για την Γ΄ Τάξη ---------------------------------------------------------------------------------------------------------------------

γ. Αν επιπλέον η f είναι συνεχής, τότε να αποδείξετε ότι υπάρχει ένα τουλάχιστον 3 x0 (1,3) τέτοιο, ώστε  2f(x0 )  1   nx0  2 Λύση α. Από την (1) για x=1 έχουμε: 1  f (1)  1  f (1)  1 (2) Εξετάζουμε αν η f είναι παραγωγίσιμη στο x 0  1 (με τον ορισμό της παραγώγου). Για 0< x  1 είναι f (x)  f (1) (2) f (x)  1  (3) x 1 x 1  Αν x>1, τότε: 1  x  1  f (x)  1  (x  1)(x  nx) f(x) 1  1  x  nx, και lim1  1  lim(x  nx) . x 1 x 1 x 1 Άρα σύμφωνα με το κριτήριο παρεμβολής είναι f (x)  1 1 lim x 1 x 1  Αν 0
f (x)  1 f (x)  1  1  x  nx  x  nx   1, x 1 x 1 και lim(x  nx)  1  lim1 . Άρα σύμφωνα με το   :(x 1)0

x 1

x 1

κριτήριο παρεμβολής είναι lim x1

f(x) 1 1 Ισχύει x 1

f (x)  1 f (x)  1  lim  1 , οπότε x 1 x 1 x 1 x 1 (3) f (x)  1 f (x)  f (1) lim  1 lim  1 . Επομένως, η x 1 x  1 x 1 x 1 f είναι παραγωγίσιμη στο x 0  1 , με f ΄(1)=1 (4)  Η εξίσωση της εφαπτομένης της γραφικής παράστασης της f στο σημείο (1,f(1)) είναι: y  f (1)  f ΄(1) (x  1), δηλαδή y  (1)  1  (x  1) και τελικά y  x  2 . lim

β. Για να βρούμε τα lim g  x  ,lim h  x  , εργαζόx 1

x 1

μαστε με τον ορισμό της παραγώγου της f στο ση0 μείο x 0  1 , διότι έχουμε μορφή και η f είναι 0 παραγωγίσιμη στο x 0  1 . Για x>0 και κοντά στο 1 έχουμε:



gx  (2)





x  f 2 (x)  f 2 (1)   xf 2 (1)  x 2  3  1

x  f (x)  1   x  1  x 2  3

x  f 2 (x)  1 x-1

x-1



 x  1 

x2  3

x 1







x2  3



2

=x

f (x)  1 (f (x)  1) + = x-1  x  1 x  1  x 2  3

=x

f (x)  1 2(x  1) (f (x)  1) + = x-1  x  1 x  1  x 2  3









f (x)  1 2 (f (x)  1) + x-1 x  1  x2  3 2 1 3 Άρα: limg x 1 f ΄(1)  (f(1)-1)+  -2+   επειδή x1 4 2 2

=x

(2)

f ΄(1)=1 και limf (x)  f (1)   1 , (5) αφού η f είναι x 1

συνεχής στο x 0  1 , ως παραγωγίσιμη σε αυτό. 

hx 

  f (x)  1 x 1



1  f (x)  (x  2) 2 2

  f (x)  1 f (x)  1 1 1    f(x)  1 x  1 f (x)   x  2 f (x)   x  2

lim

Όμως έχουμε:

  f (x)  1

x 1

f(x)  1

 lim u 0

(5)

u  1, u (2)

όπου u  f (x)  1 μ lim(f (x)  1)  f (1)  1  0 και x 1

(4) f (x)  1 lim  f ΄(1)  1 x 1 x  1 (5) (2) 1 1 1   Επίσης lim x 1 f (x)   x  2  f (1)  1  2 2

και lim x 1

1   , επειδή f (x)  (x  2) (5)

(2)

lim  f (x)  (x  2)   f (1)  (1  2)  0 x 1

 f (x)  (x  2)  0 , αφού για κάθε x κοντά στο x 0  1 , λόγω της (1) είναι: f  x   x  2, με το ίσον μόνο για x=1.  1 Άρα: lim h  x   1  1          x 1  2 γ. Θεωρούμε τη συνάρτηση 3 (x)   2f (x)  1   nx  2, x   0,   . Η φ είναι συνεχής στο κλειστό διάστημα [1,3], ως αποτέλεσμα πράξεων συνεχών συναρτήσεων και (2)

(1)   2f (1)  1   n1  2  (3)3  2  29  0 3

(3)   2f(3) 1   n3  2  2f(3) 1  1  n3 1  0 επειδή από την (1) για x=3 έχουμε f (3)  1  2f (3)  2  2f (3)  1  1 3

2

x-1

(x  1) 2 

3

  2f (3)  1  1   2f (3)  1  1  0 3

ΕΥΚΛΕΙΔΗΣ Β΄ 90 τ.2/63

3

------------------------------------------------------------------------------------------------------------------------- Μαθηματικά για την Γ΄ Τάξη --------------------------------------------------------------------------------------------------------------------nx 

και 3  e  n3  ne  n3  1  n3  1  0 Δηλαδή, η συνάρτηση φ ικανοποιεί τις προϋποθέσεις του θεωρήματος Bolzano στο [1,3]. Άρα υπάρχει ένα τουλάχιστον x 0  (1,3) τέτοιο, ώστε

(x 0 )  0 , δηλαδή  2f (x 0 )  1   nx 0  2 3

ΘΕΜΑ 7 Έστω συνάρτηση f : (0,   )   για την οποία ισχύουν οι σχέσεις:

 f (x  y)  f (x)  f (y) ,  ά x, y  (0,  ) (1)  f (x)  0,  ά x  (0,   ) (2), και  f (x)  1,  ά x  (1,   ) (3) Α. Να αποδείξετε ότι: 1  1 α. f (1)  1 και f    ,  ά x (0, )  x  f(x) β. f (x)  0,  ά x  (0,   ) γ. Η f είναι γνησίως φθίνουσα. 1 δ. Η εξίσωση f (e  x  2 )  f ( )  f (x 2  2x) , έ4x χει ακριβώς μία θετική πραγματική ρίζα. Β. Αν επιπλέον η f είναι παραγωγίσιμη στο x0  1, τότε δείξετε ότι η f είναι παραγωγίσιμη με f(x) f ΄(x)  f΄(1)  ,  ά x  (0,  ) x Λύση (2)

Α. α. Αν x=y=1, τότε: 1 f(1)  f(1)  f(1)f(1) 1 1 1 1  x  0 , τότε: 1  f(x  )  f(x)  f( ) x x x f (1) 1 1 1  f (1)  f (x)  f ( )  f (x)  f ( )  1 x x (2) 1 1 f ( )  ,  ά x  (0,  ) x f (x) β. Αν θέσουμε στην (1) όπου x, y το x  x  0 , τότε: 1  f ( x  x ) 

Αν y 

 f ( x)  f( x)  f  x   f 2 ( x)  ά x (0, ) . Άρα f  x   0 και λόγω της (2) f (x)  0 , οπότε f (x)  0 ,  ά x  (0,  ) γ. Για κάθε x1 , x 2  (0,  ) έχουμε: :x1 0

x1  x 2 

(3)  1  (1) x2 x   1 f  2   1  f   x 2   1 x1  x1   x1 

) ) 1 1 f   f  x2   1  f  x2   1f  x1   f  x2  f (x1 )  x1  Άρα η συνάρτηση f είναι γνησίως φθίνουσα. δ. Το πεδίο ορισμού της εξίσωσης είναι το

A  (0,  ) . Στο Α λοιπόν έχουμε:  1  f (e x  2 )  f    f (x 2  2x)  4x 

(1)  x 2  2x   x 2  x 2  f (e x  2 )  f    f (e )  f    4   4x  f "11" x2  e x  2   4e x 2  x  2  0 (4) (*) 4 Αν g(x)  4e x  2  x  2, x   0,   , ό :

 4   g(x)  g(2)  x  2,

διότι η g είναι 1-1 ως

γνησίως φθίνουσα, επειδή g΄(x) 4ex2 1 0, για 1 κάθε x>0. Άρα η εξίσωση f(ex2)f( )f(x2 2x) , 4x έχει ακριβώς μία θετική πραγματική ρίζα το 2. (*) Σημείωση: Διαφορετικά θα μπορούσαμε να πούμε ότι η εξίσωση (4) έχει προφανή ρίζα στο (0, ) το 2 που είναι και μοναδική, διότι η συνάρτηση g με g(x)  4ex2 x2, x(0,) είναι γνησίως φθίνουσα (…)

Β. Η συνάρτηση f είναι παραγωγίσιμη στο f (x)  f (1) x 0  1 , οπότε υπάρχει το όριο lim και x 1 x 1 είναι πραγματικός αριθμός. Είναι f (x)  1 (5), διότι f(1)=1. f ΄(1)  im x 1 x 1 Έστω x 0  (0,  ) . Για x>0 και κοντά στο f(x)f(x0) . x 0 θεωρούμε το λόγο μεταβολών  x  xx0 x Θέτουμε: h  , δηλαδή x  x0  h, όπου προφαx0

νώς 0  h  1, αφού x, x 0   0,   και x  x 0 . Έτσι έχουμε:   x   (1)



f (x 0  h)  f (x 0 )  x0  h  x0

f(x0 )  f(h)  f(x0 ) f(x0 )   f(h) 1 f(x0 ) f(h) 1    x0  h  x0 x0   h 1 x0 h 1

Άρα  f (x0 ) f (h)  1  (5) f (x0 ) lim   x   lim    f ΄(1)   h 1 x x0 h 1  x0  x0 Δηλαδή, η συνάρτηση f είναι παραγωγίσιμη στο f (x 0 )  f ΄(1) τυχαίο x 0  (0,  ) με f ΄(x 0 )  x0 Επομένως η συνάρτηση f είναι παραγωγίσιμη με f (x) για κάθε x  (0,  ) . f ΄(x)   f ΄(1) , x

ΕΥΚΛΕΙΔΗΣ Β΄ 90 τ.2/64

------------------------------------------------------------------------------------------------------------------------- Μαθηματικά για την Γ΄ Τάξη ---------------------------------------------------------------------------------------------------------------------

Σχόλιο της Συντακτικής επιτροπής: Θεωρούμε αναγκαίο να επισημάνουμε τρεις βασικές παρανοήσεις που έχουμε παρατηρήσει ότι προκύπτουν κατά την αναζήτηση ενός ορίου. Α) Επικρατεί (κακώς) η άποψη ότι για να βρούμε ή για να ελέγξουμε αν έχει νόημα ένα όριο lim f (x) , πρέπει αρχικά να βρούμε το x x0

σύνολο ορισμού της συνάρτησης f(x), ως διάστημα ή ένωση διαστημάτων. Όμως εκτός του ότι αυτό δεν είναι πάντα εφικτό, δεν είναι και απαραίτητο. Αν π.χ. μας ζητηθεί το limf (x) x 1

με f(x) = x  2x  10 , τότε το σύνολο ορισμού της Α={xR/ x5 -2x+10≥0} δεν μπορεί να γραφεί με μορφή διαστήματος ή ένωσης διαστημάτων. Ωστόσο μπορούμε να διαπιστώσουμε ότι έχει νόημα αυτό το όριο, δηλαδή ότι η συνάρτηση ορίζεται κοντά στο x0=1 και να το βρούμε. Πράγματι, αφού lim ( x5 -2x+10)=9>0, θα είναι x5 -2x+10>0 5

x 1

κοντά στο x0=1. Άρα η f(x) ορίζεται κοντά στο x0=1. Εξάλλου limf (x) = 9 =3. x 1

Β) Συνήθως για να βρούμε ένα όριο lim f (x) x x0

που έχει απροσδιόριστη μορφή, φροντίζουμε να δώσουμε στη συνάρτηση f(x), διαδοχικές μορφές f1(x), f2(x) για x κοντά στο x0 μέχρις ότου καταλήξουμε σε μια μορφή για την οποία μπορούμε να αποφανθούμε αν έχει ή όχι όριο. Έστω π.χ. την f2(x). Συνηθίζεται τότε να γράφεται (κακώς) lim f (x) = lim f1 (x) = lim f 2 (x) . Αυτό όμως είx x0

xx0

x x0

ναι απαράδεκτο διότι πιθανόν να μην υπάρχει lim f 2 (x) , πράγμα που δεν γνωρίζουμε εκ των x x0

προτέρων. 3x 2  2x  1 και ζητάμε να x 3  3x  2 βρούμε το limf (x) , τότε είναι παράλογο να

Π.χ. Αν f(x)= x 1

γράψουμε: limf (x) = lim x 1

x 1

(x  1)(3x  1) 3x  1 = lim 2 = 2 x  1 (x  1)(x  x  2) x x2

3x  1  3x  1 1   = lim   , αφού x 1 (x  1)(x  2) x 1  x  2 x 1 1 προφανώς δεν υπάρχει lim . x 1 x  1

= lim

Κανονικά πρέπει να εκφραζόμαστε ως εξής: για x κοντά στο x0=1 έχουμε:

(x  1)(3x  1) 3x  1 = 2 = 2 x x2 (x  1)(x  x  2) 3x  1 3x  1 1  = . Όμως x  2 x 1 (x  1)(x  2) 3x  1 4 1 lim = και lim = +  ενώ x 1 x  2 x  1 3 x 1 1 lim = - , οπότε x 1 x  1 4 4 lim f(x)= (+)=+ και lim f(x)= (-)=x 1 x 1 3 3 . Άρα δεν υπάρχει lim f (x) .

f(x)=

x 1

Γ) Σχετικά με τον κανόνα De L΄ Hospital, επικρατεί (κακώς) η άποψη, ότι αν το f (x) έχει μια από τις απροσδιόριστες g(x) 0     μορφές , , , , , τότε: 0     f (x) f΄(x) lim = lim . Όμως το σωστό είναι x  x 0 g(x) x  x 0 g΄(x) f΄(x) ότι: Αν lim = ℓ R , τότε x  x 0 g΄(x) f (x) lim = ℓ. Το αντίστροφο δεν ισχύει πάντα. x  x 0 g(x) lim

x x0

Αν

f(x) = x+ημx, g(x)=2x, τότε για το

f (x) lim παρατηρούμε ότι: x  g(x)

lim g(x) = +.

x 

Εξάλλου:

f(x)≥x-1 κοντά στο + και lim (x  1) = + οπότε lim f (x) = +. Άρα το

x 

x 

f (x)  lim έχει απροσδιόριστη μορφή . x  g(x)  f΄(x) 1  x Αλλά = και δεν υπάρχει 2 g΄(x) f΄(x) lim , αφού δεν υπάρχει lim συνx (γιαx  g΄(x) x  f (x) τί;). Ωστόσο υπάρχει το lim , αφού x  g(x) f (x) 1 1 x x = +  και lim =0 (γιατί;). x  x 2 2 x g(x) f (x) 1 Άρα lim = . x  g(x) 2

ΕΥΚΛΕΙΔΗΣ Β΄ 90 τ.2/65

Τράπεζα ΘΕΜΑΤΩΝ Α΄ Λυκείου Σ΄ αυτό το τεύχος εγκαινιάζουμε μια νέα στήλη με θέματα Μαθηματικών Α΄ Λυκείου. Η στήλη αυτή φιλοδοξεί να περιλαμβάνει θέματα ευρύτερου ενδιαφέροντος, πρωτότυπα, συνδυαστικά για τη βαθύτερη κατανόηση της ύλης. Σ’ αυτή την προσπάθεια συμμετέχουν όλοι οι συνάδελφοι μαθηματικοί με ιδέες και προτάσεις, σε ύφος και επίπεδο δυσκολίας θεμάτων. Θα υπάρχουν πολλά άρθρα και ασκήσεις προς αυτή τη κατεύθυνση. Στα επόμενα τεύχη θα συνεχίσουμε με περισσότερες λεπτομέρειες. Περιμένουμε τις δικές σας ασκήσεις και παρατηρήσεις.

Θ1

2 , 1

Α) Να διατάξετε τους αριθμούς

και 3 σε μία σειρά από το μικρότερο προς το μεγαλύτερο (με αιτιολόγηση) Β) Να βρείτε τις τιμές του λ για τις οποίες η συνάρτηση f(x)=(λ–1)x2–2(λ–1)x+1 έχει διπλή ρίζα. Γ) Για τις τιμές του λ που βρήκατε στο προηγούμενο ερώτημα να απλοποιήσετε τον τύπο της συνάρτησης h(x)= f (x) – x 2  3  x 12 , στο ανοικτό διάστημα με άκρα τους αριθμούς και να βρείτε τις ρίζες της ΛΥΣΗ: Α) 0<1<2 1 < 2

2 και 1

0<2<3 2 < 3 . Άρα 1< 2 < 3 Β) Δ=4(λ–1)2 –4(λ–1)=…=4(λ–1)(λ–2) Για να έχει η f διπλή ρίζα πρέπει και αρκεί: λ  1 και Δ=0. Άρα λ=2 Γ) Για λ=2 έχουμε f(x)=x2–2x+1, οπότε h(x)= x 2  2x  1  x 2  3  x 12 =

(x  1)2  x 2  3  2x 3 = 2 2 = (x  1)  (x  3) = x  1  x  3 =

=x–1–(–x+ 3 )=2x–1– 3 ,

αφού

x>1

και

x  2  3 . Στο (1, 2 ) έχουμε: h(x)=02x–1– 1 3 3 =0  x= . Η τιμή αυτή είναι δεκτή, αφού 2 1 3 . ισχύει: 1< 3 2<1+ 3 1< 2 1 3 < 2  1+ 3 <2 2  Επίσης 0< 2  (1+ 3 )2<(2 2 )2  1+2 3 +3<8  (…) 3 <2  3<4, αληθής. [Τζελέπης Άλκης - Ευαγγελική Σχολή Ν. Σμύρνης]

Θ2

Α) Να γίνει η γραφική παράσταση των συναρτήσεων f(x)=x2 και g(x)=x+2 στο ίδιο σύστημα αξόνων.

Β) Να λυθεί γραφικά η ανίσωση: x 2  x+2 (1) και να επιβεβαιωθεί αλγεβρικά το συμπέρασμα. Γ) Ένας μαθητής, αφού έλυσε την (1) ανακοίνωσε 2

 1  χωρίς να συγκρίνει τους αριθμούς x1=2 +    2014  2 1 – και x2=4– πώς είναι απολύτως σί1007 2014 γουρος ότι: x1
Β) Γραφικά: Τα κοινά σημεία των Cf και Cg είναι τα Α(–1,1) και Β(2,4). Παρατηρούμε ότι η Cf βρίσκεται κάτω από τη Cg όταν και μόνον: –10, διακρίνουσα Δ=9 και ρίζες -1 και 2) Γ) Παρατηρούμε ότι 1 1 2 1 2 +( ) =(2– )= x1=22–2  2  2014 2014 2014 1 1 1 ) και x2 =(2– ) +2 =g(2– ). =f(2– 2014 2014 2014 1 1 1 <2. Άρα f(2– )
Θ3

ΕΥΚΛΕΙΔΗΣ Β΄ 90 τ.2/66

------------------------------------------------------------------------------------------------------------------------ Τράπεζα Θεμάτων Α΄Λυκείου -----------------------------------------------------------------------------------------------------------------------

στοιχα και Ζ το σημείο τομής των ευθειών ΔΗ,   , ii) ΕΘ. Αποδείξτε ότι: i) ˆ ˆ ˆ   , iii) Το τετράπλευρο ΑΒΖΓ είˆ   , OZE  2 2 ναι εγγράψιμο iv) Τα σημεία Α, Ο, Ζ είναι συνευθειακά. Λύση: ˆ ˆ B ˆ B ˆ  B και  ˆ  ˆ ,    1 . i)   O 2 1 2 1 1 2 Το τρίγωνο λοιπόν ΔΒΟ είναι ισοσκελές, οπότε η ευθεία ΔΗ είναι μεσοκάθετος του ευθύγραμμου τμήματος ΟΒ. Άρα το σημείο Η είναι μέσο του ΟΒ. Όμοια αποδεικνύουμε ότι το Θ είναι μέσο του ΟΓ. Άρα από το τρίγωνο ΟΒΓ παίρνουμε    . ii) Το σημείο Ζ είναι σημείο τομής των μεσοκαθέτων των ΟΒ, ΟΓ επομένως περίκεντρο του τριγώˆ ˆ  ˆ  ˆ  2 ,  O νου ΟΒΓ. Έτσι έχουμε ˆ 1  1 2 2 ˆ  ˆ .   ομοίως παίρνουμε 2 ˆ   ˆ O   ˆ  ABZΓ ˆ  ˆ  ˆ  180   iii) Z εγγράψιμο. 1 ˆ ˆ ˆ  A  Eˆ  A  ˆ . Από το ορθογώνιο iv) Είναι:  3 1 2 2  2 ˆ ˆ  90  ˆ  90   . τρίγωνο ΟΗΖ παίρνουμε:  1 1 2

ˆ ˆ   . Τελικώς παίρνουμε Έχουμε και  2 2 ˆA ˆ ˆ ˆ ˆ ˆ ˆ  ˆ  ˆ   ˆ   90    90   180 ,  3 2 1 2 2 2 2 δηλαδή ότι τα σημεία Α, Ο, Ζ είναι συνευθειακά. Β΄τρόπος Η ΑΟ διχοτομεί την Aˆ . Αλλά ΖΒ=ΖΟ=ΖΓ, οπότε  στον περιγεγραμτο Ζ είναι μέσο του τόξου  μένο κύκλο του ΑΒΓ. Άρα η ΑΖ διχοτομεί την Aˆ , οπότε διέρχεται από το Ο, δηλαδή τα Α,Ο,Ζ είναι συνευθειακά. [Λευτέρης Τσιλιακός – Αθήνα]

Θ4

Θεωρούμε την συνάρτηση f(x)=λx2 +(λ–2)x + 3λ –1. Α) Nα δείξετε ότι η εξίσωση f(x)=0 έχει

ρίζα τον αριθμό x1 = –

2 3 , όταν και μόνο λ= – . 3 25

3 να λύσετε την εξίσωση f(x)=0 και 25 53 103 )<00, αρκεί – [ x2,x1]. Για να είναι f(– 6 6 103 103 102 1 103 +17= =– <0– <–17. Πράγματι:– 6 6 6 6 Παρατήρηση: Το θέμα θα ήταν προφανώς δυσκο53 103 λότερο αν ζητούσαμε απλώς: f(– )
2 )(x  17) x  17 3 Στο Α έχουμε: h(x)= = . x 1 2 3(x  (x  1) 3 3(x 

ii) Στο Α έχουμε: 5h(x)≤–49 

ΕΥΚΛΕΙΔΗΣ Β΄ 90 τ.2/67

5  x  17  x 1

+49 ≤0

------------------------------------------------------------------------------------------------------------------------ Τράπεζα Θεμάτων Α΄Λυκείου -----------------------------------------------------------------------------------------------------------------------

5x  85  49x  49 ≤0 (54x+36)(x—1)≤0 x 1 2 18(3x+2)(x–1)≤0  (3x+2)(x–1)≤0  – 3 2 2 ≤x≤1  –
7  7x  10x  10  13x  13  ...  109x  109  4.060 γ) Να εξετάσετε αν ο αριθμός   5  3 6  2 είναι λύση της παραπάνω ανίσωσης. Λύση α) Οι όροι του αθροίσματος είναι διαδοχικοί όροι αριθ. προόδου με 1  7 και ω=3, οπότε

   109  7     1 3  109    35 . Άρα το

ζητούμενο άθροισμα είναι: S35  (7 109)

β) Η ανίσωση ισοδύναμα γράφεται : 7 x  1  10 x  1  ....109 x  1  4060

 (7  10  13  ...109) x  1  4060 

 1  0   1  0  Προφανώς   μ=1, ενώ   17    17  0   1     1  0  1  0   1   , ύ ή  17 2    17    1  1   1   3 0  18     1 49  ή 49 . Τελικά πρέπει και αρκεί μ=1 ή μ=– 5    5

[Γιώργος Σ. Τασσόπουλος – Αθήνα] Εστω x πραγματικός αριθμός που έχει Θ5 την ιδιότητα: το τετράγωνό του είναι μικρότερο ή ίσο από το πενταπλάσιό του. α) i) Να δείξετε ότι 0  x  5 . ii) Αν   x  5  10  2x  x  6 τότε κ=1 β) Από τους πραγματικούς που έχουν την παραπάνω ιδιότητα επιλέγεται τυχαία ένας ακέραιος : i) Να γράψετε τον δειγματικό χώρο του πειράματος . ii) Να βρείτε την πιθανότητα του ενδεχομένου Α: «ο αριθμός που επιλέγεται είναι ο αριθμητικός μέσος των αριθμών κ και 5». Λύση α) Είναι x 2  5x  x 2  5x  0 .Βρίσκουμε το πρόσημο του τριωνύμου και ισοδύναμα προκύπτει ότι 0  x  5 β) Είναι x  5  x  5  0 και 2x  10 10  2x  0 επίσης x  5  x  6  x  6  0 οπότε έχουμε Κ=-(x-5)-(10-2x)-(x-6)=11-10=1 γ) ii) Εχουμε 0  x  5 και x ακέραιος συνεπώς   0,1, 2,3, 4,5  . Ο αριθμητικός μέσος των κ

N(A) 1  . N() 6 [Παναγιώτης Στεφανής – Παράρτημα ΕΜΕ Λαμίας]

και 5 είναι ο 3 αρα A  3 και P(A) 

Θ6

α) Να δείξετε ότι: 71013... 1092030 β) Να λυθεί η ανίσωση:

35  2030 2

2030 x  1  4060  x  1  2  ( x  1  2 ή x  1  2 )  x  3 ή x  1 γ)

Γι’

αυτό

αρκεί:

5 3 6  2  3,

ή

2  3 6  2 , ή 8  6  2, ή 2  2, ή 4  2, που ισχύει. Άρα ο αριθμός ρ είναι λύση της. [Παναγιώτης Στεφανής – Παράρτημα ΕΜΕ Λαμίας] Μία ομάδα ατόμων έλαβαν μέρος σε έναν διαγωνισμό τύπου «Κυνήγι του Θησαυρού». Εισερχόμενοι σε μία ειδικά διαμορφωμένη επίπεδη έκταση το μόνο που είδαν ήταν δύο μικρά αλλά ευκρινή σημάδια Α, Β πάνω σε αυτή και έκρυψαν τον θησαυρό ακολουθώντας την εξής διαδικασία: Σχημάτισαν πάνω στην έκταση το ευθύγραμμο τμήμα ΑΒ και αρχικά έσυραν ευθύγραμμο τμήμα ΑΓ κάθετο στην ΑΒ και μικρότερο του ΑΒ. Στην συνέχεια έσυραν ευθύγραμμο τμήμα ΒΔ κάθετο στην ΑΒ, με τα Γ, Δ προς το ίδιο μέρος της ΑΒ, έτσι ώστε ΑΓ+ ΒΔ = ΑΒ. Στο μέσον του ευθύγραμμου τμήματος ΓΔ κρύφτηκε ο «Θησαυρός». Στα πλαίσια του παιχνιδιού μία δεύτερη ομάδα κλήθηκε να βρει τον «Θησαυρό» με μόνα δεδομένα την επίπεδη έκταση, τα δύο σημάδια Α, Β και τον τρόπο με τον οποίο η πρώτη ομάδα έκρυψε τον θησαυρό με την δέσμευση όμως να μη μπορεί να τον χρησιμοποιήσει. Σε σύντομο χρόνο η δεύτερη αυτή ομάδα που έτυχε να έχει στις τάξεις της ένα φοιτητή του Μαθηματικού, βρήκε τον «Θησαυρό» και κέρδισε. Α) Εξηγήστε πώς και γιατί έγινε με επιτυχία η επισήμανση του «Θησαυρού» από την δεύτερη ομάδα. Β) Ποια η πιθανότητα να ανασύρει τον θησαυρό η δεύτερη αυτή ομάδα με την πρώτη προσπάθεια της, αμέσως μετά την εύρεση του ακριβούς τρόπου προσδιορισμού του; Λύση: A) Αν θεωρήσουμε από το μέσον Μ του ευθύγραμμου τμήματος ΓΔ κάθετη στην ευθεία ΑΒ, θα περάσει από το μέσον Τ του ευθύγραμμου τμήματος ΑΒ. Αυτό

Θ7

ΕΥΚΛΕΙΔΗΣ Β΄ 90 τ.2/68

------------------------------------------------------------------------------------------------------------------------ Τράπεζα Θεμάτων Α΄Λυκείου -----------------------------------------------------------------------------------------------------------------------

συμβαίνει από το γνωστό θεώρημα που λέει ότι αν από το μέσο μίας από τις μη παράλληλες πλευρές τραπεζίου θεωρήσουμε την παράλληλη προς τις βάσεις του ευθεία αυτή θα διέλθει από το μέσον της άλλης από τις μη παράλληλες πλευράς.

Στην περίπτωση μας οι ευθείες ΓΑ, ΔΒ, ΜΤ είναι παράλληλες σαν κάθετες στην ίδια ευθεία. Εδώ      . Άρα το παρατηρούμε ότι   2 2 σημείο Μ απέχει από το σταθερό μέσον Τ σταθερή απόσταση. Αυτό σημαίνει ότι δεν είναι πλέον αναγκαία η παρουσία των σημείων Γ, Δ για τον προσδιορισμό του σημείου Μ αφού αυτό επίσης προσδιορίζεται αν από το σημείο Τ θεωρηθεί ευθεία κάθετη επί την ευθεία ΑΒ και ληφθεί εκεί σημείο  . Μ απέχον από το Τ απόσταση 2 Β) Η ευθεία ΑΒ χωρίζει το επίπεδο σε δύο ημιεπίπεδα τα 1 ,  2 . Το σημείο Μ μπορεί να βρίσκεται σε δύο θέσεις συμμετρικές ως προς την ευθεία ΑΒ, στο ημιεπίπεδο H1 ή στο ημιεπίπεδο H 2 . Επομένως όταν μία ομάδα γνωρίζει τον τρόπο εύρεσης του θησαυρού, πιθανότητα να προσδιορίσει το ση1 μείο Μ με την πρώτη προσπάθεια είναι . 2 Παρατήρηση: Αν τα ευθύγραμμα τμήματα ΑΓ, ΒΔ επιλέγονταν σε διαφορετικά επίπεδα ως προς την ευθεία ΑΒ, ώστε |ΑΓ- ΒΔ| = ΑΒ, μπορεί η αναζήτηση του θησαυρού στο μέσο Μ του ΓΔ να γίνει με τον ίδιο τρόπο που έγινε στο ερώτημα Α. Αιτιολόγηση: Το ευθύγραμμο τμήμα που έχει άκρα τα μέσα των διαγωνίων τραπεζίου ισούται με την απόλυτη τιμή της ημιδιαφοράς των βάσεων του. Στην περίπτωση μας εδώ οι βάσεις είναι οι AΓ και ΒΔ και οι διαγώνιοι οι ΑΒ, ΓΔ. Πάλι λοιπόν για AB . να προσδιορίσουμε το Μ θα πάρουμε TM  2 Το σκεπτικό λοιπόν της εύρεσης του θησαυρού παραμένει αναλλοίωτο, με την ίδια πιθανότητα 1 προσδιορισμού του σημείου Μ δηλαδή με την 2

πρώτη προσπάθεια .  Τόσο με την μέθοδο του ημιαθρίσματος όσο και με τη μέθοδο της ημιδιαφοράς των βάσεων η πιθανότητα εύρεσης του θησαυρού με την πρώτη 1 προσπάθεια είναι , έστω και αν «φαινομενικά» 2 έχουμε τέσσερις πιθανές τοποθετήσεις. [Σωτήρης Ε. Λουρίδας – Αθήνα] Α) α) Χαρακτηρίστε τους ισχυρισμούς που ακολουθούν χρησιμοποιώντας έναν από τους χαρακτηρισμούς Αληθής ή

Θ8

Ψευδής: i) αβ  α  β , για κάθε ζεύγος πραγματικών αριθμών α, β. ii) α+β+γ  α + β + γ , για κάθε τριάδα πραγματικών αριθμών α, β, γ. iii) Υπάρχουν πραγματικοί αριθμοί α, β με α  0 ή α-β δεν αντιπροβ  0 , τέτοιοι ώστε ο αριθμός α+β

σωπεύει πιθανότητα ενδεχομένου. iv) Υπάρχει ζεύγος πραγματικών αριθμών α, β με α2  β2 να αντιπροσωαβ  0 , ώστε ο αριθμός 3αβ πεύει πιθανότητα ενδεχομένου. 2 2 Β) i) Αποδείξτε ότι: 2xy x  y για κάθε x,y  . ii) Αν α, β, γ πραγματικοί αριθμοί τέτοιοι που α  0 ή β  0 ή γ  0 , αποδείξτε ότι:

αβ+βγ+γα

 1. α 2 +β 2 +γ 2 Γ) Δίδεται η εξίσωση ως προς x  R , x 2 -2x-λ 2  0, λ   1 , i) Αποδείξτε ότι η εξίσωση (1) έχει ρίζες πραγματικές και άνισες, για κάθε λ   . ii) Αποδείξτε ότι δεν υπάρχει ρίζα της (1) που να είναι πιθανότητα κάποιου ενδεχομένου. iii) Αν x1 ,x 2 είναι οι ρίζες της εξίσωσης (1) προσδιορίστε την παράμετρο λ, ώστε να ισχύει x 2  x 22 λ 2  2  1 λ+ 2  0 και η ποσότητα 1 να 4 x1 x 2 δύναται να είναι πιθανότητα κάποιου ενδεχομένου. Λύση: Α) α) (i), (ii), (iii) Θεωρία. (iv) Είναι Αληθής και αυτό επειδή για α=β=1 α2  β2 2 2  με 0<  1. παίρνουμε: 3αβ 3 3 Β) i) Για τους τυχόντες πραγματικούς x,y έχουμε: 2 0   x  y   0  x 2  y 2  2xy  2xy  x 2  y 2 .





ΕΥΚΛΕΙΔΗΣ Β΄ 90 τ.2/69

------------------------------------------------------------------------------------------------------------------------ Τράπεζα Θεμάτων Α΄Λυκείου -----------------------------------------------------------------------------------------------------------------------

ii)

αβ+βγ+γα α 2 +β 2 +γ 2



|αβ|+|βγ|+|γα|  |α|2 +|β|2 +|γ|2

2|αβ|+2|βγ|+2|γα| 2|α|2 +2|β|2 +2|γ|2   1, κα2|α|2 +2|β|2 +2|γ|2 2|α|2 +2|β|2 +2|γ|2 θότι γνωρίζουμε ότι: για κάθε δυάδα πραγματικών 2 2 αριθμών x, y ισχύει 2xy  x +y . Γ) i) Η διακρίνουσα της εξίσωσης (1) είναι   4  4λ 2  4 1  λ 2   0, για κάθε λ    . Άρα 

η εξίσωση (1) έχει δύο ρίζες πραγματικές και άνισες για κάθε λ   . ii) Οι ρίζες της εξίσωσης (1) είναι οι x1  1  1  λ 2  0 και x 2  1  1  λ 2  1, επομένως καμία από αυτές δεν είναι δυνατόν να είναι πιθανότητα κάποιου ενδεχομένου. x 2  x 22  1  2  και iii) Πρέπει και αρκεί 0  1 4 | x1 x 2 |

λ2 





2  1 λ+ 2  0

 2  0  0

 x1  x 2 

2

 3 .

 2x1x 2

4 | x1 x 2 |

Παρατηρούμε ότι:

1

4  2λ 2 4  2λ 2 4  2λ 2  1  0  και 1 4λ 2 4λ 2 4λ 2

4  2λ 2  1  4  2λ 2  4λ 2  4  2λ 2  2 4λ  2  λ  λ  - 2 ή λ  2.

 3  

2  λ  1 και τούτο διότι οι ρίζες τις ε-

ξίσωσης λ 2 





2  1 λ+ 2  0 είναι οι αριθμοί

 2,  1 και επειδή από την απαίτηση ισχύος της (3) θέλουμε τριώνυμο ετερόσημο του συντελεστή του λ 2 . Τελικά παίρνουμε λ   2. (*) Ένας τρόπος υπολογισμού των ριζών της εξί-

 2  1 λ+ 2  0 , είναι και ο εξής: λ   2  1 λ+ 2  0  λ  2 λ+λ+ 2  0   λ  λ+ 2   λ+ 2  0   λ  2   λ  1  0  λ =  2 ή λ  1.

σωσης λ 2  2

2

[Σωτήρης Ε. Λουρίδας – Αθήνα]

Προτεινόμενες Ασκήσεις Θ7

 4  1 5 1   2   3 i) Να βρείτε το λόγο λ της προόδου 2 ii) Αν f (x)  x  7x   να δείξετε ότι

β) Αν σε Γ.Π είναι

α) Σε κάθε γεωμετρική πρόοδο να δείξε 4  1 1    2 τε ότι: 1   2  3

 77  f (P(A))f    0 για κάθε ενδεχόμενο Α ενός  13  δειγματικού χώρου Ω. [Παναγιώτης Στεφανής – Παράρτημα ΕΜΕ Λαμίας]

Θ8

Δίνεται η συνάρτηση 2 2 f :    :f  x     1 x  3x  5,  

. Θεωρούμε το σύνολο      /   3 το οποίο αποτελεί το δειγματικό χώρο ενός πειράματος τύχης με ισοπιθανα απλά ενδεχόμενα και τα ενδεχόμενα Α={λΩ/η Gf διέρχεται από τα σημεία





Κ(1,10) και Λ(-2,7)}, B   / d f 1 ,f  2  13 .

i) Να βρείτε τις πιθανότητες των Α,Β ii) Αν λ>0 τότε: α) Να δείξετε ότι η f(x) έχει δύο ρίζες x1, x2 με x1 <0< x2 και |x1|> |x2|  1 1 1   τότε να βρεθεί β) Αν      /  x1 x 2 5  

η πιθανότητα του Γ. iii) Αν συμβαίνει το ενδεχόμενο Α, τότε: α) Να βρείτε τον τύπο της f. β) Να βρείτε τα σημεία τομής Μ,Ν της y  f  x   4 με τους άξονες. γ) Να βρείτε το εμβαδόν του τριγώνου ΟΜΝ και το ύψος του που αντιστοιχεί στην ΜΝ [Βασίλης Τράντος – Σχολή Αυγουλέα – Λιναρδάτου] 2

  2  Δίνεται η εξίσωση  x   = 4   1 2  (1)      2 16 2 1 1 2  2 i) Να δείξετε ότι      2 =  2 16 2 16 ii) Να δείξετε ότι μια ρίζα της (1) είναι το 1 και να βρείτε την άλλη ρίζα της. iii) Ας ονομάσουμε Α,Β και Γ τα σημεία που παριστάνουν στον άξονα x΄x τα (άκρα) α, β και το κέντρο του διαστήματος [α, β] με α>0, αντιστοίχως. Να τα τοποθετήσετε στον άξονα και να δείξεα  β  α β . τε γεωμετρικά ότι α= 2 iv) Να δείξετε γενικά ότι max(α,β)= α β  α β

Θ9

2 [ΚωνσταντίνοςΤσαγκάρης -4ο Λύκειο Πετρούπολης]

ΕΥΚΛΕΙΔΗΣ Β΄ 90 τ.2/70

----------------------------------------------------------------------------------------------------------------------------------- Το βήμα του Ευκλείδη --------------------------------------------------------------------------------------------------------------------------------

Το Βήμα του Ευκλείδη

Επιμέλεια: Γιάννης Στρατής – Βαγγέλης Ευσταθίου  

Περί  Πολυωνύμων (Ασάφειες  και Παρανοήσεις)  Γιώργος Σ. Τασσόπουλος – Σχολικός Σύμβουλος Μαθηματικών. Είναι γνωστό ότι οι Αλγεβρικές και γενικότερα οι Μαθηματικές έννοιες δεν εμφανίσθηκαν εξ αρχής με τη μορφή που σήμερα τις θεωρούμε και τις χειριζόμαστε. Για παράδειγμα, στη θεωρία των Μιγαδικών αριθμών τον αριθμό i αρχικά τον θεωρούσαν ως i  1 και με συγκεκριμένες υποχωρήσεις, ως προς τις ιδιότητες ριζικών με αρνητικά υπόρριζα, έβρισκαν συμβατά αποτελέσματα. Μετά την ανάπτυξη των Αλγεβρικών Δομών (Ομάδων, Δακτυλίων, Σωμάτων και Διανυσματικών Χώρων) δόθηκε η δυνατότητα να ορίσουμε με αυστηρότητα έννοιες και πράξεις που ήδη χρησιμοποιούσαμε. Μοιάζει, βέβαια, ανακόλουθο να ορίζουμε π.x. μία πράξη, αυθαίρετα εκ πρώτης όψεως, ενώ στην ουσία δίνουμε αυτό τον ορισμό λαμβάνοντας υπόψη μας τον τρόπο με τον οποίο εκτελούσαμε, επί χρόνια, αυτή την πράξη χωρίς αυστηρότητα. Για παράδειγμα, ορίσαμε στους μιγαδικούς αριθμούς (διατεταγμένα ζεύγη πραγματικών αριθμών) το γινόμενο (α,β)  (α,β)  (αα  ββ , αβ  αβ) , έχοντας στο πίσω μέρος του μυαλού μας την μη αυστηρή διαδικασία: (α  β 1)(α  β 1)   1ββ(  1)2  αα  ββ  (αβ  αβ) 1 .  αα αβ 1αβ Θα προσπαθήσουμε στη συνέχεια, μέσα από τον αυστηρό ορισμό της έννοιας του πολυωνύμου, να βοηθήσουμε ώστε να αποφευχθούν διάφορες παρανοήσεις, που έχουμε παρατηρήσει ότι προκύπτουν κατά τη διδασκαλία τους. Για τους μαθητές δεν είναι απαραίτητο και ίσως είναι ανέφικτο να γίνει αυστηρή και πλήρης θεμελίωση των πολυωνύμων. Είναι αρκετό, νομίζω, ο διδάσκων να τους βοηθήσει να καταλάβουν τη διαφορά ανάμεσα στο σύμβολο Χ (κεφαλαίο) σε ένα πολυώνυμο P(X) , όπου με Χ συμβολίζουμε , όπως θα δούμε στη συνέχεια , το βασικό πολυώνυμο και στο σύμβολο x (μικρό) στην αριθμητική τιμή P(x) του πολυωνύμου , όπου x   , μεταβλητή. Όταν λοιπόν στο σχολικό βιβλίο αναφέρεται ότι στα πολυώνυμα P(x) το σύμβολο x είναι μία μεταβλητή

που μπορεί να πάρει οποιαδήποτε πραγματική τιμή, δημιουργείται η ψευδαίσθηση ότι x   , ενώ όπως θα διαπιστώσουμε στη συνέχεια, ακριβώς αυτή η παραδοχή δημιουργεί όλες τις παρανοήσεις στη θεωρία των πολυωνύμων. Όπως θα δούμε λοιπόν το x ούτε μεταβλητή είναι ούτε ανήκει στο  . Είναι απλώς ένα στοιχείο από το σύνολο των πολυωνύμων που θα ορίσουμε, όπως ακριβώς το i είναι ένα στοιχείο των Μιγαδικών Αριθμών και όχι μία μεταβλητή. Για να γίνουν ευκολότερα κατανοητά τα επόμενα, θα

κάνουμε μια αναφορά στους μιγαδικούς αριθμούς, όπου, όπως θα διαπιστώσουμε, υπάρχει μία αντιστοιχία στον ορισμό τους με τον ορισμό των πολυωνύμων. Αρχικά, θα προσπαθήσουμε να γίνει κατανοητό και κυρίως αποδεκτό από τους μαθητές το πώς , ενώ i   , θεωρήσαμε παραστάσεις της μορφής α  βi με α,β   , δηλ. πράξεις που δεν έχουν πουθενά οριστεί, όπως για παράδειγμα το γινόμενο των (β,i) και το άθροισμα των (α,βi) . Αποκρύψαμε βέβαια εύσχημα από τους μαθητές ότι τέτοιες πράξεις δεν έχουν οριστεί. Η αυστηρή επέκταση του  στο  γίνεται βέβαια (βλέπε παλαιότερα σχολικά βιβλία) θεωρώντας ως  το σύνολο C  z  (α,β) / α,β   δηλ. το σύνολο των διατεταγμένων ζευγών πραγματικών αριθμών. Έτσι, η ισότητα ορίζεται ως: (α,β)  (α,β)  α  α,β  β και για οικονομία συμβόλων γράφεται με (α,β)  (α,β) . Οι πράξεις  και  στο C ορίζονται ως εξής: Αν z  (α,β) , z  (α,β) ,τότε z  z  (α  α,β  β) και z  z  (α  α  β  β, α  β  α  β) . Η προέλευση και η σκοπιμότητα αυτού του ορισμού εξηγήθηκαν στην αρχή. Διαφορετικά, φαντάζει εντελώς αυθαίρετος. Στη συνέχεια παρατηρήσαμε ότι: Αν z  (α,0), z  (α,0) , τότε z  z  (α  α,0) ,z  z  (α  α,0) . Δηλαδή τα αποτελέσματα των πράξεων στο C0  (α, 0) / α   είναι γνωστά όταν είναι γνωστά τα αποτελέσματα των αντιστοίχων πράξεων στο R και αντιστρόφως. Από Αλγεβρική άποψη ,δηλαδή, το σύνολο C0 μπορεί να θεωρηθεί ταυτιζόμενο με το R , οπότε απλουστεύοντας το συμβολισμό να γράφουμε (α, 0)  α . Αυτό ακριβώς είναι το επίμαχο σημείο στη θεμελίωση των μιγαδικών αριθμών. Δεν μπορούν δηλαδή οι μαθητές (αλλά και εμείς οι ίδιοι) να αποδεχθούν (να χωνέψουν) πώς είναι δυνατόν το ζεύγος (α,0) να είναι το ίδιο με το πρώτο στοιχείο του α, δηλαδή (α,0)=α. Στο σημείο αυτό πρέπει να γίνει κατανοητό ότι δεχτήκαμε να ταυτίσουμε το ζεύγος (α,0) με τον πραγματικό αριθμό α, όχι ως οντότητες αλλά από Αλγεβρική άποψη (άποψη χρήσης)(i). Για να κάνω αυτό όχι απλά κατανοητό αλλά αποδεκτό, κατέφευγα συνήθως στο εξής παράδειγμα ,το οποίο, κατά κανόνα, απέβαινε λειτουργικό: Όπως ακριβώς ένα πλαστικό μαχαίρι και ένα σιδερένιο μαχαίρι ως οντότητες είναι διαφορετικά αντικείμενα μεν, αλλά από χρηστική άποψη (Αλγεβρική προηγουμένως) είναι και τα δύο ίδια (μαχαίρια) αντικείμενα κοπής. (Τα περί ι-

ΕΥΚΛΕΙΔΗΣ Β΄ 90 τ.2/71

----------------------------------------------------------------------------------------------------------------------------------- Το βήμα του Ευκλείδη -------------------------------------------------------------------------------------------------------------------------------σομορ-φισμών δεν ενδείκνυται να διδαχθούν σε μαθητές Λυκείου κυρίως διότι δεν θα γίνουν αποδεκτά (εύπεπτα) αλλά θα εισπραχθούν σαν ένα απλό τυπολόγιο χωρίς ουσία). Ο μεγάλος Poincaré προκαταλαμβάνει οποιαδή-ποτε ένσταση επ’ αυτού με την ακόλουθη αδιαμφισβήτητη ρήση του: «Οι μαθηματικοί δεν μελετούν αντικείμενα, αλλά σχέσεις μεταξύ αντικειμένων. Επομένως, διαθέτουν την ελευθερία να αντικαταστήσουν κάποια αντικείμενα με άλλα, εφόσον οι σχέσεις παραμένουν αμετάβλητες» ( Βιβλιογραφία (2) ) Έχοντας πλέον αντιληφθεί πώς γίνεται η δημι-ουργία ενός νέου συνόλου C και των νέων πράξεων  ,  μέσω του ήδη γνωστού συνόλου  και των πράξεών του  ,  μπορούμε πλέον να ορίσουμε ως βασικό μιγαδικό αριθμό (μιγαδική μονάδα) το ζεύγος i  (0,1) , για το οποίο, προφανώς ισχύει i  (0,1)  (0,1)   (0  0  1 1, 0 1  1  0)=(-1,0)=-1 . Έχουν φυσικά πλέον νόημα και οι πράξεις λ (α,β)  (λ,0) (α,β)  (λ  α, β) και λ  (α,β)  (λ, 0)  (α,β)  (λ  α  0  β , λ  β  0  α)  (λ  α, λ  β) οπότε κάθε μιγαδικός αριθμός z  (α,β) γράφεται: z  (α,0)  (0,β)  α  (β  0,β 1)  α β (0,1) = α  β  i . Για οικονομία συμβόλων θα γράφουμε αντί του α  β  i το α  β  i , όπως ακριβώς για τις πράξεις 2



  

α  β , α β μεταξύ διανυσμάτων ή για τις πράξεις

f  g , f  g μεταξύ συναρτήσεων κλπ χρησιμοποιούμε τα σύμβολα της πρόσθεσης και του πολλαπλασιασμού στο  , ενώ πρόκειται για εντελώς διαφορετικές πράξεις. Θα ακολουθήσουμε, όπως θα διαπιστώσετε, μια ανάλογη διαδικασία για να ορίσουμε την έννοια του πολυωνύμου με συντελεστές πραγματικούς αριθμούς. Ως γνωστόν κάθε διατεταγμένη ν–άδα (x1 , x 2 , x 3 ,..., x ν ) λέγεται και διαδοχή με ν όρους. Από κάθε διαδοχή με ν όρους προκύπτει μια διαδοχή με άπειρους όρους, αν τα στοιχεία μετά το x ν τα θεωρήσουμε όλα μηδενικά, δη-

λαδή η διαδοχή (x1 , x 2 , x 3 ,..., x ν , 0, 0,...) . Τη διαδοχή, λοιπόν, p  (α 0 , α1 , α 2 , α 3 ,..., α ν , 0, 0,...) , όπου α 0 , α1 ,..., α ν πραγματικοί αριθμοί και α ν  0 την ονομάζουμε πολυώνυμο ν βαθμού (βαθ p=ν) με συντελεστές α 0 , α1 ,..., α ν , 0, 0,... Αν θεωρήσουμε δύο πολυώνυμα p  (α 0 , α1 , α 2 , α 3 ,..., α ν , 0, 0,...) και q  (β 0 ,β1 ,β 2 ,β3 ,...,β μ , 0, 0,...) με α ν βμ  0 , δεχόμενοι φυσικά ότι 0  αν1  αν2 ...,0  βμ1  βμ2 .... . τότε ορίζουμε την ισότητα p  q και τις πράξεις p  q , p  q μεταξύ αυτών, ως εξής : p  q  α0  β0 ,α1  β1,...,ακ  βκ , όπου κ  max  μ , ν  p  q  (α1  β1 , α 2  β 2 ,..., α κ  β κ , 0, 0,...) με (p  q)  κ , αν έχει μη μηδενικό συντελεστή. p  q  (α 0β 0 , α 0β1  α1β 0 , α 0β 2  α1β1  α 2β 0 ,... ..., α 0β λ  α1β λ 1  ...  α λ 1β1  α λ β 0 , 0, 0,...)

βαθ

με

βαθ( p  q )  μ  ν  λ . Στη συνέχεια όπως ακριβώς δεχθήκαμε στο  ότι: (λ, 0)  0 , από Αλγεβρική άποψη το ίδιο και εδώ, δεχόμαστε ότι (λ, 0, 0,...)  λ και αυτό το λέμε σταθερό πολυώνυμο. Το πολυώνυμο (0,1, 0, 0,...) το λέμε βασικό πολυώνυμο και το συμβολίζουμε με x, δηλαδή x  (0,1, 0,...)  x1 , όπως ακριβώς ως βασικό μιγαδικό αριθμό ορίσαμε το i  (0,1) . Αντιλαμβανόμαστε , λοιπόν, ότι δεν πρόκειται για μεταβλητή. Θα μπορούσαμε να το συμβολίσουμε με I  (0,1, 0, 0,...) όπως κάναμε για το i  (0,1) ώστε να μην παραπέμπει σε άγνωστο (μεταβλητή). Τέλος ορίζουμε x 0  (1, 0, 0...)  1 . Μπορούμε, πλέον, να ορίσουμε δυνάμεις x2  xx  (0,0,1,0,0...),x3  (0,0,0,1,0,0,...) και γενικά x k  (0, 0,...,1, 0, 0...) με τη μονάδα να βρίσκεται στην k+1 θέση, καθώς και τις πράξεις: λ  x k  (0, 0,..., λ, 0, 0,...) με λ στην k+1 θέση , λ  x k  (λ, 0, 0,...,1, 0, 0...) για κ  0 με την μονάδα στην k+1 θέση, ενώ λ  x 0  (λ  1, 0, 0,...)  λ  1 . Μπορούμε πλέον να γράψουμε: p  (α0 ,α1,α2 ,...,αν ,0,0...)  α 0  x 0  α1  x1  ...  α ν  x ν  0  x ν 1  ... και για οικονομία συμβόλων p  α 0  x 0  α1  x1  ...  α ν  x ν  0  x ν 1  ...

ή απλούστερα p(x)  α 0  α1  x  ...  α ν  x ν καθώς και p(x)  α ν  x ν  ...  α1  x  α 0 . Στο προηγούμενο πολυώνυμο p(x) προφανώς με τα σύμβολα + , . εννοούμε τις πράξεις μεταξύ πολυωνύμων (όπως τις ορίσαμε), αφού το x είναι το βασικό πολυώνυμο. Αν όμως με τα σύμβολα +, . εννοήσουμε τις αντίστοιχες πράξεις στο  , τότε ορίζεται μια συνάρτηση p :    με τύπο p(x)  α ν  x ν  ...  α1  x  α 0 , όπου το x είναι τώρα μεταβλητή στο  και όχι το βασικό πολυώνυμο. Η συνάρτηση αυτή λέγε-ται πολυωνυμική συνάρτηση αντίστοιχη του πολυωνύμου p(x) . Την αριθμητική τιμή της συνάρτησης p(x) για x  λ ,δηλαδή τον αριθμό p(λ)  α ν λν  ...  α1λ  α 0 τη λέμε και αριθμητική τιμή του πολυωνύμου p(x) στη θέση λ. Κακώς, βέβαια, λέγεται και αριθμητική τιμή του πολυωνύμου p(x) για x  λ , αφού το βασικό πολυώνυμο x αποκλείεται να είναι ίσο με το σταθερό πολυώνυμο λ, δηλαδή αποκλείεται να είναι: 1  x  0  0  x  λ . Καλό θα ήταν να συμβολίζουμε με Χ (κεφαλαίο) το βασικό πολυώνυμο και με x (μικρό) τη μετα-βλητή στην αντίστοιχη πολυωνυμική συνάρτηση. Έτσι, θα είχαμε το πολυώνυμο p(X) και την αντίστοιχη συνάρτηση p(x) χωρίς κίνδυνο παρανόησης. Μηδενικό πολυώνυμο λέγεται το πολυώνυμο O(x)  0  0  0  x  0  0  x  0  x2  ... και δεν έχει συγκεκριμένο βαθμό, ενώ κάθε πολυώνυμο της μορφής p  (0, 0,..., α κ , 0, 0...)  α κ x κ με α κ  0 λέγεται μονώ-

ΕΥΚΛΕΙΔΗΣ Β΄ 90 τ.2/72

----------------------------------------------------------------------------------------------------------------------------------- Το βήμα του Ευκλείδη -------------------------------------------------------------------------------------------------------------------------------νυμο κ βαθμού. Παρατηρούμε πλέον ότι αν οι πράξεις εκτελεστούν σαν πράξεις μεταξύ αριθμών ( όπως γινόταν πριν τον αυστηρό ορισμό του πολυωνύμου), θα δώσουν αποτέλεσμα συμβατό με τον ορισμό. Αν π.x. p  α 0  α1  x , q  β 0  β1  x  β 2  x 2 , τότε p  q  α 0  β 0  (α1  β1 )  x  β 2  x 2 

α 0  β 0  (α1  β1 )  x  (α 2  β 2 )  x 2 και p  q=α 0β 0 +(α 0β1 +α1β 0 )  x+(α 0β 2 +α1β1 )  x +α1β 2  x 2

παίρνει τη μορφή q(x) ή περιφραστικά το p(x) να είναι ίσο με το q(x) κ.λ.π. 2η παρανόηση: Για τις συναρτήσεις f (x)  2x 2  3x  5 και g(x)  x 2  4 η f (x) συνάρτηση ορίζεται μόνον όταν x 2  4  0 , g(x) δηλαδή

3

 α 0β 0  (α 0β1  α1β 0 )  x  (α 0β 2  α1β1  α 2β 0 )  x 2

(α0β3  α1β2  α2β1  α3β0 )  x αφού α2  α3  0 και β 3  0 . Έγινε πλέον κατανοητό ότι το βασικό πολυώνυμο x δεν είναι μεταβλητή στο  (εξάλλου , τότε το x 0 δεν θα είχε νόημα για κάθε x   , αφού το 00 δεν ορίζεται). Η κατάργηση του συμβόλου () για την ισότητα των πολυωνύμων που υπήρχε στα παλιότερα σχολικά βιβλία και η χρήση του ίδιου συμβόλου x για το βασικό πολυώνυμο και για τη μεταβλητή στην πολυωνυμική συνάρτηση δημιουργούν επικίνδυνες παρανοήσεις. Διαφορετικά θα ήταν ευδιάκριτο ότι οι σχέσεις p(x)  q(x) ή p(X)  q(X) δηλώνουν ισότητα πολυωνύμων ενώ η σχέση p(x)  q(x) για κάθε x   δηλώνει ισότητα συναρτήσεων. 3

1η παρανόηση: Αν θεωρήσουμε τα πολυώνυμα p(x)  x 2  λx  9 και q(x)  x  λ και ζητήσουμε να βρεθεί το λ   ώστε p(x)  q(x) , μπορεί κάποιος να ισχυριστεί ότι αυτό συμβαίνει όταν και μόνο όταν η εξίσωση x 2  λx  9  x  λ 2 δηλαδή η εξίσωση x  (λ  1)x  9  λ  0 έχει ρίζες στο R. Γι αυτό πρέπει και αρκεί Δ  0 , όπου Δ  (λ  1) 2  4(9  λ)  λ2  2λ  35 η διακρίνουσά της. Θα έβρισκε λοιπόν Δ  0  λ  (, 7]  [5, ) . Αν όμως απαιτούσαμε p(x)  q(x) θα καταλάβαινε ότι εννοούμε πως οι αντίστοιχοι συντελεστές πρέπει και αρκεί να είναι ίσοι, δηλαδή 1  0, λ  1, λ  9 πράγμα αδύνατο. Μπορούμε εν προκειμένω να αποφύγουμε τέτοιες παρανοήσεις χωρίς να χρησιμοποιήσουμε το σύμβολο () που απου-σιάζει από το σχολικό βιβλίο, με το να ανάγουμε τους συμβολισμούς και τα προβλήματα μεταξύ πολυωνύμων σε πολυωνυμικές συναρτήσεις με βάση το παρακάτω θεώρημα. (Για την απόδειξή του βλέπε Άλγεβρα Σπύρου Κανέλλου σελ. 164 – 165 , 1ος τόμος, εκδόσεις Παπαδημητρόπουλου, Αθήνα 1966) Αποδεικνύεται, λοιπόν, ότι η ισότητα πολυωνύμων εκφράζεται και με τις παρακάτω ισοδυναμίες: p(x)  O(x)  p(λ)  0 , για κάθε λ    α 0  α1  ...  α ν  0

και p(x)  q(x)  p(λ)  q(λ) , για κάθε λ    α 0  β 0 ,α1  β1 ,...,α κ  β κ , όπου κ  max μ, ν .

Το προηγούμενο πρόβλημα θα μπορούσε να αποφευχθεί αναγόμενο στις αντίστοιχες συναρτήσεις ως εξής: Να βρεθεί το λ   , ώστε p(x)  q(x) , για κάθε x   , δηλαδή x 2  λx  9  x  λ , για κάθε x   . Αλλιώς θα αναγκαστούμε όπως συμβαίνει στο σχολικό βιβλίο, να χρησιμοποιήσουμε τις εκφράσεις: Να βρεθεί το λ   , ώστε το p(x) να

x  2, 2 . Η διαίρεση όμως του πολυωνύμου

p(x)  2x 2  3x  5 διά του q(x)=x 2 -4

έχει πάντα νόημα αφού q(x) O(x) , δηλαδή υπάρχει x   , ώστε q(x)  0 . Αν ρωτήσετε όμως τους μαθητές πότε ορίζεται η διαίρεση p(x) : q(x) , αυτοί που δεν έχουν κατά-νοήσει τη διαφορά μεταξύ πολυωνύμου και πολυωνυμικής συνάρτησης, θα μας απαντήσουν όταν x  2, 2 , λές και υπήρχε περίπτωση να x  2 ,δηλαδή x  2 , 1 x  0  0  x  2 ή είναι δηλ.1  x  0  0  x  2 . Γενικά η διαίρεση του πολυωνύμου Δ(x) διά του δ(x) , δη-

λαδή η πράξη Δ(x) : δ(x) ορίζεται μόνον όταν δ(x)  O(x) , (Ι). Επειδή όμως τα σύμβολα () και (  ) απουσιάζουν από το σχολικό βιβλίο, τη συνθήκη (Ι) θα την αποδώσουμε με την αντί-στοιχη πολυωνυμική συνάρτηση δ(x) ως εξής: για κάποιο x , αφού (Ι)  δ(x)  0 , δ(x)  O(x)  δ(x)  0 , για κάθε x   . Αν γράψουμε όμως, απλώς, δ(x)  0 , μπορεί να δημιουργηθεί η προηγούμενη παρανόηση όπου είχαμε δ(x)  x 2  4 . Οι εκφράσεις «για κάθε x   » και « υπάρχει x   » αντιλαμβανόμαστε πλέον ότι αναφέρονται σε πολυωνυμική συνάρτηση και όχι σε πολυώνυμο. Τέλος, τονίζουμε ότι σχέση διάταξης p  x   q  x  στο σύνολο των πολυωνύμων (όπως και των μιγαδικών αριθμών) δεν ορίζεται. Δύο βασικούς στόχους πρέπει να πετύχουμε κατά τη διδασκαλία των πολυωνύμων. α) Να καταλάβουν οι μαθητές ότι το σύμβολο x στα πολυώνυμα δεν είναι αριθμός (μεταβλητή στο  ), αλλά το βασικό πολυώνυμο. β) Να καταλάβουν τη διαφορά ανάμεσα στο πολυώνυμο και την πολυωνυμική συνάρτηση.  Όπως γράφει (εκλαϊκευτικά) ο Θ. Ν. Καζαντζής στο ιστορικό βιβλίο του «Πολυώνυμα» (Εκδόσεις Αθανασιάδη – Τορνακίδη , Θεσσαλονίκη 1977 ) το X είναι ένα σύμβολο για το οποίο έχουν οριστεί δυνάμεις X ν καθώς και πράξεις της μορφής λ  X ν , λ   και αθροίσματα της μορφής π.χ. 2  X 2  3  X  5  X 0 τα οποία ονομάζουμε πολυώνυμα. Έχουμε γνωρίσει ήδη τέτοια σύμβολα όπως τις συναρτήσεις f, τους τετραγωνικούς πίνακες Α, κ. λ. π, για τα οποία έχουν νόημα εκφράσεις της μορφής 2  f 2  3  f  5  f 0 , 2  A 2  3  A  5  A0 . Τη λέξη «μεταβλητή» τη χρησιμοποιεί σε εισαγωγικά , ακριβώς για να μας προβληματίσει χωρίς όμως να μας ξενίσει. Θεωρεί το x απλώς ως το βασικό πολυώνυμο, από το οποίο προκύπτουν τα υπόλοιπα. Με αυτό τον αρκετά απλουστευτικό τρόπο μπορούν να επιτευχθούν και οι δύο στόχοι που προαναφέραμε και να αποφευχθούν οι σχετικές παρανοήσεις. Σε όλες αυτές τις περιπτώσεις βέβαια, ως αντίστοιχη πολυωνυμική συνάρτηση μπορούμε να ορίσουμε την p(x)  2x 2  3x  5  1 , για κάθε x   .

ΕΥΚΛΕΙΔΗΣ Β΄ 90 τ.2/73

----------------------------------------------------------------------------------------------------------------------------------- Το βήμα του Ευκλείδη -------------------------------------------------------------------------------------------------------------------------------1η παρανόηση: Αν θεωρήσουμε τα πολυώνυμα p(x)  x 2  λx  9 και q(x)  x  λ

και ζητήσουμε να βρεθεί το λ   ώστε p(x)  q(x) , μπορεί κάποιος να ισχυριστεί ότι αυτό συμβαίνει όταν και μόνο όταν η εξίσωση x 2  λx  9  x  λ δηλαδή η εξίσωση

x 2  (λ  1)x  9  λ  0 έχει ρίζες στο R. Γι αυτό πρέπει και αρκεί Δ  0 , όπου Δ  (λ 1)2  4(9  λ)  λ2  2λ  35 η διακρίνουσά της. Θα έβρισκε λοιπόν Δ  0  λ  (, 7]  [5, ) . Αν όμως απαιτούσαμε p(x)  q(x) θα καταλάβαινε ότι εννοούμε πως οι αντίστοιχοι συντελεστές πρέπει και αρκεί να είναι ίσοι, δηλαδή 1  0, λ  1, λ  9 πράγμα αδύνατο. Μπορούμε εν προκειμένω να αποφύγουμε τέτοιες παρανοήσεις χωρίς να χρησιμοποιήσουμε το σύμβολο () που απου-σιάζει από το σχολικό βιβλίο, με το να ανάγουμε τους συμβολισμούς και τα προβλήματα μεταξύ πολυωνύμων σε πολυωνυμικές συναρτήσεις με βάση το παρακάτω θεώρημα. (Για την απόδειξή του βλέπε Άλγεβρα Σπύρου Κανέλλου σελ. 164 – 165 , 1ος τόμος, εκδόσεις Παπαδημητρόπουλου, Αθήνα 1966) Αποδεικνύεται, λοιπόν, ότι η ισότητα πολυωνύμων εκφράζεται και με τις παρακάτω ισοδυναμίες: p(x)  O(x)  p(λ)  0 , για κάθε λ    α 0  α1  ...  α ν  0 και p(x)  q(x)  p(λ)  q(λ) , για κάθε λ    α 0  β 0 ,α1  β1 ,...,α κ  β κ , όπου κ  max μ, ν .

Το προηγούμενο πρόβλημα θα μπορούσε να αποφευχθεί αναγόμενο στις αντίστοιχες συναρτήσεις ως εξής: Να βρεθεί το λ   , ώστε p(x)  q(x) , για κάθε x   , δηλαδή

x 2  λx  9  x  λ , για κάθε x   . Αλλιώς θα αναγκαστούμε όπως συμβαίνει στο σχολικό βιβλίο, να χρησιμοποιήσουμε τις εκφράσεις: Να βρεθεί το λ   , ώστε το p(x) να παίρνει τη μορφή q(x) ή περιφραστικά το p(x) να είναι ίσο με το q(x) κ.λ.π. 2η παρανόηση: Για τις συναρτήσεις f (x)  2x 2  3x  5 και g(x)  x 2  4

f (x) g(x)

ορίζεται

x  2, 2 .

Η

μόνον

διαίρεση

η

συνάρτηση

όμως

του

δηλαδή πολυωνύμου

p(x)  2x  3x  5 διά του q(x)=x -4 2

2

έχει πάντα νόημα αφού q(x) O(x) , δηλαδή υπάρχει

x   , ώστε q(x)  0 .

x  2 ,δηλαδή 1  x  0  0  x  2 ή x  2 , είναι δηλ. 1  x  0  0  x  2 . Γενικά η διαίρεση του πολυωνύμου Δ(x) διά του δ(x) , δηλαδή

η

πράξη

Δ(x) : δ(x)

ορίζεται

μόνον

όταν

δ(x)  O(x) , (Ι). Επειδή όμως τα σύμβολα () και (  ) απουσιάζουν από το σχολικό βιβλίο, τη συνθήκη (Ι) θα την αποδώσουμε με την αντί-στοιχη πολυωνυμική συνάρτηση δ(x) ως εξής: (Ι)  δ(x)  0 , για κάποιο x   , αφού

δ(x)  O(x)  δ(x)  0 , για κάθε x   . Αν γράψουμε όμως, απλώς, δ(x)  0 , μπορεί να δημιουργηθεί η προηγούμενη παρανόηση όπου είχαμε δ(x)  x 2  4 . Οι εκφράσεις «για κάθε x   » και « υπάρχει x   » αντιλαμβανόμαστε πλέον ότι αναφέρονται σε πολυωνυμική συνάρτηση και όχι σε πολυώνυμο. Τέλος, τονίζουμε ότι σχέση διάταξης p  x   q  x  στο σύνολο των πολυωνύμων (όπως και των μιγαδικών αριθμών) δεν ορίζεται. Δύο βασικούς στόχους πρέπει να πετύχουμε κατά τη διδασκαλία των πολυωνύμων. α) Να καταλάβουν οι μαθητές ότι το σύμβολο x στα πολυώνυμα δεν είναι αριθμός (μεταβλητή στο  ), αλλά το βασικό πολυώνυμο. β) Να καταλάβουν τη διαφορά ανάμεσα στο πολυώνυμο και την πολυωνυμική συνάρτηση.  Όπως γράφει (εκλαϊκευτικά) ο Θ. Ν. Καζαντζής στο ιστορικό βιβλίο του «Πολυώνυμα» (Εκδόσεις Αθανασιάδη – Τορνακίδη , Θεσσαλονίκη 1977 ) το X είναι ένα σύμβολο για το οποίο έχουν οριστεί δυνάμεις X ν καθώς και πράξεις της μορφής λ  Xν , λ  και αθροίσματα της μορφής π.χ.

2  X2  3  X  5  X0 τα οποία ονομάζουμε πολυώνυμα. Έχου-

x2  4  0 ,

όταν

Αν ρωτήσετε όμως τους μαθητές πότε ορίζεται η διαίρεση p(x) : q(x) , αυτοί που δεν έχουν κατά-νοήσει τη διαφορά μεταξύ πολυωνύμου και πολυωνυμικής συνάρτησης, θα μας απαντήσουν όταν x  2, 2 , λές και υπήρχε περίπτωση να

με γνωρίσει ήδη τέτοια σύμβολα όπως τις συναρτήσεις f, τους τετραγωνικούς πίνακες Α, κ. λ. π, για τα οποία έχουν νόημα εκφράσεις της μορφής 2 f 2  3 f  5 f 0 , 2 A2 3 A5 A0 . Τη λέξη «μεταβλητή» τη χρησιμοποιεί σε εισαγωγικά , ακριβώς για να μας προβληματίσει χωρίς όμως να μας ξενίσει. Θεωρεί το x απλώς ως το βασικό πολυώνυμο, από το οποίο προκύπτουν τα υπόλοιπα. Με αυτό τον αρκετά απλουστευτικό τρόπο μπορούν να επιτευχθούν και οι δύο στόχοι που προαναφέραμε και να αποφευχθούν οι σχετικές παρανοήσεις. Σε όλες αυτές τις περιπτώσεις βέβαια, ως αντίστοιχη πολυωνυμική συνάρτηση μπορούμε να ορίσουμε την p(x)  2x 2  3x  5 1 , για κάθε x   .

Βιβλιογραφία 1. Συμεών Μιχ. Μποζαπαλίδης ( Γραμμική Άλγεβρα, έκδοση Α. Αλτιντζή . Θεσσαλονίκη 1984) 2. Δ. Βάρσος – Δ. Δεριζιώτης – Μ. Μαλιάκας – Σ. Παπασταυρίδης – Ε. Ράπτης – Ο. Ταλέλλη (Μια εισαγωγή στην Άλγεβρα, εκδόσεις «σοφία» 2003) 3. John B. Fraleigh ( Εισαγωγή στην Άλγεβρα, Πανεπιστημιακές εκδόσεις Κρήτης 2003. απόδοση Α. Γιαννόπουλος. Επιμέλεια Ν. Μαρμαρίδης) 4. Θ. Ν. Καζαντζής ( Πολυώνυμα, εκδόσεις Αθανασιάδη – Τορνακίδη , Θεσσαλονίκη 1977 ) 5. Σ. Γ. Κανέλλος ( Άλγεβρα, εκδόσεις Παπαδημητρόπουλου, Αθήνα 1966 ) 6. Γ. Λεγάτος – Ν. Παπαδόπουλος ( Άλγεβρα και στοιχεία από την Μαθηματική Ανάλυση, έκδοση Gutenberg , Αθήνα 1974 ) i

Διαβάζουμε στο κλασικό πλέον βιβλίο των Γεράσιμου Λεγάτου – Νίκου Παπαδόπουλου « Άλγεβρα και στοιχεία από την Μαθηματική Ανάλυση» ( Έκδοση Gutenberg 1974) σελ. 386 , τόμος (ΙΙ) με γενικευμένη διατύπωση (εδώ) ότι: Είναι φανερό πως μπορούμε να λέμε ότι τα σύνολα A  (α, 0) / α   και B   , ως προς τις δομές τους, δεν διαφέρουν παρά μόνο κατά τον συμβολισμό των στοιχείων τους και να θεωρούμε το Α ταυτιζόμενο με το Β. Για τον λόγο αυτό μπορούμε να απλουστεύσουμε το συμβολισμό των στοιχείων του Α γράφοντας α αντί (α,0). Και με αυτή όμως την αρκετά κατανοητή έκφραση δεν κατάφερνα να πείσω τους μαθητές μου.

ΕΥΚΛΕΙΔΗΣ Β΄ 90 τ.2/74

---------------------------------------------------------------------------------------------------------------------------------- Ευκλείδης Προτείνει … ------------------------------------------------------------------------------------------------------------------------------

O Ευκλείδης προτείνει ...

«Η καρδιά των μαθηματικών είναι τα προβλήματα και οι λύσεις και ο κύριος λόγος ύπαρξης του μαθηματικού είναι να λύνει προβλήματα».

Ρ. R. HALMOS

Επιμέλεια: Γ. Κ. ΤΡΙΑΝΤΟΣ - Ν. Θ ΑΝΤΩΝΟΠΟΥΛΟΣ – Θ. A. ΤΖΙΩΤΖΙΟΣ Συνάδελφοι, η στήλη επανορθώνει. Δεν δόθηκαν στους λύτες των προτεινομένων θεμάτων οι συνάδελφοι: Νίκος Παύλου – Χαλκίδα για την άσκηση 195/81, Κωνσταντίνος Τζαγκαράκης – Χανιά για τις ασκήσεις 196 – 202 – 204, Δημήτρης Κουκάκης – Κιλκίς για τις ασκήσεις 202 – 204 – 205 – 206 , Βασίλειος Λαγογιάννης – Αγία Παρασκευή, για την άσκηση 197/81 και Αντώνης Ιωαννίδης – Λάρισα για την άσκηση 205/83.

ΑΣΚΗΣΗ 209 ( ΤΕΥΧΟΥΣ 83) Να βρεθούν οι πραγματικές λύσεις της εξίσωσης 6 x  1  8x  27 x 1 (1)

R ΟΓ  , το τρίγωνο ΟΡΓ είναι ορθογώ2 2   π νιο στο Ρ. δηλ. OΡΓ   OΝΓ οπότε το τετράπλευρο 2 ΟΡΝΓ είναι εγγράψιμο σε κύκλο c1 και συνεπώς, ισχύει

ναι και ΟΡ 





ΡΝΒ  ΡΟΓ 

π . 3

(ΓΙΑΝΝΗΣ ΗΛΙΟΠΟΥΛΟΣ – Καλαμάτα) ΛΥΣΗ ( ΔΙΟΝΥΣΗΣ ΓΙΑΝΝΑΡΟΣ – Πύργος)

Η εξίσωση (1) είναι αδύνατη στο διάστημα (, 0] αφού , x  0  6 x  27 x 1  1  1  80  8x . Αν x  0 , τότε: (1)  2 x  3x  (3x )3  33  1  (2 x )3 και αν τεθεί 2x  a  0,3x  b  0

η

εξίσωση

γράφεται

27ab  b  27  27a και τέλος b (3a) 3  3 b (3a)3, οπότε ισχύει: b  3a  3  0 (2) ή b  3a  3 (3). Επειδή η σχέση (3) απορρίπτεται, ισχύει μόνο η (2), δηλ ισχύει b  3a  3 δηλ. 3x  3  2x  3 ή 3x 1  2x  1 . Θέτουμε x  1  u , οπότε είναι 3u  2  2 u  1 ή 3u  2u  2u  1 ή f (3)  f (2)  f (2)  f (1) (4), όπου έχει 3

3

3

3

3

τεθεί f (t)  t u , t  0 συνάρτηση που ικανοποιεί τις προϋποθέσεις του Θ.Μ.Τ σε καθένα από τα διαστήματα [1, 2],[2,3] . Συνεπώς, υπάρχουν z1  (1, 2), z 2  (2,3) τέτοια, ώστε να ικανοποιούν τις σχέσεις: u 1 u 1 f (2)  f (1)  uz1 , f (3)  f (2)  uz 2 οπότε η (4) u 1 1

 uz

 uz

u 1 2

u 1 1

 u(z

z

u 1 2

)0

z1 u 1 ) 1 z2 u  0  u  1  0  u  0  u  1 και επειδή είναι u  x  1 , παίρνουμε τελικά x  1  x  2 . Λύσεις έστειλαν επίσης οι: Γιώργος Μήτσιος – Ράμια Άρτας, Ροδόλφος Μπόρης – Δάφνη, Αντώνης Ιωαννίδης – Λάρισα , Δημήτρης Κουκάκης – Κιλκίς. ΑΣΚΗΣΗ 210 (ΤΕΥΧΟΥΣ 84) (ΓΙΩΡΓΟΣ ΜΗΤΣΙΟΣ – Ράμια Άρτας )

Έστω ΑΒ , ΒΓ δύο διαδοχικές πλευρές κανονικού 9 – γώνου , Ο το κέντρο του κύκλου που διέρχεται από τα Α , Β , Γ και Μ , Ν τα μέσα των πλευρών ΑΒ , ΒΓ αντίστοιχα. Θεωρούμε σημείο Ρ του ΟΜ τέτοιο, ώστε ΟΑ ΟΡ  . 1)Να βρεθεί το μέτρο της γωνίας ΡΝΒ . 2 2) Αν είναι ( Β Ρ Ν )  k τότε να δειχθεί ότι το k είναι η (Α Ο Μ )

μικρότερη θετική ρίζα της εξίσωσης 8x 3  18x  9  0 . ΛΥΣΗ ( ΡΟΔΟΛΦΟΣ ΜΠΟΡΗΣ – Δάφνη)

Παρατηρούμε 

ΝΟΓ 

ότι:







ΡΟΝ  ΡΟΒ ΒΟΝ 

R ΡΝ 2       2π π ημ(ΡΟΝ) ημ(ΡΝΟ) ημ ημ(  ΡΝΒ) 9 2 ΡΝ R ΡΝ R 2π      ΡΝ  Rημ  2π 2π 1 9 2συν ΡΝΒ ημ ημ 2 9 9 2 ΡΝ

ΟΡ

(1)

Σύμφωνα με την υπόθεση, έχουμε:

 u  0  z1u 1  z 2u 1  u  0  (

1)

2) Ο νόμος των ημιτόνων στο τρίγωνο ΟΡΝ δίνει:

2π , 9

   π π , οπότε ΡΟΓ  ΡΟΝ  ΝΟΓ  . Επειδή εί9 3

 1  ΡΝ  ΒΝ  ημ(ΡΝΒ) (ΒΡΝ) k  2  1 (ΑΟΜ )  ΑΜ  ΟΜ 2  λ9 π 2π 3  ΡΝ   ημ(ΡΓΟ) ΡΝ  ημ (1) Rημ 3 9 2 2    λ9 π α9 Rσυν  α9 9 2

π π 3 2ημ συν  9 9 2  3  ημ π  k  3  ημ π (2)  π 9 9 συν 9 π 3π π π  3ημ  4ημ 3 , δηλαδή Όμως, ημ  ημ 3 9 9 9 3 π π  ημ (3  4ημ 2 ) και λόγω της (2) 2 9 9

3 k k2  (3  4 )  8k3 18k  9  0 . Δηλ, ο k είναι ρίζα 2 3 3 8x 3  18x  9  0 της εξίσωσης (3). Επειδή

ΕΥΚΛΕΙΔΗΣ Β΄ 90 τ.2/75

---------------------------------------------------------------------------------------------------------------------------------- Ευκλείδης Προτείνει … -----------------------------------------------------------------------------------------------------------------------------0

π π π π π 3 3 (4).   , είναι: 3ημ  3ημ  0  k  9 6 2 9 6 2 2

3 ). 2 Πράγματι, η συνάρτηση f με f (x)  8x 3  18x  9 είναι

Η (3) δέχεται μοναδική ρίζα στο διάστημα (0,

3 συνεχής στο [0, ] ως πολυωνυμική και ισχύει 2 3 f (0)f ( )  9  (6 3  9)  27(3  2 3)  0 , οπότε από 2 το θεώρημα του Bolzano η (3) έχει τουλάχιστον μία 3 (0, ) ρίζα στο διάστημα και επειδή 2 3 f (x)  24x2 18  6(4x2  3)  0 , για κάθε x  (0, ) η 2 3 f είναι γνησίως φθίνουσα στο [0, ] οπότε η ρίζα 2 αυτή είναι μοναδική και είναι η μικρότερη από τις θετικές ρίζες της (3). ( η άλλη θετική ρίζα της βρίσκεται στο διάστημα [1,2] και μία τρίτη αρνητική στο διάστημα [-2,-1]). Λύση έστειλε: Διονύσης Γιάνναρος – Πύργος.

ΑΣΚΗΣΗ 211 ( ΤΕΥΧΟΥΣ 84 ) Να βρεθούν όλα τα ζεύγη (x,y) των ακεραίων x,y έτσι, ώστε 3x  7 οι ρητοί αριθμοί A  να είναι πολ/σια του 17. 11y  4 (ΛΕΥΤΕΡΗΣ ΤΣΙΛΙΑΚΟΣ - Γαλάτσι ). ΛΥΣΗ (ΓΙΑΝΝΗΣ ΗΛΙΟΠΟΥΛΟΣ – Καλαμάτα).

Έστω ότι υπάρχει k  Z :

3x  7  17k . Τότε 11y  4

3x  7  17k(11y  4) ή 3x  7  187ky  68k (1)

ή 3x  6  1  186ky  ky  66k  2k δηλ. 1  ky  2k(mod 3)  k  1  ky(mod 3) (2). Διακρίνουμε περιπτώσεις: 1) Αν k  0(mod 3) , δηλ. k  3m , m  Z , τότε από την (2) έχουμε 3m  1  3my  2k(mod 3) δηλ. 3m  1  3my  6m  3t , t  Z δηλ. 1  0(mod 3) άτοπο. 2) Αν k  1(mod 3) δηλ. k  3m  1, m  Z , τότε (2)  3m  (3m  1)y(mod 3) , οπότε ισχύει 3m  (3m  1)y  3t , t  Z και τελικά 3m  3my  y  3t , δηλ. y  0(mod 3) ή y  3n όπου n  Z . Με αντικατάσταση στην (1) έχουμε: 3x  7  187(3m  1)3n  68(3m  1) , m , n  Z ή 3x  561n  1683mn  204m  75 και τελικά: x  187n  561mn  68m  25 , m , n  Z (3). (Αν για παράδειγμα είναι m  n  1 , τότε x  841 y  3 είναι μία λύση του προβλήματος με k  4 ). 3) Αν k  2(mod 3) , δηλ, k  3m  2, m  Z τότε (2)  3m  1  (3m  2)y(mod 3) , δηλ, 3m  1  3my  2y  3t , t  Z ή 3m  1  y  3my  3y  3t δηλ. y  1  0(mod 3) δηλ. y  3n  1, n  Z . Με αντικα-

τάσταση στην (1) παίρνουμε διαδοχικά: 3x  7  187(3m  2)(3n  1)  68(3m  2)  3x  1683mn  561m  1122n  204m  231  x  561mn  119m  374n  77 (4) ( π.χ m  n  1 , x  739, y  2 και k  5 ). Λύσεις έστειλαν επίσης οι: Δημήτρης Καραβότας Κάτω Αχαϊα , Ροδόλφος Μπόρης – Δάφνη, Δημήτριος Καρβελάς – Πεύκη.

ΑΣΚΗΣΗ 212 ( ΤΕΥΧΟΥΣ 84 ) 

Σε ορθογώνιο τρίγωνο ΑΒΓ ( A900) με πλευρές α,β,γ, θεˆ ˆ αντιστοίχως. ωρούμε τις διχοτόμους ΒΕ και ΓΔ των B,Γ 1 1 1   , όπου ρ η ακτίνα του ΒΔ ΓΕ ρ εγγεγραμμένου κύκλου στο τρίγωνο ΑΒΓ. α 2βγ 2) Να βρεθεί η ελάχιστη τιμή της A  . (α  β)(α  γ)

1) Να δειχθεί ότι

(ΓΙΩΡΓΟΣ ΑΠΟΣΤΟΛΟΠΟΥΛΟΣ – Μεσολόγγι) ΛΥΣΗ ( ΔΗΜΗΤΡΗΣ ΚΑΡΑΒΟΤΑΣ – Κάτω Αχαϊα).

Έστω α,β,γ τα μήκη των πλευρών του ορθογωνίου στο Α τριγώνου ΑΒΓ και Ε το εμβαδόν του. Από το θεώρημα της εσωτερικής διχοτόμου για τις διχοτόμους ΓΔ και ΒΕ των γωνιών Γ, Β έχουμε: ΔΑ β ΔΑ  ΔΒ β  α γ βα       ΔΒ α ΔΒ α ΔΒ α 1 βα 1 γα  (1). Όμοια, (2). Έτσι,  ΔΒ αγ ΕΓ αβ 1 1 β  α γ  α β 2  βα  γ 2  γα α  β  γ      ΔΒ ΕΓ αγ αβ αβγ βγ 2τ τ τ 1    . Υποθέτουμε ότι η ακτίνα ρ είναι 2Ε Ε τρ ρ σταθερή κατά μέγεθος. Παρατηρούμε ότι α 2βγ αγ αβ . Από την ανισότητα A   (α  β)(α  γ) α  β α  γ των Cauchy – Schwarz παίρνουμε 

αγ αβ 2 2α2βγ     α β α  γ α β α  γ αγ(α  γ)  αβ(α β)  αγ αβ 2βγ 4Ε 2Ε 2τρ      2ρ Α 4ρ2 , με την ισότητα α  β  γ 2τ τ τ A

να ισχύει όταν και μόνον όταν: Επειδή είναι: ρ  τ  α 

αγ αβ  ... β  γ . α β α  γ

β  γ  α 2β  α 2β  β 2   2 2 2

β(2  2)  2ρ  β(2  2)  β  ρ(2  2) κατα2 λήγουμε ότι η παράσταση Α δέχεται ελάχιστη τιμή ίση 

ΕΥΚΛΕΙΔΗΣ Β΄ 90 τ.2/76

---------------------------------------------------------------------------------------------------------------------------------- Ευκλείδης Προτείνει … ------------------------------------------------------------------------------------------------------------------------------

με 4ρ 2 όταν το τρίγωνο είναι ισοσκελές ορθογώνιο με μήκη καθέτων πλευρών β  γ  ρ(2  2) . Λύσεις έστειλαν επίσης: Γιάννης Ηλιόπουλος – Καλαμάτα , Ροδόλφος Μπόρης – Δάφνη , Διονύσης Γιάνναρος – Πύργος , Γιώργος Μήτσιος – Άρτα, Φίλιππος Σερέφογλου – Μελίσια.

ΑΣΚΗΣΗ 213 (ΤΕΥΧΟΥΣ 85 ). Δίνονται οι μιγαδικοί z1 , z 2 ,..., z ν

τέτοιοι, ώστε

z1  z 2  ...  z ν  0 , z1  z 2  ...  z ν  1

z  z1  z  z2  ...  z  zν  z  z1  z  z2  ... 



zz 1 zz 1 zz2 1 1  1   ...  ν  zν z1 z2 zν

zz2 1 z2

 ... 

zzν 1 zν

228. Δίνεται τρίγωνο ΑΒΓ με ΑΒ  ΑΓ  α και 

Β  Γ  400 . Προεκτείνουμε την ΑΓ κατά τμήμα 

ΑΔ  ΒΓ . Να αποδειχθεί ότι είναι: ΓΒΔ  100 . (ΓΙΩΡΓΟΣ ΚΥΡΙΑΚΟΠΟΥΛΟΣ – Βόλος )

(ΑΝΤΩΝΗΣ ΙΩΑΝΝΙΔΗΣ – Λάρισα). ΛΥΣΗ 1Η ( ΓΙΩΡΓΟΣ ΑΠΟΣΤΟΛΟΠΟΥΛΟΣ – Μεσολόγγι) . Έχουμε:

z

(ΝΙΚΟΛΑΟΣ ΕΠΑΜ. ΤΣΙΛΙΑΚΟΣ – Χίος ) 

Αν z είναι τυχαίος μιγαδικός αριθμός, τότε να δειχθεί ότι: z  z1  z  z 2  ...  z  z ν  ν .

z  zν  z  z1  z  z2  ...  z  zν  z 

ΠΡΟΤΕΙΝΟΜΕΝΑ ΘΕΜΑΤΑ

226. Να επιλυθεί στο ZxZ η εξίσωση: 5x 3  6y3  1331 (ΛΕΥΤΕΡΗΣ ΤΣΙΛΙΑΚΟΣ – Γαλάτσι ) 227. Σε ημικύκλιο διαμέτρου ΑΔ τρείς διαδοχικές χορδές ΑΒ , ΒΓ , ΓΔ έχουν μήκη 1 , 2 , 3 , αντιστοίχως. Να υπολογισθεί το μέτρο της ακτίνας R του ημικυκλίου αυτού.

1 1  z   ....  z1 z2 zz1 1 z1



 zz1 1  zz2 1  ... 

229. Να δειχθεί ότι για οποιαδήποτε συνεπίπεδα διανύ         σματα α ,β , γ , δ με α  β  γ  δ  ο ισχύει η σχέση:           α  β  γ  δ  αδ  βδ  γδ . (ΑΝΤΩΝΗΣ ΙΩΑΝΝΙΔΗΣ – Λάρισα ).

230. Οξυγώνιο τρίγωνο ΑΒΓ με μήκη πλευρών α ,β , γ είναι εγγεγραμμένο σε κύκλο (Ο, R) . Αν Η είναι το ορθόκεντρο του τριγώνου και οι ευθείες ΑΗ , ΒΗ , ΓΗ τέμνουν τον κύκλο στα σημεία A1 , B1 , Γ1 ,αντιστοίχως, να αποδειχθούν οι σχέσεις: 1)

zzν 1  (zz1  zz2  ...  zzν )  (1  1  ...  1) 

3

1

ΛΥΣΗ 2 ( ΒΑΣΙΛΗΣ ΛΑΓΟΓΙΑΝΝΗΣ – Αγία Παρασκευή).

Υποθέτουμε ότι οι μιγαδικοί z k είναι της μορφής: z k  συνθ k  iημθ k , k  1, 2,..., ν . Σύμφωνα με το πρό-

συνθ k1

k

ν

 0, ημθk  0 (1) Ο τυχαίος μιk1

γαδικός z είναι της μορφής: z  x(συνω  iημω),x  z  0 , οπότε η παράσταση γράφεται στη μορφή:

ν

 zz k 1

k



ν

  (xσυνω  συνθ k ) 2  (xημω  ημθ k ) 2 k 1 ν

  x 2  2xσυν(θ k  ω)  1  f (x)

(2)

k 1

Από την (2) με παραγώγιση ως προς x έχουμε: ν x  συν(θ k  ω) (3). f (x)   2 k 1 x  2xσυν(θ k  ω)  1 με f (0)  0 ( λόγω των σχέσεων (1)). Επίσης, ν

(3)  f (x)  

ημ 2 (θk  ω)

 0 , για κάθε 3 [x 2  2xσυν(θ k  ω)  1]2 x  0 . Άρα, η f  είναι γνησίως αύξουσα στο [0, ) και συνεπώς, ισχύει f (x)  f (0)  0 , για κάθε k 1

x  0 , δηλ, η f είναι γνησίως αύξουσα στο [0, ) με

αποτέλεσμα: f(x)  f(0)  ν , για κάθε x  0 , ή το αυτό: ν

 z z k1

k

1

1

αβ γ

Η

βλημα ισχύουν:

, όπου ρ η ακτίνα του εγγε-

γραμμένου κύκλου στο τρίγωνο ΑΒΓ. 2) E A B Γ Α Β Γ  R . (Γ. ΑΠΟΣΤΟΛΟΠΟΥΛΟΣ – Μεσολόγγι)

z(z1  z2  ...  zν )  ν  z  0  ν  ν  ν

ν

α 2β 2 γ 2  6ρ ( Η Α 1 )( Η Β 1 )( Η Γ1 )

 ν γεγονός, που αποδεικνύει το ζητούμενο.

Λύσεις έστειλαν επίσης: Ροδόλφος Μπόρης – Δάφνη, Γεώργιος Ούτρας – Τρίκαλα, Διονύσης Γιάνναρος – Πύργος.

2

231. Δίνεται τρίγωνο ΑΒΓ. Να βρεθεί σημείο Μ εσωτερικό του τριγώνου τέτοιο, ώστε να ισχύει: 





ΜΑΒ  ΜΒΓ  ΜΓΑ  θ και στη συνέχεια να αποδειχθεί ότι σφθ  σφΑ  σφΒ  σφΓ .(Γ. ΤΡΙΑΝΤΟΣ – Αθήνα ). 2 2 232. Δίνεται η έλλειψη C : x 2  y 2  1 , όπου a  b  0 ,

a

b

με εστίες E , E  . Θεωρούμε τυχαίο σημείο Μ της έλλειψης και την εφαπτομένη (ε) της έλλειψης στο σημείο Μ, που τέμνει τον άξονα x x στο σημείο Ζ. Ευθεία (η) κάθετη στην (ε) στο σημείο Μ τέμνει τον άξονα x x στο σημείο Η. Να δειχθεί ότι τα σημεία Ζ , Η είναι αρμονικά συζυγή των εστιών E , E  της έλλειψης. (ΚΩΝΣΤΑΝΤΙΝΟΣ ΤΣΟΥΒΑΛΑΣ – Λήμνος ).

233. Να βρεθούν οι ακέραιοι αριθμοί α , β, γ που ικα α 2  β  γ  480 (1) νοποιούν τις ισότητες:  . 2 α  β  γ  512 (2) (ΚΩΝΣΤΑΝΤΙΝΟΣ ΤΣΟΥΒΑΛΑΣ – Λήμνος ).

234. Αν α , β είναι μη αρνητικοί πραγματικοί αριθμοί, τότε να δειχθεί ότι ισχύει η σχέση: (α  3β)(2α  3β)(β  3α)(2β  3α)  4(α  β  αβ ) 4 (ΓΙΩΡΓΟΣ ΑΠΟΣΤΟΛΟΠΟΥΛΟΣ – Μεσολόγγι)

235. Δίνεται κύκλος (O, R) ,μία διάμετρος ΑΒ και σταθερό σημείο Λ εσωτερικό του τμήματος ΟΑ. Θεωρούμε τυχαία χορδή ΓΔ του κύκλου αυτού που διέρχεται από το Λ και την εφαπτομένη Αx του κύκλου στο σημείο Α. Ονομάζουμε Ε , Ζ τα σημεία τομής των ΒΓ , ΒΔ με την Αx αντιστοίχως. Να αποδειχθεί ότι το γινόμενο AE  AZ είναι σταθερό. (ΓΙΩΡΓΟΣ ΤΡΙΑΝΤΟΣ – Αθήνα ). Αναμένουμε προτεινόμενα θέματά σας!!!.

ΕΥΚΛΕΙΔΗΣ Β΄ 90 τ.2/77

 

Παναγιώτης Π. Χριστόπουλος "H Γεωμετρία χρειάζεται έμπνευση όσο και η ποίηση" Alexander Puskin "Δεν αξίζει πάντα κάτι που μετριέται. Και κάτι που αξίζει δεν μετριέται πάντα". Albert Einstein

Τα Μαθηματικά του διαστήματος Γρίφοι Τα Αρνιά Ρώτησαν το βοσκό πόσα αρνιά έχει και αυτός απάντησε: Άσε μη ρωτάς, δύσκολη χρονιά, πριν 10 μέρες μου έφαγε ο Λύκος τέσσερα πρόβατα αλλά ευτυχώς πριν 3 μέρες γέννησαν τα μισά από ένα αρνί. Όλα αυτά που γέννησαν μου δίνουν μισό κιλό γάλα κάθε μέρα το καθένα εκτός από δέκα που μου δίνουν από ένα κιλό το καθένα και γεμίζω το δοχείο των 25 κιλών. Πόσα αρνιά έχει ο βοσκός; Οι ορειβάτες Τέσσερις ορειβάτες βρίσκονται στην άκρη μιας μεγάλης γέφυρας και θέλουν να περάσουν απέναντι, είναι όμως νύχτα και ο φακός που διαθέτουν έχει ακόμα 60 λεπτά διάρκεια πριν τελειώσει η μπαταρία. Εκτός αυτού η γέφυρα αντέχει μόνο δυο άτομα και συνεπώς δεν μπορούν να περάσουν όλοι μαζί. Οι ορειβάτες είναι διαφόρων ηλικιών και φυσικής κατάστασης και ο καθένας κάνει διαφορετικό χρόνο για να περάσει απέναντι. Ο πρώτος κάνει 5 λεπτά, ο δεύτερος 10 λεπτά, ο τρίτος 20 λεπτά και ο τέταρτος 25 λεπτά. Όπως κατά

λαβαίνετε όταν πάνε να περάσουν δυο μαζί την γέφυρα ο χρόνος που θα κάνουν θα είναι αυτός που χρειάζεται ο πιο αργός π.χ. αν πάει ο δεύτερος με τον τρίτο θα κάνουν 20 λεπτά. Πώς θα καταφέρουν να περάσουν απέναντι και οι τέσσερις πριν να τους σβήσει ο φακός; Το 6 Δίνονται τρεις διαδοχικοί θετικοί ακέραιοι εκ των οποίων ο μεγαλύτερος διαιρείται με το 3. Παίρνουμε το άθροισμα των τριών αριθμών και από το άθροισμα που προκύπτει παίρνουμε το άθροισμα των ψηφίων του και από το άθροισμα των ψηφίων πάλι το νέο άθροισμα των ψηφίων έως ότου καταλήξουμε σε μονοψήφιο αριθμό. Αυτός θα είναι ο 6. [Ιάμβλιχος 250μ.Χ.-325μ.Χ.] π.χ. 1534+1535+1536=4605, 4+6+0+5=15, 1+5=6. Οι απαντήσεις στα μαθηματικά μας διασκεδάζουν Τα Αρνιά: Είχε x+4 πρόβατα μείον 4 που έφαγε ο Λύκος έμειναν x από αυτά πήρε

αρνιά. Όμως παίρνει

( -10). +10=25 κιλά γάλα. Άρα x=80 δηλ έχει 40 αρνιά. Οι ορειβάτες: Θα ξεκινήσει ο 5 με τον 10 και θα γυρίσει πίσω ο 5 σύνολο 15 λεπτά. Θα ξεκινήσει ο 20 με τον 25 θα περάσουν απέναντι και θα γυρίσει ο 10 σύνολο 50 λεπτά. Και τέλος θα περάσει απέναντι ο 5 με τον 10 σύνολο 60 λεπτά. Το 6: Η εξήγηση είναι με διαιρετότητα. Έχω τις 3άδες 1η 1, 2, 3 2η 4, 5, 6 3η 7, 8, 9 …… αν αν1=(1,4,7,…)=1+(ν-1).3 είναι όροι αριθμητικής προόδου με πρώτο όρο 1 και διαφορά 3, ομοίως αν2=(2, 5, 8, …)=2+(ν-1).3 και αν3=(3, 6, 9, …)=3+(ν-1).3 Επομένως αν1+αν2+αν3=6+9(ν-1) Το τμήμα του αριθμού που είναι 9(ν-1) είναι πολλαπλάσιο του 9 άρα το άθροισμα των ψηφίων του διαιρείται με 9 και το άθροισμα αυτό των ψηφίων καταλήγει σε ψηφίο 9 οπότε το άθροισμα των ψηφίων του αν1+αν2+αν3=6+9(ν-1) καταλήγει σε 6+9=15 και 1+5=6. Πολλά Μ μας Δ. στο επόμενο

Μάνος Δανέζης: Τα Μαθηματικά του Σύμπαντος Τη Δευτέρα 2 Δεκεμβρίου 2013 έγινε διάλεξη από τον Μάνο Δανέζη στο αμφιθέατρο (αίθουσα διαλέξεων) της ΕΜΕ με θέμα τα Μαθηματικά του Σύμπαντος. Οι πολλές μαθηματικές αναφορές για τη διάσταση του χρόνου για τη γεωμετρία του Σύμπαντος, τα χαρακτηριστικά παραδείγματα για την κορδέλα του Mobius η μπουκάλα του Klein και οι ενδιαφέρουσες φιλοσοφικές αναφορές του δημιούργησαν μια μαγευτική ατμόσφαιρα με πολλά ερωτήματα και μια υπόσχεση για μια επόμενη συνάντηση από τον ομιλητή. ΕΥΚΛΕΙΔΗΣ Β΄ 89 τ.1/78

 

«Καθήκον του δάσκαλου είναι να ψυχανεμιστεί κατά που τραβούν τα νιάτα και να πλωρίσει την ψυχή του κατά κει» έγραψε ο Νίκος Καζαντζάκης. Πράγματι ο εκπαιδευτικός, ο άνθρωπος που έχει το προνόμιο να παρακολουθεί εκ του σύνεγγυς τη ζωή του παιδιού σε όλες του τις εκφάνσεις, έχοντας σ’ αυτή τη συμμετοχή που επιβάλει ο ρόλος του, έχει πλείστες όσες ευκαιρίες για να πετύχει αυτό, που για το μεγάλο μας συγγραφέα φαίνεται ν’ αποτελεί προυπόθεση για να πάρει κάποιος τον τίτλο του δάσκαλου. Ο χώρος δράσης του εκπαιδευτικού είναι από τη φύση του χώρος έμπνευσης. Κάθε στιγμή επικοινωνίας, κάθε στιγμή συνεργασίας με το παιδί για τη μεταλαμπάδευση της γνώσης, μπορεί ν’ αποτελέσει για το δάσκαλο στιγμή έμπνευσης, και να μετουσιωθεί μες της ψυχής του το εργαστήρι σε αφετηρία για καινούργια μέθοδο, που οδηγεί σε καινούργια γνώση με αποδέκτη και πάλι το μαθητή. Τις τελευταίες δεκαετίες έντονος είναι ο προβληματισμός της εκπαιδευτικής κοινότητας, σχετικά με τις μεθόδους διδασκαλίας που πρέπει ν’ εφαρμοστούν, ώστε να διεγερθεί το διαφέρον του μαθητή και να μετάσχει στη διαδικασία για την απόκτηση της γνώσης. Πολύ περισσότερο τα τελευταία χρόνια που η εξάπλωση της χρήσης του διαδικτύου σε συνδυασμό με την εξέλιξη της κινητής τηλεφωνίας παρέχει δελεαστικούς τρόπους «διασκέδασης» με αποπροσανατολιστικές και ίσως εκμαυλιστικές επιδράσεις. Ελληνική διάκριση σε Πανευρωπαϊκό Διαγωνισμό Δυο σημαντικές διακρίσεις στον πανευρωπαϊκό διαγωνισμό «Le – Math: Learning mathematics through new communication Factors 2013» για τη συγγραφή θεατρικού έργου με θέμα τα μαθηματικά, έλαβε ο μαθηματικός Κωνσταντίνος Κορδώσης. Διακρίθηκε με το τρίτο βραβείο του διαγωνισμού για το θεατρικό έργο, An autcast for a blueblood και με τη δεύτερη τιμητική διάκριση για το έργο, Decimal form of number: to be «huge» or no to be… Αν όμως ένας οποιοσδήποτε εκπαιδευτικός προβληματίζεται, ο μαθηματικός προβληματίζεται κατά μείζονα λόγο, αφού και λόγω της φύσης του μαθήματος, δύσκολα ο τελευταίος θα μπορούσε να κάνει επίκληση στο συναίσθημα του μαθητή και να το θέσει αρωγό στη διαδικασία της μάθησης. Πώς ένας δάσκαλος Μαθηματικών θα μπορούσε να βελτιώσει την εικόνα που έχει μορφώσει ο μέσος μαθητής για τα μαθηματικά; «που είναι δύσκολα, αφηρημένα και στείρα», «που εκτός από κάποιες πολύ βασικές γνώσεις δεν μας χρειάζονται», πώς θα μπορούσε να δώσει χρώμα στο για τους πολλούς ασπρόμαυρο και σκυθρωπό μαθηματικό τοπίο; Από ποιο δίαυλο να διοχετεύσει τη γνώση δεδομένου ότι ο κάθε μαθητής λαμβάνει στις δικές του συχνότητες; Υπάρχει κάποιος δίαυλος ανοικτός για πολλούς αποδέκτες; Υπάρχει μια γλώσσα που την κατανοούν σχεδόν όλοι; Μα φυσικά αφού μιλάμε για παιδιά ναι, υπάρχει μια τέτοια γλώσσα. Είναι η γλώσσα του παιχνιδιού. Και βέβαια παιχνίδι είναι και το θεατρικό παιχνίδι. Θέατρο και Μαθηματικά λοιπόν, δυο τουλάχιστον φαινομενικά ετερόκλητα αντικείμενα. Πόσες πιθανότητες υπάρχουν να είναι επιτυχημένο ένα τέτοιο ζευγάρωμα; Από τις αρχές της δεκαετίας του 1990 στο 59ο Γυμνάσιο Αθηνών μια ομάδα καθηγητών που ασχολήθηκε με θεατρικές παραστάσεις επί σειρά ετών, διαπίστωσε την καταλυτική δράση του θεάτρου στην επικοινωνία μεταξύ διδασκόντων και διδασκομένων, αλλά και των διδασκομένων μεταξύ τους. Η επιτυχημένη συμμετοχή προσέδιδε στους μαθητές αυτοεκτίμηση με άμεση συνέπεια την αύξηση της αυτοπεποίθησης , στη συνέχεια βελτίωση της σχέσης του παιδιού με το σχολείο, που τελικά οδηγούσε σε άνοδο των δεικτών απόδοσης στα μαθήματα. Η σκέψη πως ήταν δυνατόν να τεθούν η ενάργεια και η δύναμη του θεατρικού λόγου στην υπηρεσία της διδακτικής των μαθηματικών, οδήγησε στην ιδέα πως αν θα μπορούσαν να γραφούν θεατρικά σενάρια με θέματα από τα μαθηματικά όπου θα πρωταγωνιστούσαν προσωποποιημένες οι μαθηματικές οντότητες, ίσως η παρουσίαση των αντίστοιχων θεατρικών παραστάσεων στη σχολική σκηνή, να βοηθούσε έναν ικανό αριθμό μαθητών να δουν τα μαθηματικά με άλλη οπτική γωνία και να τα νιώσουν φιλικότερα. Η πρώτη θεατρική παράσταση με θέμα απ’ τα Μαθηματικά ( Ας είναι επιτρεπτός ο τίτλος «μαθηματικό θέατρο») πραγματοποιήθηκε στη σχολική σκηνή το 2002. Ήταν μια παρουσίαση των Αρχαιοελληνικών μαθηματικών σε τρείς πράξεις. Τύχη αγαθή ήταν ν’ ανταποκριθεί στην πρόσκλησή η Ελληνική Μαθηματική Εταιρεία. Έτσι παρακολούθησε την παράσταση ο τότε Γενικός Γραμματέας της Εταιρείας κ. Τυρλής Ιωάννης ο οποίος αποκομίζοντας άριστες εντυπώσεις, κατέστη εις το εξής ένθερμος υποστηρικτής της προσπάθειας για την ανάπτυξη τη βελτίωση και τη διάδοση του μαθηματικού θεάτρου. Το 2006 Παρουσιάστηκε το θεατρικό έργο «ο άνθρωπος που μετρούσε την άμμο» Ήταν μια ελεύθερη θεατρική διασκευή από το ομώνυμο μυθιστόρημα της Αμερικανίδας ελληνίστριας Τζίλιαν Μπράντσοου σχετική με τη ζωή και τα επιτεύγματα του νεαρού Αρχιμήδη στα Μαθηματικά και τη μηχανική στα χρόνια που ακολούθησαν την επιστροφή του από την Αλεξάνδρεια. Για την ανταπόκριση στις απαιτήσεις του έργου χρειάστηκε να κατασκευαστούν κάποια λειτουργικά ομοιώματα μηχανών του Αρχιμήδη ( κοχλίας, καταπέλτης), Επίσης Κιθάρα, αυλός, κ.λ.π . Η κατασκευές πραγματοποιούνταν στο χώρο του σχολείου κι έτσι οι μαθητές είχαν την ευκαιρία να παρακολουθούν όλες τις φάσεις κατασκευής. Το έργο παρουσιάστηκε και στο «μαθηματι-

ΕΥΚΛΕΙΔΗΣ Β΄ 90 τ.2/79

 

-------------------------------------------------------------------------------------------------------------------------------------- Μαθηματικό Θέατρο -----------------------------------------------------------------------------------------------------------------------------------

  κό πανηγύρι» εκείνης της χρονιάς που διοργάνωσε η εταιρία « ΘΑΛΗΣ ΚΑΙ ΦΙΛΟΙ» και εντυπωσίασε.Μια περαιτέρω μελέτη του έργου του Συρακούσιου σοφού οδήγησε στην απόφαση να διευρυνθεί η προσπάθεια που αφορούσε ανακατασκευές των επιτευγμάτων των μεγάλων κατασκευαστών της Αρχαιότητας. (Κτησίβιου, Αρχιμήδη, Φίλωνα του Βυζαντίου και Ήρωνα του Αλεξανδρινού), με απώτερο στόχο την μεσοπρόθεσμη παρουσίαση μιας έκθεσης. Ένας άμεσος καρπός αυτής της προσπάθειας ήταν η κατασκευή και η τοποθέτηση μαρμάρινων ηλιακών ρολογιών, σε δυο δήμους της χώρας. Στο Δήμο Παρανεστίου Δράμας (2007) και στο Δήμο Τενέας Κορινθίας ( 2009 ). Ένα νέο έργο με θέμα απ’ τα Μαθηματικά με τίτλο «ομοζυγωτικά τρίδυμα» που παρουσιάστηκε στο 59ο Γυμνάσιο Αθηνών, και στο αντίστοιχο μαθηματικό πανηγύρι, είχε επίσης επιτυχία και την αμέριστη συμπαράσταση της ΕΜΕ της οποίας ο τότε αντιπρόεδρος κ. Δημάκος και ο γενικός γραμματέας κ. Τυρλής βράβευσαν τους μαθητές που υποδύθηκαν τους ρόλους. Παράλληλα εντάθηκαν οι προσπάθειες από μέρους του τελευταίου, για να εξευρεθεί τρόπος, ώστε να ενταχθεί το μαθηματικό θέατρο στο πρόγραμμα του ετήσιου πανελληνίου συνεδρίου της ΕΜΕ. Τέλος η προσπάθεια τελεσφόρησε το 2010 και δυο μονόπρακτα (καλλιστεία τετραπλεύρων και μονόπρακτο τεσσά  ρων πράξεων) που το καθένα είχε διάρκεια 20 περίπου λεπτών, παρουσιάστηκαν έστω και εΝτίνος Κορδώσης κτός προγράμματος, γεγονός που είχε σαν συνέπεια την σχετικά μικρή προσέλευση συνέδρων, που όμως ενθουσιάστηκαν με την ιδέα και έγιναν ενθουσιώδεις πρεσβευτές της. Η αρχή είχε γίνει.

Το επόμενο βήμα βασίστηκε στην σκέψη να γίνει εισαγωγή του μαθηματικού θεάτρου μέσα στην αίθουσα διδασκαλίας. Για το σκοπό αυτό γράφτηκαν δέκα ολιγόλεπτης διάρκειας ευέλικτα θεατρικά μαθηματικά δρώμενα το φθινόπωρο του 2010. Κατά τη διάρκεια της σχολικής χρονιάς 2010- 2011 ένας ενθουσιώδης περιοδεύων θίασος από μαθητές του 59ου Γυμνασίου Αθηνών, περιέτρεχε τα σχολεία της περιοχής Πατησίων, και μπαίνοντας μέσα στις αίθουσες διδασκαλίας την ώρα των Μαθηματικών προσέφερε υπό μορφή ψυχαγωγίας στους μαθητές των σχολείων αυτών, κάποια λεπτά εύκολης μαθηματικής γνώσης. Ίσως το μαθηματικό θέατρο θα μπορούσε ωριμάζοντας ν’ αποτελέσει ένα ακόμα βέλος στη φαρέτρα της διδακτικής των Μαθηματικών. Παρά το ότι τουλάχιστον προς το παρόν δεν φαίνεται να φιλοδοξεί να εξελιχθεί σε αυτοτελή μέθοδο διδασκαλίας, θα μπορούσε ν’ αποτελέσει μέρος της διδακτικής πρακτικής του διδάσκοντος, προσφέροντας έναν εύκολο κι ευχάριστο για το μαθητή τρόπο εμπέδωσης της σχετικής παραγράφου, που μόλις έχει ολοκληρωθεί η διδασκαλία της. Σταθμό αποτέλεσε το συνέδριο της ΕΜΕ στην Αθήνα το φθινόπωρο του 2011 όπου δόθηκε η δυνατότητα να ενταχθούν στο πρόγραμμα και να παρουσιαστούν σ’ ένα κατάμεστο αμφιθέατρο τα παραπάνω μαθηματικά δρώμενα. Έτσι σκεδάστηκε η ιδέα του μαθηματικού θεάτρου όπως διαμορφώθηκε, στα περισσότερα μέρη της Ελλάδας και στην Κύπρο. Η ΕΜΕ μάλιστα προχώρησε στην εκτύπωση κάποιων εκατοντάδων αντιτύπων των μαθηματικών θεατρικών δρωμένων, τα οποία με μορφή μικρού εγχειριδίου διανεμήθηκαν στους συνέδρους. Το Μάρτιο του 2012 οι εκπρόσωποι της ΕΜΕ που προσήλθαν στο 59ο Γυμνάσιο Αθηνών και ξεναγήθηκαν απ’ τους μαθητές στην έκθεση αρχαίας ελληνικής τεχνολογίας η οποία στο μεταξύ είχε εμπλουτιστεί με νέα εκθέματα, μεταξύ των οποίων συμπεριλαμβανόταν το αρχαίο οδόμετρο και το υδραυλικό ρολόι του Αρχιμήδη, που αποτελεί ένα θαύμα αυτοματισμού, με τον συνολικό αριθμό εκθεμάτων να ξεπερνά το τριάντα. Ή έκθεση λειτούργησε και την επόμενη σχολική χρονιά και την επισκέφθηκαν αρκετά σχολεία. Είναι πολύ πιθανόν οι παραπάνω προσπάθειες ανάπτυξης και διάδοσης του σχολικού μαθηματικού θεάτρου να είναι μια από τις αιτίες που η συμμετοχή των Ελλήνων στον πανευρωπαϊκό διαγωνισμό «Le- math: Learning mathematics through new communication Factors» για τη συγγραφή σεναρίου με θέμα τα μαθηματικά, με ελεύθερη συμμετοχή και για πολίτες εκτός Ευρωπαικής ένωσης, απέφερε σημαντικές διακρίσεις. Ο διαγωνισμός που έγινε στη Λευκωσία της Κύπρου, ήταν ο πρώτος του είδους και ενταγμένος στο ειδικό πρόγραμμα COMENIUS έχει στόχο την ανάπτυξη της μεθοδολογίας για τη διδασκαλία και την εκμάθηση των μαθηματικών μέσω νέων παραγόντων επικοινωνίας. Άμεση επιδίωξη η δημιουργία εγχειριδίου με τα έργα των διαγωνιζομένων, το οποίο θα τεθεί στη διάθεση των απανταχού διδασκόντων τα Μαθηματικά.

ΕΥΚΛΕΙΔΗΣ Β΄ 90 τ.2/80

30ο ΣΥΝΕ∆ΡΙΟ της ΕΜΕ

Το 30ο Συνέδριο της ΕΜΕ φέτος έγινε στις αρχές Νοέμβρη στην Καρδίτσα. Τριάντα χρόνια πριν όταν οργανώσαμε το πρώτο συνέδριο δεν φανταζόμασταν ότι μετά από λίγα χρόνια το συνέδριο της ΕΜΕ θα ήταν μια τόσο σημαντική εκδήλωση. Περισσότερα από χίλια άτομα, μαθηματικοί και άλλοι εκπαιδευτικοί συγκεντρώθηκαν στην Καρδίτσα και αντάλλαξαν απόψεις για τα Μαθηματικά, τις νέες τεχνολογίες και την εκπαίδευση. Φοιτητές αλλά και μαθητές ήταν εκεί.

Οι σημαντικές εκδηλώσεις, οι κεντρικές εισηγήσεις, οι ομιλίες των συνέδρων και για πρώτη φορά συμμετοχή των μαθητών με πρωτότυπες παρουσιάσεις στο χώρο των Μαθηματικών παρουσίασαν ένα άλλο πνεύμα στο συνέδριο και άφησαν θετικές εμπειρίες για το μέλλον.

ΣΤΑΓΟΝΕΣ ΜΑΘΗΜΑΤΙΚΩΝ, ΦΥΣΙΚΩΝ ΕΠΙΣΤΗΜΩΝ ΚΑΙ ΑΣΤΡΟΝΟΜΙΑΣ

Μαθητές από το 15ο Λύκειο Αθήνας ήρθαν στην Καρδίτσα στο 30ο Συνέδριο μας και παρουσίασαν πειραματικές κατασκευές για θέματα μαθηματικών, Φυσικής, Χημείας, Αστρονομίας, κ.ά. με τους καθηγητές τους Κατσαμένη Σύλβια και Μαΐστρο Κωνσταντίνο. Τις εργασίες αυτές οι μαθητές έκαναν στα πλαίσια του project. Το αποτέλεσμα ήταν μια μονοήμερη εξόρμηση στην Καρδίτσα με 14 μαθητές όλων των τάξεων του Λυκείου. (…) Μπήκαμε στο Συνεδριακό Κέντρο λίγο σαν σίφουνες με πολλές απαιτήσεις… μας έδωσαν 5 τραπέζια, στην καλύτερη θέση, έξω από την αίθουσα των κεντρικών ομιλιών… στρατοπεδεύσαμε… ξεδιπλώσαμε την πραμάτεια και στηθήκαμε… περιμένοντας τους «βαρβάρους» και ήρθαν κατά κύματα και ήρθαν ένας-ένας, μεγάλοι και μικροί και κάποιοι πολύ μικροί, που έπαιξαν με το μπάσκετ, ζωγραφίζοντας ανυποψίαστοι με τη μπίλια παραβολικές τροχιές, όλοι έδειξαν μεγάλο ενδιαφέρον, με ερωτήσεις, με παρατηρήσεις, με διευκρινιστικές ερωτήσεις, με διορθωτικές παρατηρήσεις, τι μαθηματικοί, αλλά και τι δάσκαλοι θα ήταν άλλωστε! Και οι μαθητές; Τι έκαναν; Από τις 12 το μεσημέρι ως τις 7 το απόγευμα με μόνο διάλειμμα μισή ώρα για φαγητό… Το χάρηκαν, εξήγησαν, ώρες ευχαριστήθηκαν, μίλησαν, ασταμάτητες… και όσο εξηγούσαν τόσο μάθαιναν και όσο μάθαιναν τόσο πιο πολύ όμορφα ένιωθαν. Η παρακολούθηση της ομιλίας του κ. Δανέζη ήταν μια πρωτόγνωρη εμπειρία που τη συζητούσαν για μέρες στο σχολείο με τους συμμαθητές τους. Πιστεύουμε εδώ και χρόνια ότι οφείλουμε να δίνουμε με κάθε ευκαιρία το λόγο στους μαθητές μας. Αυτό προσπαθήσαμε και αν κρίνουμε από τα επαινετικά σχόλια των συνέδρων προς τα παιδιά και τις αντιδράσεις των παιδιών, βρισκόμαστε σε καλό δρόμο. Οι τίτλοι των θεμάτων που παρουσιάστηκαν είναι: «Περιμένοντας τον Ερατοσθένη», «Κωνικές τομές», «Η ηλεκτρική ενέργεια μπορεί να είναι πράσινη», «Παιχνίδια λογικής και στρατηγικής», «Δυαδική αναζήτηση», «Οι πρώτοι αριθμοί στην υπηρεσία της κρυπτογραφίας - RSA», «Βραχυστόχρονο σε κύκλο και ευθεία», «Περίοδος περιστροφής της Γης», «Ελάχιστες επιφάνειες», «Συμμετρία και αναμορφώσεις», «Ηλιακό ρολόι», «Robot σε δράση» Μαθητές: Α΄τάξη: Αλστάφα Ελντόνα Γιαννακοπούλου Μαρία, Γκελοβάνι Μαριάμ, Γκιούλ Κλαούντια, Γούλα Ρομάν, Σαρμά Βαρντάν. Β΄τάξη: Σαραφίδου Μαρία Βαρβάρα, Τζούμα Φωτεινή, Χαλάτση Κωνσταντίνα. Γ΄τάξη: Λαγουβάρδος Ιωσήφ, Λογοθέτης Δημήτρης, Μπότσης Νίκος, Φραγκιαδάκης Μιχάλης, Χαντάτ Χρήστος.

100

95

Εκδόσεις της Ελληνικής Μαθηματικής Εταιρείας

100

95

75

75

25

25

5

5

0

0

Τιμή τεύχους: 3€

Τιμή τεύχους: 10€

Τιμή τεύχους: 3€

Τιμή τεύχους: 10€

Τιμή τεύχους: 3,5€

Τιμή τεύχους: 10€

100

100

95

95

75

75

Τιμή βιβλίου: 30€ 25

5

0

Τιμή βιβλίου: 20€

Τιμή βιβλίου: 20€

Κεντρική Διάθεση: Πανεπιστημίου 34 - Αθήνα τηλ.: 210 3616532, 210 3617784 fax: 210 3641025 www.hms.gr e-mail: [email protected]

25

5

0

EYKLEIDHS_B_T90 2013-2014.pdf

Whoops! There was a problem loading more pages. EYKLEIDHS_B_T90 2013-2014.pdf. EYKLEIDHS_B_T90 2013-2014.pdf. Open. Extract. Open with. Sign In.
Missing:

6MB Sizes 0 Downloads 97 Views

Recommend Documents

No documents